PANCE PREP QUESTIONS Flashcards

1
Q

A 15-year-old male with a recent history of camping presents after 5 days of flatulence and greasy, foul-smelling diarrhea. The patient also reports nausea, weight loss, and abdominal cramps which precedes the sudden diarrhea. He denies tenesmus, urgency, or bloody diarrhea. What is the patient most likely to report about his camping activities?

A. Collecting water from a stream, without boiling or chemical treatment
B. Recent antibiotic prescription
C. Symptoms have been going on for months
D. The patient camped as a side-excursion from a cruise ship
E. The patient camped in Mexico

A

Answer: A. Collecting water from a stream, without boiling or chemical treatment

EXPLANATION
Giardiasis is an infection of Giardia lamblia, a protozoan parasite that can cause epidemic or sporadic diarrhea. The major site of infection is the small intestine. While the exact etiology is uncertain, diarrhea may be a result of both intestinal malabsorption and hypersecretion. The small intestine is the site of the major structural and functional abnormalities associated with giardiasis

Giardia lamblia is a significant cause of diarrhea in hikers and campers in wilderness areas, who drink water that has not been adequately boiled or filtered.
The most common symptoms of acute giardiasis are the sudden onset of diarrhea, malaise, foul-smelling fatty stools, abdominal cramps, flatulence, nausea, and weight loss. Vomiting and fever are less common symptoms. Symptoms usually present after a 7-14 day incubation period and may last from 2-4 weeks. Complete recovery can take many weeks, even after clearance of the parasite. [3]

How well did you know this?
1
Not at all
2
3
4
5
Perfectly
2
Q

A 67 year-old male who was recently diagnosed with colorectal cancer presents with fever and shortness of breath that started 2 days prior to presentation. Patient also noticed that his hands are “more red than usual” and there are painful lesions on his fingers. Cardiovascular examination is remarkable for a holosytolic, “blowing” murmur best heard in the mitral area which was not present during prior examinations. Preliminary results of the blood culture is positive for a gram positive cocci. Which of the following organism is most likely responsible for patient’s disease?

A. Staphylococcus aureus
B. Streptococcus viridians
C. Staphyloccus epidermidis
D. Streptococcus bovis
E. Eikenella corrodens
A

Answer: D. Streptococcus bovis.

The hand erythema is likely Janeway lesions (erythematous non-tender lesions on palms and soles) and the tender lesions on the fingers are Osler’s nodes. Given the above findings and a new murmur on physical examination, the most likely diagnosis for this patient is infective endocarditis. Endocarditis is inflammation of the endocardium that lines the surface of cardiac valves; usually due to bacterial infection. Streptococci and staphylococci account for majority (80%) of cases of infective endocarditis. Common organisms responsible for infective endocarditis include the following:

Streptococcus viridians (Choice B) is the most common overall cause. It is a low-virulence organism that infects previously damaged valves (e.g. chronic rheumatic heart disease and prolapsed mitral valve).

Staphylococcus aureus (choice A) is the most common cause in IV drug abusers. S. aureus is a high-virulence organism that infects normal valves, most commonly the tricuspid valve.

Staphyloccus epidermidis (choice C) is associated with endocarditis of prosthetic valves.

Streptococcus bovis (Choice D) is associated with endocarditis in patients with underlying colorectal carcinoma. Given our patient was recently diagnosed with colorectal cancer, the most likely causative organism is S. bovis.

How well did you know this?
1
Not at all
2
3
4
5
Perfectly
3
Q

A 46-year-old woman wanders into the clinic, rambling incoherently. When questioned, she has
some difficulty remembering what she was asked. She exhibits some perceptual disturbances
and is not oriented to time. Which of the following is the most likely diagnosis?

(A) Bipolar disorder 
(B) Delirium 
(C) Major neurocognitive disorder (dementia) 
(D) Paranoid personality disorder 
(E) Schizophrenia
A

Answer: B - Delirium

How well did you know this?
1
Not at all
2
3
4
5
Perfectly
4
Q

Six days ago, a 2-year-old boy had a temperature of 40.0°C (104.0°F). No specific cause was
found. His fever has persisted, and he now has injected conjunctivae, strawberry tongue, dry
fissured lips, erythema and desquamation of his hands and feet, and bilateral cervical
adenopathy. Which of the following is the most likely complication of this condition?

(A) Chorea 
(B) Heart failure 
(C) Coronary artery aneurysm 
(D) Mesenteric arteritis 
(E) Valvular heart disease
A

Answer: C - Coronary Artery Aneurysm = deduce that the patient has Kawasaki syndrome: most common complication = coronary artery aneurysm

  • Kawasakis = think warm CREAM
    ⦁ fever (warm)
    ⦁ Conjunctivitis (bilateral & nonexudative) +/- photophobia
    ⦁ Rash = polymorphous
    ⦁ Extremity changes (desquamation - especially perineum), edema, erythema of palms/soles, arthritis
    ⦁ Adenopathy = cervical lymphadenopathy
    ⦁ Mucous membrane = lip swelling, lip fissures, strawberry tongue
  • Unidentified respiratory agent or viral pathogen that has a propensity towards vascular tissue –> necrotizing vasculitis –> coronary vessel arteritis, CORONARY ARTERY ANEURYSM, MI, pericarditis, myocarditis, peripheral arterial occlusion
  • TX = IVIG + high dose aspirin; steroids if refractory
How well did you know this?
1
Not at all
2
3
4
5
Perfectly
5
Q

A 31-year-old African American woman has had worsening malaise, dyspnea, and low-grade
fever for the past four weeks. She was recently treated for iritis. On physical examination, red
nodules over the anterior lower legs and mild hepatomegaly are noted. Chest x-ray study shows
bilateral hilar adenopathy. Which of the following is the most likely diagnosis?

(A) Lymphoma 
(B) Mesothelioma 
(C) Sarcoidosis 
(D) Tuberculosis 
(E) Wegener granulomatosis
A

Answer: Sarcoidosis

SARCOIDOSIS
- multisystemic inflammatory disorder of unknown etiology;
- INCREASED PREVALENCE IN AFRICAN AMERICANS, NORTHERN EUROPEANS, FEMALES
- 50% = asymptomatic
- NONCASEATING GRANULOMA
- most common complications = pulmonary
⦁ pulmonary: dry, nonproductive cough, dyspnea, chest pain
⦁ lymphadenopathy - hilar nodes - intrathoracic lymphadenopathy
⦁ skin = 2nd most common organ involved with sarcoidosis = ERYTHEMA NODOSUM = bilateral tender red nodules on anterior legs; may also be seen with coccidiomycosis; maculopapular rash, LUPUS PERNIO = raised discoloration of nose/ear/cheek/chin - resembles frost bite
⦁ PAROTID GLAND ENLARGEMENT
⦁ VISUAL = ANTERIOR UVEITIS, blurred vision, conjunctivitis
⦁ Myocardial = arrhythmias, cardiomyopathies
⦁ Rheumatologic = arthralgias, fever, malaise, weight loss, hepatosplenomegaly
⦁ Neurologic = cranial nerve palsies (usually CN7 - facial nerve)
⦁ HYPERCALCEMIA, increased ACE levels, Calcinosis Cutis

TX = observation; majority of patients have spontaneous remission within 2 years and require no treatment

  • ORAL STEROIDS = treatment of choice when treatment is needed
How well did you know this?
1
Not at all
2
3
4
5
Perfectly
6
Q

For the past three weeks, a 47-year-old man has had the feeling of heaviness in his chest while
pushing his lawn mower. He says he has never been treated for similar symptoms and has no
symptoms now. He smoked one pack of cigarettes daily for 20 years but quit smoking seven
years ago. Findings on physical examination are normal. Electrocardiography shows no
abnormalities. Which of the following is the most appropriate initial diagnostic study?

(A) Coronary arteriography 
(B) Echocardiography 
(C) Exercise stress test 
(D) 24-Hour ambulatory cardiac monitoring 
(E) Myocardial perfusion scan
A

Answer: C - exercise stress test

How well did you know this?
1
Not at all
2
3
4
5
Perfectly
7
Q

A 32-year-old woman has had a rash on her legs for the past three weeks. There are no known infectious contacts. One month ago, she had a bladder infection and was treated with
trimethoprim-sulfamethoxazole. Physical examination shows a diffuse rash on the shins, the left medial ankle, and the right medial calf. The rash is tender, diffuse, and recurs in the same areas. A few of the lesions have the appearance of bruising. Which of the following is the most likely diagnosis?

(A) Erythema multiforme 
(B) Erythema nodosum 
(C) Lichen planus 
(D) Lichen simplex 
(E) Nummular eczema
A

answer: B - erythema nodosum

  • painful, erythematous, inflammatory nodules seen on anterior shins
  • range in colors from pink, red, purple
  • usually bilateral
  • may also occur on other parts of body
  • An idiopathic inflammatory skin condition; commonly associated with
    ESTROGEN EXPOSURE - OCPs, Pregnancy
    ⦁ certain medications = SULFA related drugs, OCPs, estrogen
    ⦁ Infections: MC STREP, TB, Sarcoidosis, Coccidioidomycosis - fungal
    ⦁ IBD (crohns + UC)
    ⦁ leukemia
    ⦁ Behcets

Erythema nodosum is generally self-limiting and resolves spontaneously within a few weeks. Treat underlying cause

  • can give NSAIDS for pain
  • if persistent = give STEROIDS
How well did you know this?
1
Not at all
2
3
4
5
Perfectly
8
Q

A 33-year-old woman at 35 weeks’ gestation has constant pelvic pain. She says that she had a small amount of dark red vaginal bleeding after sexual intercourse. Her blood pressure is 80/50 mmHg, and she has tachycardia. The uterus is firm and tender to palpation. Which of the following is the most likely diagnosis?

(A) Abruptio placentae 
(B) Cervical laceration  
(C) Placenta previa 
(D) Preterm labor 
(E)   Vasa previa
A

Answer: A - abruptio placentae

  • DARK RED vaginal bleeding
  • painful
  • hypotension (80/50)
  • tachycardia

Both vasa previa & placenta previa = lots of bright red blood

cervical laceration, is incorrect because this patient is in the antepartum stage of pregnancy, and cervical laceration typically results from passage of the fetus through the birth canal

preterm labor, is incorrect because there is no history of this patient having any uterine contractions, an obvious sign of preterm labor

vasa previa, is incorrect. This is a condition whereby the umbilical vessels overlay the internal cervical os and may be a cause of third-trimester bleeding. However, this patient does not have copious vaginal bleeding of bright red blood.

How well did you know this?
1
Not at all
2
3
4
5
Perfectly
9
Q

) A 41-year-old woman has a nine-month history of nausea, constipation, dyspepsia, general
fatigue, arthralgia, and increasing memory loss. She has no history of illness other than her
present complaints, and her menses have been regular. Physical examination shows no
abnormalities. Laboratory findings include the following levels:

Serum  
⦁	  Albumin 4.9 g/dL 
⦁	  Calcium  13.0 mg/dL 
⦁	  Chloride 111 mEq/L 
⦁	  Creatinine 1.0 mg/dL 
⦁	  Phosphate 0.3 mg/dL 
⦁	  Blood urea nitrogen  17 mg/dL 

Which of the following is the most likely diagnosis?

(A) Cushing syndrome 
(B) Hyperparathyroidism 
(C) Hypopituitarism 
(D) Malabsorption 
(E) Multiple myeloma
A

Answer: B - Hyperparathyroidism

The patient has the classic signs of Hyperparathyroidism - which include gastrointestinal, musculoskeletal, and neurologic abnormalities. Additionally, the laboratory values clearly show hypercalcemia and hypophosphatemia, both of which indicate hyperparathyroidism.

Cushing syndrome, is incorrect because the patient does not have the typical
symptoms of Cushing syndrome, which include oligomenorrhea or amenorrhea and a host of
changes to the body habitus. Option (C), hypopituitarism, is incorrect because the patient does
not have severe metabolic, growth, and menstrual abnormalities as a result of a decrease in
pituitary hormone levels. Option (D), malabsorption, is incorrect because there is no indication in the patient’s history of weight loss, other illness, or abnormal menses. In addition, the laboratory values do not support a diagnosis of malabsorption.

Option (E), multiple myeloma, is incorrect because the patient does not have anemia, bone pain, proteinuria, and renal failure, which are characteristic of this disease. Although the patient does have hypercalcemia, which is another characteristic of multiple myeloma, none of the other presenting symptoms support this diagnosis. In addition, multiple myeloma is most frequently diagnosed in individuals aged 65 to 74 years, and the patient described is significantly younger.

How well did you know this?
1
Not at all
2
3
4
5
Perfectly
10
Q

A 40-year-old man is being examined because he feels weak and tired and has had symptoms of
sexual dysfunction. His blood pressure is 100/60 mmHg. Physical examination shows decreased
facial, axillary, and pubic hair. Laboratory studies show decreased serum levels of luteinizing and
follicle-stimulating hormones, thyroxine, and testosterone; serum thyroid-stimulating hormone
level is within normal limits. Which of the following is the most likely diagnosis?

(A) Diabetes insipidus 
(B) Diabetes mellitus 
(C) Hyperprolactinemia 
(D) Hypopituitarism 
(E) Hypothyroidism
A

Answer: D - Hypopituitarism

Option (A), diabetes insipidus, is incorrect because the patient does not have urinary symptoms or signs of dehydration, and the laboratory findings are not consistent with this condition.

Option (B), diabetes mellitus, is incorrect because diabetes mellitus is a metabolic disorder, specifically affecting carbohydrate metabolism. It is a disease characterized by persistent hyperglycemia. There is no mention of serum glucose findings in the history or laboratory studies of this patient.

Option (C), hyperprolactinemia, is incorrect because there is no evidence of increased serum
prolactin levels or galactorrhea in the patient. Option (E), hypothyroidism, is also incorrect.
Weakness and fatigue are symptoms of hypothyroidism, but the normal thyroid-stimulating
hormone level rules out a thyroid disorder in this patient.

The correct answer is Option (D), hypopituitarism,
based on the common symptoms of gonadotropin deficiency, weakness, fatigue, and sexual
dysfunction. The decreased levels of luteinizing and follicle-stimulating hormones indicate
hypogonadism. These laboratory findings, in addition to the decreased levels of testosterone,
indicate hypopituitarism.

How well did you know this?
1
Not at all
2
3
4
5
Perfectly
11
Q

A 22-year-old woman is brought to the emergency department for evaluation three hours after falling backward out of a chair and striking her head on a carpeted floor. She has tenderness over the back of her head but does not report loss of consciousness. Physical examination, including neurologic and musculoskeletal evaluation, shows no abnormalities except a 2-cm area of swelling and minimal abrasion overlying the left occiput. Which of the following is the most appropriate next step?

(A) Anteroposterior and lateral x-ray studies of the skull
(B) CT scan
(C) Discharge and observation by family or friends
(D) Hospital admission for observation
(E) MRI

A

Answer: (C), discharge and observation by family or friends.

Patients with minor head injuries can be discharged with observation instructions in the care of a reliable adult.

Option (B), CT scan, is incorrect because this patient does not meet the criteria for CT scan. These criteria include: instability after multiple traumas, unreliable history or examination because of possible alcohol use or drug ingestion, loss of consciousness for longer than five minutes, repeated vomiting or vomiting for more than eight hours after injury, post-traumatic seizures, progressive headache, physical signs of basilar skull fracture, or amnesia.

Option (D), hospital admission for observation, is incorrect because this patient does not meet
the criteria for hospitalization. These criteria include: coma, underlying pathology such as
coagulopathy and/or hydrocephalus, unreliable history or examination because of possible
alcohol use or drug ingestion, documented loss of consciousness for longer than five minutes,
severe and persistent headache, protracted vomiting, suspected child abuse, unreliable
caregiver, altered mental status or seizures, and focal neurologic deficit.

How well did you know this?
1
Not at all
2
3
4
5
Perfectly
12
Q

A previously healthy 15-month-old boy becomes anxious and begins crying and drooling
copiously. A few minutes earlier he had been calmly playing with his toys. Temperature is 36.7°C (98.1°F), pulse rate is 84/min, and respirations are 18/min. On physical examination, the posterior pharynx is mildly injected but otherwise clear. The lungs are clear to auscultation and percussion. Findings on chest x-ray study are normal. Within an hour he is calmer, but he
continues to drool heavily. Which of the following is the most appropriate next step?

(A) Administration of syrup of ipecac 
(B) Barium swallow x-ray study 
(C) Chest physiotherapy 
(D) Esophagogastroduodenoscopy 
(E) Insertion of a nasogastric tube
A

Answer: D - Esophagogastroduodenoscopy

This question tests the examinee’s ability to recognize foreign body ingestion and to determine
the appropriate next step in management and evaluation. The correct answer is Option (D),
esophagogastroduodenoscopy, which is the appropriate step to confirm a diagnosis and provide therapeutic treatment.

Option (A), administration of syrup of ipecac, is incorrect and may be dangerous. It is not
recommended that health care providers or parents attempt to dislodge a foreign body from a
spontaneously breathing patient by administering syrup of ipecac. Option (B), barium swallow
x-ray study, is incorrect because the barium may obscure the view of the foreign body. This
intervention is also contraindicated if esophageal perforation cannot be ruled out as a possible
diagnosis. Option (C), chest physiotherapy, is incorrect and not recommended as an intervention because it will most likely be ineffective or could potentially lead to aspiration of the foreign body. Option (E), insertion of a nasogastric tube, is also incorrect because this intervention does not have any diagnostic or therapeutic value in patients who ingest foreign bodies.

How well did you know this?
1
Not at all
2
3
4
5
Perfectly
13
Q

A mother brings her 6-day old infant to ED with fever & skin rash. fever began 2 days ago: reached 102. Irritable, numerous erythematous bullae across chest/abdomen/extremities. Nikolsky’s sign is present. No meds for infant

a) Pemphigus Vulgarus
b) SSSS - Staphylococcal scalded skin syndrome
c) Impetigo
d) Rubella
e) Erysipelas

A

Answer: SSSS

How well did you know this?
1
Not at all
2
3
4
5
Perfectly
14
Q

32 YO female complains of “acne” & persistent cough for last several months. She has tried OTC meds for her cough and acne, but both have gotten worse. Normal vital signs. PE = multiple flesh colored papules ranging from 2-8mm involving the alae nares, eyelids, forehead and rear of neck and hairline. Cervical lymphadenopathy. CXR = hilar adenopathy & pulmonary reticular opacities. What is appropriate treatment?

a) Albuterol
b) Prednisone
c) Dextromethorphan
d) Sulfasalazine
e) Topical Clindamycin/benzoyl peroxide

A

Answer: Prednisone

  • Patient has sarcoidosis - Lupus Pernio = occurs in sarcoidosis
  • patient also may have erythema nodosum
  • albuterol if asthma; dextromethorphan = cough suppressant. Sulfasalazine = IBD & RA. Clinda/benzo if actually was acne
How well did you know this?
1
Not at all
2
3
4
5
Perfectly
15
Q

A 45-year-old man with history of previously well-controlled hypertension comes in because colleagues at his office are telling him that his appearance is changing. The patient has recently noted that his old shoes felt too tight and he had to buy new, larger shoes. He also complains of shooting pains in his hands which his company nurse told him were consistent with carpal tunnel syndrome. Review of systems reveals increased thirst and urination. His temperature is 37.0°C (98.6°F), blood pressure 190/90 mm Hg, pulse 73/min, and respirations 13/min. Physical examination shows frontal bossing, mandibular enlargement, and widening of the spaces between his teeth. His hands and feet seem larger and more out of proportion to his body size than usual.

Which of the following is the most appropriate diagnostic study at this time?

a) cortisol level
b) CT of brain
c) fasting glucose level
d) growth hormone
e) insulin-like growth factor

A

Answer = E: insulin-like growth factor

The correct answer is E. This patient has acromegaly, an excess of growth hormone, evidenced by increased hand and foot size and coarse facial features with a protruding jaw. Other features, such as tooth-space widening and macroglossia, can be present. Impaired glucose tolerance is often present, as is diabetes. Hypertension is present in 1/3 of these patients. Complaints of headache and visual field losses can occur. Patients may have carpal tunnel syndrome. The test of choice for diagnosis is insulin-like growth factor, which is elevated >5 times the normal limit. Another test which can be used for diagnosis is measurement of growth hormone after the administration of 75 g of glucose. In a normal patient, the growth hormone level would be suppressed, but in a patient with acromegaly, the levels remain high (greater than 5 ng/mL).

Growth hormone levels (choice D) should not be used as a screening test to diagnose acromegaly, because patients have a normal daily fluctuation of gonadotropin-releasing hormone (GNRH) and growth hormone. Growth hormone levels can also fluctuate with exercise and acute illness. Pregnant and adolescent patients can also have increased levels and not have acromegaly. A lack of the normal nadir may be useful in identifying patients who have acromegaly, because these patients will not have full suppression of growth hormone throughout the 24-hour period.

Cortisol levels (choice A) might be helpful as part of the diagnosis of Cushing syndrome. Cushing syndrome can be associated with insulin resistance, hypertension, and weight gain but doesn’t share other features associated with acromegaly.

How well did you know this?
1
Not at all
2
3
4
5
Perfectly
16
Q

A young mother brings her 6-month-old firstborn son for a health supervision visit. She states that the baby has been doing well and she has not noticed any abnormalities in his development, though her experience is really not very significant, she says. The boy is breastfeeding and is increasing the number of solid foods he eats with each passing day. He sleeps almost through the night and stopped having colic 2 months earlier. On physical examination the boy appears to be in no distress. He is in the 50th percentile for weight and length and his vital signs are normal. The health care provider performs a thorough neurologic exam in front of the mother, who is very curious about everything the health care provider is doing. On extension of the head, the boy flexes his arms and legs. When a finger is put in his palm, he flexes his hand, elbow and shoulder ipsilaterally. Tactile stimulation of the cheek prompts him to turn his mouth to that side, and touching the dorsum of his foot leads to a step-up movement. During a simulated fall, the baby reacts by extending his arms. The mother asks about each and every one of the newborn reflexes and when they are supposed to disappear. Which of the following is likely to be the one reflex that persists throughout life?

a) Extension of the head leads to flexion of the arms and legs
b) Finger in palm leads to flexion of the hand, elbow, and shoulder ipsilaterally
c) Stimulation of a fall leads to extension of arms
d) Stimulation of the cheek prompts turning of mouth to that side
e) Stimulation of the dorsum of the foot leads to a step-up movement

A

Answer: C - Stimulation of a fall leads to extension of arms
- the rest of the reflexes above disappear around age 4-6 months

How well did you know this?
1
Not at all
2
3
4
5
Perfectly
17
Q

A 40-year-old woman is brought to the emergency department following a suicide attempt with imipramine (Tofranil). Her fiancé found her unresponsive, with an empty bottle of the imipramine at her side. The imipramine had been his, and the prescription had been filled that morning. Her past medical history is significant for hypertension, atrial fibrillation, diabetes, and asthma. Her medications include furosemide, procainamide, glyburide, prednisone, and albuterol. She has no known drug allergies. She is afebrile, has a blood pressure of 100/60 mm Hg, pulse of 62/min, and respirations of 22/min. A gastric lavage yields multiple pill fragments. She is confused and somnolent, and has shallow respirations. Her physical examination is otherwise unremarkable. On an electrocardiogram, which of the following abnormalities would most likely reflect possible cardiac toxicity?

a) Left deviation of the QRS axis
b) Prolongation of the QT interval
c) Shortening of the PR interval
d) ST-segment depression
e) T-wave inversion

A

Answer: B - Prolongation of the QT interval

The correct answer is B. A prolongation of the QT interval is highly predictive of both cardiac and CNS toxicities from tricyclic antidepressant ingestion. This medication has high lethality associated with its overdose because of its cardiac effects.

How well did you know this?
1
Not at all
2
3
4
5
Perfectly
18
Q

A patient with systemic lupus erythematosus (SLE) very much wants to become pregnant. What should her health care provider tell her regarding pregnancy?

a) There is no increased risk to the fetus
b) There is an increased risk for cardiovascular malformations
c) There is an increased risk for nervous system malformations
d) There is an increased risk for renal malformations
e) There is an increased risk for spontaneous abortions and prematurity

A

answer: E - There is an increased risk for spontaneous abortions and prematurity

Patients who have SLE have an increased incidence of spontaneous abortion, fetal death in utero, and prematurity

How well did you know this?
1
Not at all
2
3
4
5
Perfectly
19
Q

A 54-year-old woman comes to the emergency department with severe left-sided flank pain that is referred to the left labia majora. She is very uncomfortable lying still on the stretcher and is continuously shifting, trying to find a comfortable position. A nonenhanced helical CT scan shows a 10-mm ureteral stone at the ureteropelvic junction. She has a normal coagulation profile. Which of the following would most likely be the best therapy in this case?

a) Plenty of fluids and analgesics and await spontaneous passage
b) Extracorporeal shock wave lithotripsy (ESWL)
c) Endoscopic retrograde basket extraction
d) Endoscopic retrograde laser vaporization of the stone
e) Open surgical removal

A

answer: B - Extracorporeal shock wave lithotripsy (ESWL)

Extracorporeal shock wave lithotripsy (ESWL) is the most commonly used method to fragment urinary stones and allow their passage. Pregnancy and coagulation problems are contraindications. The first one is ruled out by her age; the second one we have been told she does not have. Because the stone is 10 mm, it is less likely to pass on its own because of its large size.
Waiting for spontaneous passage (choice A) would have been perfect for a much smaller stone (3 mm) that had already negotiated most of the ureter. A 7-mm stone way up at the ureteropelvic junction has a very small chance of spontaneous passage.
Retrograde endoscopic approaches (choices C and D) are more invasive than ESWL. They would not be the first choice for this scenario.

Open surgical removal (choice E) would have been good for a much bigger stone. A huge target (3 cm or larger) could indeed be fragmented by ESWL, but then we would be contending with dozens of still very large stones. In those cases, a direct approach to extract the huge intact stone would work better.

How well did you know this?
1
Not at all
2
3
4
5
Perfectly
20
Q

A 49-year-old man is recuperating in a hospital after having sustained a recent cerebrovascular accident that damaged part of his right temporal lobe. His wife and children are at his bedside day and night. They want to actively participate in his rehabilitation but specifically ask the health care provider what they can expect from him in the future. Once the patient has recuperated from the immediate effects of his stroke, to which of the following psychiatric disorders will he be most predisposed?

a) Conversion disorder
b) Narcolepsy
c) Major depressive disorder
d) Schizophrenia
e) Substance abuse

A

answer: C - Major Depressive Disorder

The correct answer is C. Any event that affects the vasculature, such as a myocardial infarction or a cerebrovascular accident (CVA), has been shown to increase the risk of major depressive disorder in the months that follow. The pathophysiology of such a development is unclear, but is thought to be related to the effects of serotonin on vascular physiology.

How well did you know this?
1
Not at all
2
3
4
5
Perfectly
21
Q

A patient who is being treated for hypertension related to a myocardial infarction that occurred 2 hours ago is medicated with IV nitroprusside (Nipride). Which of the following is the expected action of this drug?

a) Constriction of arterioles alone
b) Constriction of both arterioles and venules
c) Constriction of venules alone
d) Dilatation of arterioles alone
e) Dilatation of arterioles and venules

A

answer: E - Dilatation of arterioles and venules

How well did you know this?
1
Not at all
2
3
4
5
Perfectly
22
Q

A 63-year-old woman comes to the emergency department with severe abdominal pain. She reports the pain is epigastric and worse with eating. There is some radiation to the back. There is associated diarrhea that is described as “fatty.” Symptoms have been present for 3 to 4 months. Her medical history is significant for 2 episodes of acute pancreatitis, hypertension, and hyperlipidemia, but she takes no medications for these disorders. Her temperature is 38.3ºC (100.9ºF), blood pressure 160/90 mm Hg, pulse 143/min, and respirations 16/min. Physical examination shows epigastric tenderness, mild distention, and decreased bowel sounds. She appears uncomfortable and in moderate distress. An obstruction series reveals only scattered pancreatic calcifications. Laboratory studies show:

Amylase 45 U/L (normal 23-140 U/L)
Lipase 10 U/L (normal <160 U/L)
Albumin 2.8 g/dL (normal 3.4-5.4 g/dL)
AST 98 IU/L (normal <40 IU/L)
ALT 45 IU/L (normal <40 IU/L)
Total bilirubin 1.1 mg/dL (normal <1.9 mg/dL)

Which of the following is the most likely underlying etiology for this patient’s symptoms?

a) Alcohol
b) Elevated triglycerides
c) Gallstones
d) Hypercalcemia
e) Idiopathic

A

answer: A - Alcohol

This patient has chronic pancreatitis. The finding of calcifications in the pancreas is a good clue to this patient’s diagnosis. Amylase and lipase can be normal or elevated, depending on how much intact pancreas is left. Seventy percent of chronic pancreatitis is caused by alcohol ingestion. Our patient’s malnutrition, as evidenced by her low albumin and her AST to ALT ratio of 2:1, is consistent with alcoholic hepatitis. The treatment is pain relief, a low-fat diet with medium chain triglycerides, vitamins, and pancreatic enzyme replacement. The remaining 30% of chronic pancreatitis is idiopathic (choice E).

  • **AST : ALT > 2 = ALCOHOL HEPATITIS
  • **ALT > AST = think LIVER: viral / toxic / inflammatory processes
  • **ALT > 1000 = autoimmune hepatitis
Elevated triglycerides (choice B) are a rare cause of acute pancreatitis, not chronic pancreatitis. Triglyceride levels in these patients are usually >1,000 mg/dL.
Gallstones (choice C) are an important cause of acute pancreatitis but are not typically associated with chronic disease. Patients who have gallstone pancreatitis are usually overweight, fertile, and in their forties. Correction is by ERCP removal or laparoscopic cholecystectomy with common duct exploration and removal of the choledocholithiasis.
Hypercalcemia (choice D) is a rare cause of acute pancreatitis. Pancreatic calcifications seen on obstruction series do not represent elevated serum calcium levels. Hypocalcemia can complicate acute pancreatitis as a result of saponification of fats, which takes place due to pancreatic insufficiency because of acute pancreatitis.
How well did you know this?
1
Not at all
2
3
4
5
Perfectly
23
Q

A 20-year-old woman comes to the health care provider because of left lower quadrant pain for 2 months. She states that she first noticed the pain 2 months ago but now it seems to be getting worse. She has had no changes in bowel or bladder function. She has no fever or chills and no nausea, vomiting, or diarrhea. The pain is intermittent and sometimes feels like a dull pressure. Pelvic examination is significant for a left adnexal mass that is mildly tender. Urine hCG is negative. Pelvic ultrasound shows a 7-cm complex left adnexal mass with features consistent with a benign cystic teratoma. Which of the following is the most appropriate next step in management?

a) Repeat pelvic examination in 1 year
b) Repeat pelvic ultrasound in 6 weeks
c) Prescribe the oral contraceptive pill
d) Perform hysteroscopy
e) Perform laparotomy

A

Answer: E - Perform Laparotomy

To repeat pelvic examination in 1 year (choice A) would not be correct management. This patient is symptomatic with a 6-cm ovarian mass that is at risk for torsion. She therefore should be managed surgically.
To repeat pelvic ultrasound in 6 weeks (choice B) is appropriate for some adnexal masses. For example, in a young woman who has a small complex cyst that appears consistent with a corpus luteum, it may be most prudent to recheck an ultrasound in 6 weeks to see if the cyst has resolved. This patient, however, is symptomatic with a 6-cm cyst that appears to be a dermoid, which will not resolve spontaneously. She therefore requires surgery.
To prescribe the oral contraceptive pill (choice C) may help to prevent future ovarian cysts, but it will not resolve this cyst, which requires surgical management.
To perform hysteroscopy (choice D) would not be indicated. Hysteroscopy is used to evaluate the uterine cavity and fallopian tubes and would not be used for management of an adnexal mass.

How well did you know this?
1
Not at all
2
3
4
5
Perfectly
24
Q

A 30-year-old woman with a history of tonic-clonic seizures complains of double vision, thickened gums, and growth of facial hair since starting a new medication. Which of the following anticonvulsant medications is most likely responsible for her symptoms?

a) Carbamazepine (Tegretol)
b) Ethosuximide (Zarontin)
c) Phenobarbital (Phenobarb)
d) Phenytoin (Dilantin)
e) Valproic acid (Depakene)

A

Answer: D - Phenytoin

How well did you know this?
1
Not at all
2
3
4
5
Perfectly
25
Q

A 45-year-old male comes to the emergency room after being involved in a head-on motor vehicle accident earlier in the day. The patient notes that he struck his head, but he did not experience any loss of consciousness. His blood pressure is 190/110, his respirations are irregular, and his electrocardiogram (EKG) shows sinus bradycardia with a heart rate of 42 beats per minute. The patient’s symptoms are part of which clinical triad?

Beck’s triad
Charcot’s triad
Bergman’s triad
Cushing’s triad

A

Answer: D - Cushing’s triad

  1. A change in respirations, often irregular and deep, such as cheyne stokes.
  2. A widening pulse pressure (the difference between the Systolic and the Diastolic BP
  3. Bradycardia (slow heart rate).
    - indicates increased ICP
How well did you know this?
1
Not at all
2
3
4
5
Perfectly
26
Q

All of the following are minor manifestations of acute rheumatic fever as described by the modified Jones criteria EXCEPT

Erythema marginatum
Leukocytosis
Elevated erythrocyte sedimentation rate (ESR)
arthralgia

A

Answer: A - Erythema Marginatum

Jones Criteria (JONES acronym = major criteria)
⦁ Joint (migratory polyarthrtitis) - 2 or more joints affected; medium/large = MC
⦁ Oh my heart (carditis) - mitral/aortic, myocarditis, pericarditis
⦁ Nodules (subcutaneous nodules) - rare - seen over joints, scalp & spinal column
⦁ Erythema Marginatum - non-itchy annular rash with round sharp borders on trunk/extremities, NOT on face
⦁ Sydenham’s Chorea

Jones Criteria (minor criteria)
⦁	fever, arthralgia, increased ESR/CRP/leukocytosis
How well did you know this?
1
Not at all
2
3
4
5
Perfectly
27
Q

A 19-year-old woman comes to the office complaining of a painful rash on her elbows and knees. The rash appears as raised erythematous areas topped with silvery, scaling skin. She reports, “The rash is very itchy.” She had similar symptoms several weeks before, but they spontaneously resolved without treatment. Which of the following is most likely to be the diagnosis?

a) Impetigo
b) Tinea corporis
c) Rosacea
d) Psoriasis

A

Answer: D - Psoriasis

How well did you know this?
1
Not at all
2
3
4
5
Perfectly
28
Q

During a colonoscopy, the gastroenterologist notices that the patient’s colon wall has a “cobblestone” appearance. Which of the following is the most likely diagnosis?

a) Crohn’s disease
b) Celiac sprue
c) Ulcerative colitis
d) Whipple’s disease

A

Answer: A - Crohns

How well did you know this?
1
Not at all
2
3
4
5
Perfectly
29
Q

You are evaluating an obese 37-year-old female in the ER. She has been complaining of right-sided abdominal pain and excessive flatulence. She normally has the pain after eating, but it usually resolves on its own. This episode has persisted for several hours. On physical examination, you palpate her right-upper quadrant while she takes a deep inspiration. Discomfort during this maneuver is referred to as a positive:

a) Levine’s sign
b) Brudziński’s sign
c) Psoas sign
d) Murphy’s sign

A

Answer: Murphy’s Sign

Murphy’s sign = A positive Murphy’s sign is seen in acute cholecystitis. It is elicited by firmly placing a hand at the costal margin in the right upper abdominal quadrant and asking the patient to breathe deeply. If the gallbladder is inflamed, the patient will experience pain and catch their breath as the gallbladder descends and contacts the palpating hand.

Brudzinski’s sign = to test meningeal irritation - flexion of the neck causes flexion at the hip and knees

How well did you know this?
1
Not at all
2
3
4
5
Perfectly
30
Q

You are acting as the first assist in the operating room, and the surgeon asks you to close an abdominal incision with an absorbable suture material. Based on the following choices, which suture would be your pick?

a) Silk
b) Vicryl
c) Dermabond
d) Nylon

A

Answer: B - Vicryl

How well did you know this?
1
Not at all
2
3
4
5
Perfectly
31
Q

All of the following are symptoms of esophageal achalasia EXCEPT

Dysphagia
Acid reflux
Hematochezia
Chest pain

A

answer: C - Hematochezia

How well did you know this?
1
Not at all
2
3
4
5
Perfectly
32
Q
Which of the following is NOT part of CREST syndrome?
 Calcinosis
 Sclerodactyly
 Esophageal dysmotility
 Solar urticaria
A

answer: d: solar urticaria

How well did you know this?
1
Not at all
2
3
4
5
Perfectly
33
Q

A 20-year-old female recently diagnosed with chlamydia comes to your office for swelling and pain in her knees bilaterally. The most likely diagnosis for this woman’s complaints is

Reiter’s syndrome
Sjögren’s syndrome
Down syndrome
Turner syndrome

A

answer: A - Reiter’s syndrome
- AUTOIMMUNE RESPONSE to an INFECTION in another part of the body
1) ARTHRITIS - asymmetric inflammation
2) CONJUNCTIVITIS / UVEITIS
3) URETHRITIS / CERVICITIS

How well did you know this?
1
Not at all
2
3
4
5
Perfectly
34
Q

A 26-year-old female comes to the ER with complaints of white vaginal discharge and pelvic pain. She admits to having unprotected sex. On physical examination, she has an inflamed cervix and cervical motion tenderness. Which one of the following two-medication pairs should she receive prior to leaving the ER?

Ceftriaxone 250 mg IM and clindamycin 300 mg po
Clindamycin 300 mg po and azithromycin 1 gm po
Ceftriaxone 250 mg IM and azithromycin 1 gm po
Mefoxitin 2 gm IV and azithromycin 1 gm po

A

Ceftriaxone 250 mg IM and azithromycin 1 gm po

How well did you know this?
1
Not at all
2
3
4
5
Perfectly
35
Q

Which of the following cardiovascular disorders is most likely associated with abnormal pupillary responses (Argyll Robertson pupil)?

a. Aortic regurgitation
b. Coarctation of the aorta
c. Thoracic aortic aneurysm
d. Myocarditis

A

Answer: C - The Argyll Robertson pupil (a pupil that constricts with accommodation but not in response to light) is characteristic of central nervous system syphilis that often results in thoracic aortic aneurysm.

How well did you know this?
1
Not at all
2
3
4
5
Perfectly
36
Q

Which of the following cardiovascular disorders is most likely associated with jugular venous distension and exophthalmoses?

A. Massive tricuspid regurgitation
B. Coarctation of the aorta
C. Thoracic aortic aneurysm
D. Myocarditis

A

Answer: A - Massive tricuspid regurgitation produces a markedly elevated venous pressure, usually manifested by a severely engorged (often pulsating) liver. If the venous pressure is sufficiently elevated, exophthalmoses may result.

How well did you know this?
1
Not at all
2
3
4
5
Perfectly
37
Q

Flushing and paling of the nail beds (Quincke pulse) and a bounding radial pulse are typical physical findings found in which of the following cardiovascular disorders?

A. Tricuspid regurgitation
B. Coarctation of the aorta
C. Thoracic aortic aneurysm
D. Aortic regurgitation

A

Answer: D - Aortic Regurgitation

Quincke pulse, which consists of alternate flushing and paling of the skin or nail beds, is associated with aortic regurgitation. Other characteristic features of the peripheral pulse in aortic regurgitation include the waterhammer pulse and pulsus bisferiens.

How well did you know this?
1
Not at all
2
3
4
5
Perfectly
38
Q

Conjunctivitis, urethral discharge, and arthralgia are common symptoms present in which of the following cardiovascular disease?

A. Tricuspid regurgitation
B. Coarctation of the aorta
C. Myocarditis
D. Aortic regurgitation

A

Answer: C - Myocarditis

Conjunctivitis, urethral discharge, and arthralgia are common symptoms present in myocarditis i.e. inflammation of myocardium.

How well did you know this?
1
Not at all
2
3
4
5
Perfectly
39
Q

A patient arrives with short stature, webbed neck, low-set ears, and epicanthal fold. Which of the following cardiovascular disorders he is most likely suffering from?

A. Tricuspid regurgitation
B. Coarctation of the aorta
C. Myocarditis
D. Aortic regurgitation

A

Answer: B - Short stature, webbed neck, low-set ears, and epicanthal folds are common clinical findings found in coarctation of the aorta which is a narrowing of part of the aorta (the major artery leading out of the heart). It is a type of birth defect.

How well did you know this?
1
Not at all
2
3
4
5
Perfectly
40
Q

Which of the following disorders are usually characterized by weight loss, depressed mood, feeling of tired, fatigue and frequent, nonspecific abdominal pain?

A. Pheochromocytoma
B. Pancreatic carcinoma
C. Adrenocortical insufficiency
D. Cushing syndrome

A

Answer: B - Pancreatic carcinoma should always be considered in depressed middle-aged patients. It’s symptoms include weight loss, abdominal pain, apathy, decreased energy, lethargy, anhedonia, and depression.

How well did you know this?
1
Not at all
2
3
4
5
Perfectly
41
Q

Release of which of the following hormones is inhibited by insulin?

A. Glucagon
B. ADH
C. Beta cells
D. Somatostatin

A

Answer: A - Glucagon

Increases in insulin levels inhibit glucagon release from α-cells of the pancreas. This paracrine effect is a basic element in insulin’s control of glucagon’s activation of both hepatic gluconeogenesis and lipolysis in adipose tissue.

How well did you know this?
1
Not at all
2
3
4
5
Perfectly
42
Q

Which of the following metabolic effects is caused by insulin release?

A. Increased breakdown of fats
B. Increased breakdown of proteins
C. Decreased blood sugar
D. Causes glucose to be phosphorylated in kidney

A

Answer: C - One of the key metabolic effects of insulin is to reduce blood glucose (sugar) levels. The deficiency of insulin or resistance to its effect, thus, leads to diabetes mellitus.

How well did you know this?
1
Not at all
2
3
4
5
Perfectly
43
Q

All of the following statements are true about Type I diabetes mellitus EXCEPT:

A. It may be linked to autoimmune issues
B. Onset is usually prior to age 20
C. Beta islet cells are destroyed
D. Insulin injections are unnecessary

A

D. Insulin injections are unnecessary

Answer: D - As Type 1 diabetes mellitus is caused by destruction of Beta islet cells, a deficiency of insulin results. Therefore, treatment plans involve giving insulin injections to such patients to help normalize their insulin levels.

How well did you know this?
1
Not at all
2
3
4
5
Perfectly
44
Q

Which of the following is NOT a complication of chronic diabetes mellitus?

A. Atherosclerosis
B. Neuropathy
C. Glaucoma
D. Hypotension

A

Answer: D - Hypertension, not hypotension, is considered as one of the common complications of long-term diabetes mellitus.

How well did you know this?
1
Not at all
2
3
4
5
Perfectly
45
Q

A 25-year-old African American woman comes to the clinic because of lesions on her legs. History reveals the lesions are painful, red nodules that have been present for the past five days. She also reports increased fatigue, unintentional weight loss, painful joints and blurry vision. Physical examination reveals a well-developed female with multiple erythematous macules and nodules on her shins. Which is the best initial test to detect the most likely diagnosis?

Abdominal CT
Biopsy of the rash
Chest radiograph
Electrolytes and serum markers
Peripheral blood smear and CBC with differential
A

Answer: B - biopsy the rash

To diagnose the patient with sarcoidosis, a biopsy of a skin lesion or other manifesting granuloma is needed. A biopsy would show non-caseating granulomas, a hallmark of the disease. The skin lesion described is classic for erythema nodosum which typically affects the shins

How well did you know this?
1
Not at all
2
3
4
5
Perfectly
46
Q

A 37-year-old African-American woman comes to the clinic because of fever, anorexia, and shortness of breath. She states that she has these symptoms from time to time, but this time is the worst and has lasted 3 days. Temperature is 36.6°C (97.9°F), pulse is 68/min, respirations are 16/min, and blood pressure is 128/78 mm Hg. Physical examination shows red/purple raised nodules on her shins, anterior uveitis, and an asymmetric smile. Which of the following findings is necessary for the most likely diagnosis of this condition?

Arthropathy
Bilateral hilar adenopathy
Increased serum levels of ACE
Myopathy
Non-caseating granulomas
A

Answer: E - Non-caseating granulomas

Sarcoidosis is a chronic systemic granulomatous disease often involving multiple organ systems. It occurs most often in female African-Americans. Features include constitutional symptoms, erythema nodosum, dyspnea on exertion, anterior uveitis, and Bell’s palsy, amongst others. Non-caseating granulomas are required for the diagnosis of the disease. Arthralgias and myopathies can be present. Bilateral hilar adenopathy is a common finding of the disease and is important in mobidity and mortality. Cardiac involvement is also seen in about 5% of cases, while Increased serum levels of ACE (choice D) are seen in 50% of cases. Treatment of sarcoidosis is generally with a steroid regimen.

How well did you know this?
1
Not at all
2
3
4
5
Perfectly
47
Q

A 35-year-old African American woman comes to the clinic complaining of vision problems, fatigue, and constipation. She reports a history of kidney stones and occasional rash. Her temperature is 37.1°C (98.8°F), pulse is 76/min, respirations are 16/min, and blood pressure is 120/85 mm Hg. Physical examination shows tender red nodules as displayed below. Which of the following enzymes, consistent with her other findings, is most likely over-expressed?

Alpha-1 Hydroxylase
Adenylate cyclase
Alkaline phosphatase
Desmolase
Sodium-potassium ATPase
A

Alpha-1 Hydroxylase

How well did you know this?
1
Not at all
2
3
4
5
Perfectly
48
Q

A 25-year-old African American man comes to the clinic because of cough. He also notices that he’s been becoming short of breath on his daily runs over the last few weeks. He denies any chest pain or wheezing. His temperature is 36.6 °C (97.9°F), pulse is 68/min, respirations are 16/min, and blood pressure is 128/84 mm Hg. Physical examination shows his lungs are clear to auscultation bilaterally. A chest x-ray is obtained and shown below. Which of the following serum laboratory values is most likely to be abnormal?

1,25-dihydroxyvitamin D
Adenosine deaminase
Angiotensin-converting enzyme
Calcium
Quantiferon gold
A

Answer: ACE

calcium is elevated in sarcoidosis, but ACE is elevated in 60-80% of patients with sarcoidosis, hypercalcemia is only found in 10-15% of patients

How well did you know this?
1
Not at all
2
3
4
5
Perfectly
49
Q

A 20-year-old man comes to the clinic because of shortness of breath on exertion. He says he has felt short of breath for three weeks and also had a rash occasionally. His temperature is 36.6 °C (97.9°F), pulse is 68/min, respirations are 18/min, and blood pressure is 130/78 mm Hg. Physical examination shows a raised erythematous macular rash. A chest x-ray shows bilateral hilar lymphadenopathy and his serum angiotensin converting enzyme is elevated. A biopsy of his rash shows multiple non-caseating granulomas with multinucleated giant cells. Which of the following is most likely to be seen on neurological examination?

Facial nerve palsy
Hypothalamic dysfunction
Cerebellar ataxia
Peripheral neuropathy
Papilledema
A

Facial nerve palsy

How well did you know this?
1
Not at all
2
3
4
5
Perfectly
50
Q

A 25-year-old African American man comes to the clinic because of cough for 3 months and intermittent fevers. He also notices that he’s been becoming short of breath on his daily runs over the last few weeks. He denies any chest pain or wheezing. His temperature is 36.6 °C (97.9°F), pulse is 64/min, respirations are 16/min, and blood pressure is 128/76 mm Hg. Physical examination shows an oxygen saturation of 94%. His lungs are clear to auscultation. Chest computerized tomography shows bilateral mediastinal and hilar adenopathy as well as well-defined bilateral nodules. The pathology from the biopsy of a nodule is shown. Which of the following is the most appropriate treatment for this condition?

Albuterol
Chemotherapy
Itraconazole
Prednisone
RIPE
A

Prednisone

How well did you know this?
1
Not at all
2
3
4
5
Perfectly
51
Q

A 52-year-old male presents complaining of urinary frequency, with hesitancy, and nocturia for the past few months. During his physical examination, you note a nontender, non-enlarged prostate with an isolated right posterior lobe nodule. Which of the following options is most appropriate?

A. Order a serum acid phosphatase level
B. Initiate prazosin and schedule a follow-up appointment in 6 weeks
C. Refer the patient for an ultrasound of the prostate and order a PSA level
D. Reassure the patient and schedule a follow-up appointment in six months
E. Initiate norfloxacin therapy for 7 days and schedule follow-up in two weeks

A

Answer: C

Refer the patient for an ultrasound of the prostate and order a PSA level

This patient has an isolated nodule of the prostate gland — cancer until proven otherwise. You should order an ultrasound and a PSA. BPH will present as a diffuse enlargement, and not a discrete nodule.

How well did you know this?
1
Not at all
2
3
4
5
Perfectly
52
Q

Which term is used to describe the characteristic concave or “spoon-shaped” nails of iron deficiency anemia?

A. leukonychia
B. koilonychias
C. clubbing
D. onycholysis
E. paronychia
A

Answer: B
Koilonychia

Koilonychia is a spoon-shaping of the nail itself. It is usually a result of iron deficiency anemia. Leukonychia is often associated with hypoalbuminemia that causes partial or complete white discoloration of the nails. Leukonychia may also appear as a rare side effect of systemic chemotherapy in some oncological patients but may also be present with arsenic poisoning, renal failure pneumonia, or heart disease. Clubbing of the nails is an actual thickening or elevation of the nail bed - it is a sign of a release of TNF associated with pulmonary disorders (tissue necrosis factor) typically found in bronchiectasis, lung cancers and cystic fibrosis (the nails are NOT necessarily cyanotic.) Onycholysis is a painless separation of the nail from the nail bed. Several or all nails are usually affected - there are many causes. Paronychia is an infection of the nail bed and nail margin, usually from trauma or more commonly, nail-biting.

How well did you know this?
1
Not at all
2
3
4
5
Perfectly
53
Q

Which of the following is NOT a characteristic feature of nephrotic syndrome?

A. proteinuria
B. hematuria
C. hypoalbuminemia
D. hyperlipidemia
E. generalized edema
A

Answer: B
Hematuria

Hematuria is present in NEPHRITIC syndrome. Nephrotic syndrome characteristically includes proteinuria (>3.5 gm/day), with resulting low serum albumin, hyperlipidemia, hypertension, hypercoagulability, and generalized edema (from oncotic third-spacing)

How well did you know this?
1
Not at all
2
3
4
5
Perfectly
54
Q

A patient describes a history of recurrent bouts of uveitis. Her chemistry panel reveals elevated serum calcium and uric acid levels. Her anergy screen is negative. Her chest x-ray demonstrates bilateral hilar adenopathy. Which diagnosis is most likely?

A. Silicosis
B. Sarcoidosis
C. Alpha-1 antitrypsin deficiency
D. Histoplasmosis
E. Tuberculosis
A

Answer: B
Sarcoidosis

Sarcoidosis typically presents with hilar lymphadenopathy and noncaseating granulomas of the lungs (and other organs). In addition, patients may have eye involvement (uveitis). Elevations of ACE, Calcium and uric acid are frequently seen.

How well did you know this?
1
Not at all
2
3
4
5
Perfectly
55
Q

A 34-year-old female presents complaining of symmetrical redness and swelling of the small joints of her hands (PIPs and MCPs). She has noted that the symptoms are worst in the morning. Her erythrocyte sedimentation rate is elevated and her rheumatoid factor is negative. Which of the following diagnosis is most likely?

A. progressive systemic sclerosis
B. CREST syndrome
C. osteoarthritis
D. rheumatoid arthritis
E. ankylosing spondylitis
A

Answer: D
Rheumatoid Arthritis

In spite of the negative RF, Rheumatoid arthritis is the most likely diagnosis. RA characteristically includes small joint symmetrical arthritis, with an elevated ESR (therefore inflammatory, and not OA). 80% of patients with RA will have a positive RF, but 20% will be negative. PSS involves squamous cell thickening and sclerosis causing taut skin of the face and hands and difficulty with esophageal motility. CREST syndrome is a subset of PSS; Ankylosing spondylitis would have an elevated ESR and negative RF, but mainly involves the SI joint and lumbar/thoracic spine fusion (bamboo spine)

How well did you know this?
1
Not at all
2
3
4
5
Perfectly
56
Q

A 27-year-old nulliparous female presents because she’s been trying to get pregnant for two years, but has failed. She relates a history of a misdiagnosis of appendicitis that lead to abscess formation when she was 14 years old. Which of the following diagnostic studies would be most helpful at this point in her evaluation?

A. TSH level
B. hysterosalpingogram
C. laparoscopy
D. PAP smear
E. pelvic ultrasound
A

Answer: B
Hysterosalpingogram

While I would disagree that an invasive procedure like HSG should be done first-line, the thing to remember in this question is that the patient has reason to have tubal scarring from adhesions (and there is no better answer listed to choose), so, for a board exam I would choose this answer. The TSH level would not be indicated (she has not had a pg loss), lap could diagnose the tubal scarring but would be done after an abnormal HSG. Pap smear is screening for cervical cancer and not indicated in this case of infertility, and the pelvic US would yield nothing diagnostically about the tubes.

How well did you know this?
1
Not at all
2
3
4
5
Perfectly
57
Q

A 14-year-old is experiencing a severe asthma attack. Although he is using accessory muscles to breath, auscultation of his chest reveals no audible wheezing. His heart rate is 160 and his respiratory rate is 52. Which of the following arterial blood gases represents the worst prognosis?

A. pH = 7.52; pC02 = 28; p02 = 80
B. pH = 7.44; pC02 = 38; p02 = 70
C. pH = 7.60; pC02 = 18; p02 = 60
D. pH= 7.40; pC02 = 40; p02 = 60
E. pH = 7.27; pC02 - 62; p02 = 64
A

Answer: E
pH = 7.27; pC02 - 62; p02 = 64

This patient has a respiratory rate of 52. If she is ventilating, she is blowing off C02 (an acid) and would be alkalotic and should have a low C02. A pH which is acidic with a pC02 which is elevated means that she is no longer ventilating at all (she needs mechanical ventilation or she will die).

How well did you know this?
1
Not at all
2
3
4
5
Perfectly
58
Q

While suturing a wound, you opt to use lidocaine with epinephrine. The rationale for your choice is:

A. an increase in the absorption of the lidocaine.
B. an increase in the diffusion of the lidocaine into the nerve’s myelin sheath.
C. an increase in the blood flow to the area of injection.
D. an increase in the duration of anesthesia.
E. a decrease in the risk of infection at the site of injection.

A

Answer: D
An increase in the duration of anesthesia

Lidocaine with epinephrine both increases the duration of anesthesia and decreases blood flow to the area of injection-i.e. penis, nose, fingers, toes

How well did you know this?
1
Not at all
2
3
4
5
Perfectly
59
Q

An EKG demonstrates a PR interval of 0.16 seconds, a P to QRS relationship of 1:1, a variable heart rate and an R to R interval that is noted to accelerate ad decelerate during the respiratory cycle. What is the diagnosis?

A. Wenckebach
B. third degree heart block
C. atrial fibrillation
D. sinus arrhythmia
E. atrial flutter
A

Answer: D
sinus arrhythmia

This is sinus arrhythmia. Wenckebach & third-degree AVB would have a nonconducted P wave. A-fib would have no P waves and an irregularly irregular rhythm. A flutter would have “flutter waves” or a regular rhythm of 150.

How well did you know this?
1
Not at all
2
3
4
5
Perfectly
60
Q

A 42-year-old female presents after finding a firm, painless bump in her right eyelid. On examination, you note a 6 mm mass within the tarsus of the right eye. The skin is freely movable over the mass. The remainder of the ophthalmoscopic examination is unremarkable. Which of the following is the most likely diagnosis?

A. Pterygium
B. Chalazion
C. Ectropion
D. External hordeolum
E. Internal hordeolum
A

Answer: B
Chalazion

A chalazion is a painless chronic mass in the eyelid. Hordeolum are acute and red and painful. Pterygium involves the sclera. Ectropion is when the eyelid sags outwardly and the lid doesn’t close well.

How well did you know this?
1
Not at all
2
3
4
5
Perfectly
61
Q

A 22-year-old patient was involved in an automobile accident and is comatose. Which of the following diagnostic modalities would be least useful in this patient’s evaluation?

A. CT scan
B. skull radiographs
C. MRI scan
D. EEG
E. PET scan
A

Answer: B
Skull radiographs

Of these choices, the skull film is LEAST useful. Whether or not the skull is fractured, any LOC in a head trauma requires imaging for bleeding. CT would be the imaging test of choice. MRI should be done for a more chronic bleed. EEG would help to establish brain activity. PET scan (while not done often) would establish physiologic function (uptake of glucose). A positive or negative plain film of the skull would supply NO FURTHER INFORMATION

How well did you know this?
1
Not at all
2
3
4
5
Perfectly
62
Q

Which white blood cell disorder is characterized by the presence of the Philadelphia chromosome in 90% of cases?

A. chronic lymphocytic leukemia (CLL)
B. acute lymphocytic leukemia (ALL)
C. chronic myelogenous leukemia (CML)
D. acute myelogenous leukemia (AML)
E. multiple myeloma
A

Answer: C
Chronic Myeloid Leukemia (CML) - (Philadelphia CreaM cheese)

Philadelphia Chromosome occurs in CML. ALL occurs in children. AML is associated with Auer rods. Multiple myeloma has Bence-Jones protein. CLL has no clear distinguishing feature except increased lymphocytes.

How well did you know this?
1
Not at all
2
3
4
5
Perfectly
63
Q

Which of the following thyroid profiles is most compatible with a diagnosis of primary hypothyroidism?

A. a low TSH (thyroid stimulating hormone) level and a high T4
B. a low TSH level and a normal T4
C. a low TSH level and a low T4
D. a high TSH level and a low T4
E. a high TSH and a high T4
A

Answer: D
High TSH level and a low T4

Low T4 is diagnostic for low thyroid function. If the pituitary is normal (as in primary thyroid disease) the TSH should be high as the pituitary tries to stimulate the failing thyroid gland.

How well did you know this?
1
Not at all
2
3
4
5
Perfectly
64
Q

All of the following are factors that predispose a patient to the development of gastroesophageal reflux EXCEPT:

A. hiatal hernia
B. pregnancy
C. scleroderma
D. an incompetent esophageal sphincter
E. pernicious anemia
A

Answer: E
Pernicious anemia

Pernicious anemia has no correlation with GERD. It is an autoimmune destruction of the gastric parietal cells that make intrinsic factor. Signs and symptoms are not present until B12 levels are very low (and include peripheral neuropathies & ataxia)

How well did you know this?
1
Not at all
2
3
4
5
Perfectly
65
Q

A patient warrants antihypertensive medication use for the duration of her pregnancy. Which of the following is the antihypertensive recommended for such patients?

A. alpha-methyldopa
B. captopril
C. nifedipine
D. propranolol
E. clonidine
A

Answer: A
Alpha-methyldopa

Methyldopa (Aldomet) is indicated in pregnancy. ACEI’s (captopril) are contraindicated in pregnancy. ß Blockers can be used but may cause growth restriction. Diuretics are not used in pregnancy. Calcium channel blockers and centrally acting agents (while not contraindicated) should be avoided.

How well did you know this?
1
Not at all
2
3
4
5
Perfectly
66
Q

A 6-year-old child falls onto his right arm. An x-ray demonstrates a buckle in the cortices of the distal radius, proximal to the growth plate, without angulation. What is the term used to describe this fracture?

A. Salter-Harris Type III
B. Salter Harris Type IV
C. Salter Harris Type V
D. torus
E. greenstick
A

Answer: D
Torus fracture

Torus or buckle fracture is most common in a child. This is proximal to the epiphyseal plate and so is not a Salter-Harris issue. Greenstick fracture is also common in children in long bones and is a fracture which “bends” the bone without fracturing it.

How well did you know this?
1
Not at all
2
3
4
5
Perfectly
67
Q

A 55-year-old male presents complaining of “difficulty wrtiing” using his dominant hand and some “slurred” speech. He has a h/o hypertension, DM type II, and hypertriglyceridemia. Which of the following would you anticipate to find on a CT scan of his head, as the explanation for his chief complaint?

A. hemorrhage in the distribution of his posterior cerebral artery
B. hemorrhage in the distribution of the middle cerebral artery
C. hypodensity measuring 12 mm by 21 mm in the distribution of the posterior cerebral artery
D. hypodensity measuring 4 mm by 4 mm in the internal capsule
E. calcifications bilaterally, in the third ventricles

A

Answer D
Hypodensity measuring 4 mm by 4 mm in the internal capsule

The main clue to this question is that most strokes are ischemic - especially with this patient’s history (ruling out hemorrhagic infarct - A & B). Calcifications (E) are not indicative of stroke at all. When trying to decide between C & D - the size of the area needs to correlate with the patient’s symptoms. If the patient had a HUGE infarct (12 x 21) he would have significantly more signs and symptoms than just some slurred speech and difficulty writing. (In addition, as it turns out, the internal capsule is supplied by the MCA - which is the most common vessel involved in an ischemic stroke)

How well did you know this?
1
Not at all
2
3
4
5
Perfectly
68
Q

A female in her third trimester of pregnancy developed hypertension, diffuse edema, proteinuria and hyperreflexia. She was treated with intravenous magnesium sulfate and is now hyporeflexic and drowsy. What do you prescribe now?

A. calcium
B. diazepam
C. an amphetamine
D. additional magnesium
E. oxygen
A

Answer: A
Calcium (Calcium Gluconate more specifically)

Pre-eclampsia causes hyperreflexia. Magnesium is the treatment. The sign of magnesium toxicity is a loss of reflexes… this is treated with calcium

How well did you know this?
1
Not at all
2
3
4
5
Perfectly
69
Q

This papulosquamous eruption is most common in young adults. A single oval patch is generally noted several days before a more generalized, fawn-colored rash erupts. This rash is most prevalent on the trunk, and the proximal upper and lower extremities. The rash spontaneously disappears over five to six weeks. The diagnosis is:

A. pityriasis rosea
B. tinea corporis
C. psoriasis
D. atopic dermatitis
E. sporotrichosis
A

Answer: A
Pityriasis rosea

This is the classic presentation of pityriasis rosea. Remember that the differential diagnosis includes secondary syphilis, so if there is a positive sexual history - consider ordering a VDRL/RPR. The “herald” patch which is described is pathognomonic for pityriasis.

How well did you know this?
1
Not at all
2
3
4
5
Perfectly
70
Q

A patient presents complaining of generalized swelling. Her urine is positive for protein. Her serum testing reveals hyperlipidemia and hypoalbuminemia. Which of the following is the most likely diagnosis?

A. cirrhosis
B. nephrotic syndrome
C. congestive heart failure
D. cystitis
E. pyelonephritis
A

Answer: B
Nephrotic syndrome

This is a classic presentation of nephrotic syndrome.

  • hyperlipidemia
  • proteinuria
  • hypoalbuminemia
  • generalized edema
How well did you know this?
1
Not at all
2
3
4
5
Perfectly
71
Q

A patient is experiencing an acute exacerbation of asthma. Which of the following drugs would be the least useful in the management of this asthma attack?

A. albuterol
B. cromolyn sodium
C. prednisone
D. theophyline
E. epinephrine
A

Answer: B
Cromolyn sodium

Cromolyn is a mast cell stabilizer and must be present in the system PRIOR to the symptoms. It would be useless in a patient already having symptoms.

How well did you know this?
1
Not at all
2
3
4
5
Perfectly
72
Q

A 34-year-old female patient presents to the ER with sharp pleuritic-type chest pain. An x-ray shows an enlarged cardiac silhouette suggestive of pericarditis. On more careful questioning, the patient admits to a polyarticular arthritis involving the small joints of her hands. Her obstetrical history is positive for two third trimester spontaneous abortions. At this point, you would suspect which autoimmune process as the cause of her symptoms:

A. Rheumatoid arthritis
B. Systemic Lupus Erythematosis
C. Wegener's granulomatosis
D. Sjogren's Syndrome
E. Scleroderma
A

Answer: B
Systemic Lupus Erythematosus

40-60% of patients with SLE have serositis (pleuritis and pericarditis). That, combined with joint pains, habitual AB - should cause you to order an ANA.

How well did you know this?
1
Not at all
2
3
4
5
Perfectly
73
Q

A patient with AIDS develops severe headaches. A CT scan demonstrates multiple ring-enhancing lesions of the brain. What diagnosis is most likely?

A. Toxoplasmosis
B. Histoplasmosis
C. lymphoma
D. Cytomegalovirus
E. Herpes encephalitis
A

Answer: A
Toxoplasmosis

Toxoplasmosis occurs commonly in HIV and is a parasitic infection causing ring-enhancing lesions (but lots of things can cause ring-enhancing lesions). CMV is common as well and typically causes retinitis. Herpes encephalitis can happen frequently too but causes a diffuse encephalitis.

How well did you know this?
1
Not at all
2
3
4
5
Perfectly
74
Q

A Caucasian male has a history of a meconium ileus as a newborn, steatorrhea and a positive sweat test. What is the most likely diagnosis?

A. Von Gierke’s disease
B. Hurler’s syndrome
C. Cystic fibrosis
D. Hunter’s syndrome
E. Cri du chat syndrome
A

Answer: C
Cystic fibrosis

This is the classic presentation of Cystic fibrosis

How well did you know this?
1
Not at all
2
3
4
5
Perfectly
75
Q

A 32-year-old male presents with dyspnea and a nonproductive cough. He is tachycardic, tachypneic, and febrile. Auscultation of his chest reveals scattered rhonchi. His chest x-ray demonstrates a diffuse interstitial infiltrate. His ABG demonstrates moderate hypoxemia and his LDH is elevated. What is the most likely diagnosis?

A. Streptococcal pneumoniae pneumonia
B. Mycoplasma pneumoniae pneumonia
C. Pneumocystis carinii pneumonia
D. Bowen's disease
E. Steven-Johnson syndrome
A

Answer: C
Pneumocystis carinii pneumonia

Formerly PCP Pneumonia now called (PJP) Pneumocystis jiroveci. Many AIDS patients present initially with PJP. These are the classic signs and symptoms of PJP pneumonia. While mycoplasma patients can be acutely ill, they are more likely (especially on the Boards) to present as mildly ill with a normal physical exam and no hypoxia. S. pneumonia is the most common community-acquired pneumonia but would present with a focal/lobar consolidation.
Common in HIV-infected patients with a low CD4 count of less than 200
Prophylaxis for high-risk patients with a CD4 count of less than 200 or with a history of PJP infection

How well did you know this?
1
Not at all
2
3
4
5
Perfectly
76
Q

A patient describes a history of a “curtain being brought down over my right eye it stayed there for a few minutes then was lifted back up.” In which of the following arteries is the etiology of this complaint located?

A. anterior cerebral
B. middle cerebral
C. posterior cerebral
D. internal carotid
E. external carotid
A

Answer: D
Internal Carotid

This is amaurosis fugax….a form of “TIA” of the optic artery — it arises from the internal carotid - most common location for occlusion in TIA.

How well did you know this?
1
Not at all
2
3
4
5
Perfectly
77
Q

Upon review of an EKG you notice periodic, repetitively absent QRS complexes and “grouped beating.” Which of the following is the explanation?

A. Third degree heart block
B. Mobitz Type II heart block
C. Mobitz Type I heart block
D. first degree heart block
E. sinus tachycardia
A

Answer: C
Mobitz Type I heart block

I don’t like the wording of this question, but typically “group beating” suggests Wenckebach. The fact that there are absent QRS complexes means there is a second or third-degree AVB. In Mobitz II, the PR intervals are the same - so no “grouped” beating, in Third degree, there is no rhyme or reason to the rhythm. In Wenckebach, the rhythm “repeats”, causing a “regularly irregular” rhythm.

How well did you know this?
1
Not at all
2
3
4
5
Perfectly
78
Q

A 14-month-old female is brought to your office with a 3-day history of a fever of 104 degrees rectally. Suddenly, today, the fever stopped and a rash developed. Examination reveals a diffuse, fine, maculopapular rash. Presently, the child does not appear ill. The most likely diagnosis is:

A. Rubella
B. Rubeola
C. erythema infectiosum (Fifth's disease)
D. Roseola
E. Chicken pox
A

Answer: D
Roseola

This is typical for Roseola. Rubella has a mild illness with a fine macular rash. Rubeola (measles) is an acute illness, the child is toxic, has conjunctivitis and Koplik spots. Fifth’s disease has mild illness with a “lacy” rash on the extremities. Chickenpox has vesicles.

How well did you know this?
1
Not at all
2
3
4
5
Perfectly
79
Q

At what age does the first tooth usually erupt in an infant?

A. 2-4 months
B. 6-8 months
C. 10-12 months
D. 14-16 months

A

Answer: B
6-8 months

The first tooth in an infant to erupt is the central incisor at the average age of 6-8 months.

How well did you know this?
1
Not at all
2
3
4
5
Perfectly
80
Q

A 27-year-old man comes to the clinic because of daily unilateral headaches for the past week. Episodes last 1 hour, are abrupt in onset and cessation, localized to the orbital and temporal regions, and are accompanied by lacrimation, rhinorrhea, and facial sweating. His wife says these headaches occur at night and that during the attacks her husband is restless. Which of the following is the most likely diagnosis?

A Cluster headache
B Horner syndrome
C Migraine
D Paroxysmal hemicrania
E Trigeminal neuralgia
A

A - Cluster Headache

How well did you know this?
1
Not at all
2
3
4
5
Perfectly
81
Q

A 25-year-old man comes to the emergency department because of severe pain behind his right eye. He says he has been awakened nightly by the pain over the past week. He denies nausea, vomiting, or photosensitivity. Physical examination reveals increased tearing in the right eye, but an otherwise normal neurologic examination. Which of the following is the most likely diagnosis?

A Trigeminal neuralgia    
B Conversion disorder    
C Cluster headaches    
D Migraine headaches    
E Temporal arteritis
A

C - Cluster Headache

How well did you know this?
1
Not at all
2
3
4
5
Perfectly
82
Q

A 22-year-old woman comes to the clinic because of an acute headache that feels “like a screwdiver through her temple” lasting 1 hour. She says this has happened several times in the past year, but she cannot identify a source. She rates the pain as 11/10. She additionally complains of rhinorrhea, and denies any gastrointestinal symptoms, phonophobia, dizziness, blurred vision, or photophobia. Physical examination shows ipsilateral lacrimation, ipsilateral miosis, no temporal tenderness, and no ophthalmologic pathology. Remainder of physical exam is unremarkable. Which of the following is most consistent with these symptoms?

A Tension Headache
B Cluster Headache
C Atypical Migraine Headache
D Subarachnoid hemorrhage 
E Typical Migraine Headache
A

B - Cluster Headache

How well did you know this?
1
Not at all
2
3
4
5
Perfectly
83
Q

A 27-year-old man comes to the emergency department because of severe retro-orbital pain of one-day duration. He says the pain is really severe and came on somewhat suddenly. The pain is primarily concentrated behind his left eye. He also says that his eyes, especially the left, are tearing up, and his nose is dripping. He says he has had these “attacks” before. In the emergency department, he denies a history of trauma, neck stiffness, recent illness, or vision loss. No abnormalities are found on ophthalmic examination. A computed tomography scan of the head and lumbar puncture are normal. He is afebrile. Which of the following is the most likely diagnosis?

A Cluster Headache
B Glaucoma
C Meningitis
D Migraine Headache
E Subarachnoid Hemorrhage
A

A - Cluster Headache

How well did you know this?
1
Not at all
2
3
4
5
Perfectly
84
Q

__________ is a Gram-negative bacillus that may precede Guillain-Barre syndrome or reactive arthritis.

A

Campylobacter jejuni

How well did you know this?
1
Not at all
2
3
4
5
Perfectly
85
Q

A 22-year-old man comes to the emergency department because of eye pain. The patient noticed his eyes become red and painful eyes a few days ago, and symptoms seemed to be getting worse. He also complains of right knee pain. On review of systems, he is positive for increased urinary frequency. Physical examination shows bilateral conjunctivitis and bogginess of his right knee. Which of the following organisms is most likely responsible for his condition?

A Campylobacter jejuni 
B Chlamydia trachomatis
C Escherichia coli
D Neisseria gonorrhoeae
E Adenovirus
A

B Chlamydia trachomatis

22 y/o MALE = reiter’s syndrome after MC chlamydia

How well did you know this?
1
Not at all
2
3
4
5
Perfectly
86
Q

A 32-year-old man comes to the clinic because of bilateral erythema, pain of the conjunctiva, and lesions on his feet. Patient says he has the sensation of “burning” and pain upon urination, and swelling of the knees. The patient rates the pain as a 7 on a 10-point scale. He is sexually active with multiple partners and does not use a barrier method of protection. Culture from a biopsy made from the patient’s foot lesions

A Campylobacter jejuni
B Chlamydia trachomatis
C Clostridium botulinum 
D Haemophilus ducreyi
E Neisseria gonorrhea
A

patient has - KERATODERMA BLENNORRHAGICUM*** = hyperkeratotic lesions on palms / soles

Reiter’s Syndrome - reactive arthritis

MC = B - Chlamydia trachomatis

How well did you know this?
1
Not at all
2
3
4
5
Perfectly
87
Q

A 22-year-old man comes to the clinic because of photophobia, dysuria, and arthralgias for the past week. Patient says he had a sexually transmitted disease diagnosed a month ago, which was treated with antibiotics. Physical examination shows an erythematous eye with a watery discharge and a swollen, tender, and warm left knee on palpation. Which of the following human leukocyte antigens (HLA) types is most likely related to this condition?

A HLA B27 gene
B HLA B8 gene
C HLA DR2 gene
D HLA DR3 gene
E HLA DR5 gene
A

A - HLA B27 gene

How well did you know this?
1
Not at all
2
3
4
5
Perfectly
88
Q

A 33-year-old man comes to the office because of sore knees for the past 3 days. He says that he is also experiencing painful urination and painful, red, itchy eyes. He was treated for an infection 3 weeks ago. He has an uncle with ankylosing spondylitis. Examination shows bilateral swollen knees, with effusions. There is also bilateral injection of the conjunctivae. Which of the following microbes is the most likely cause of this patient’s condition?

A Chlamydia trachomatis 
B Clostridium botulinum 
C Neisseria gonorrhoeae
D Rickettsia prowazekii
E Treponema pallidum
A

HLA - B27 = ankylosing spondylitis, psoriasis, and reiter’s syndrome

MC = A - Chlamydia trachomatis

How well did you know this?
1
Not at all
2
3
4
5
Perfectly
89
Q

A 20-year-old man presents to campus health because he has had a painful, swollen right knee. His knee swelling began three weeks ago. He also complains of itchy, red eyes and a left-sided lower back ache which began two weeks ago. He denies any trauma. His medical history includes treatment for a chlamydia infection six weeks ago. Family history is significant for paternal-sided ulcerative colitis. Examination shows bilateral conjunctival injection. The patient also walks with a limp, and has a large effusion of his right knee. Which of the following is the most likely diagnosis?

A Ankylosing spondylitis
B Psoriatic arthritis
C Reactive arthritis
D Rheumatoid arthritis
E Systemic lupus erythematosus
A

C Reactive arthritis

HLA B27 DISEASE ASSOCIATIONS
Ankylosing Spondylitis            >90%    = most common
Reactive Arthritis                  85%
Reiter’s Syndrome              80%
Inflammatory Bowel Disease 50%
Psoriatic Arthritis                  50%
Whipple’s Disease              30%

Ulcerative Colitis = IBD = HLA-B27 disease

How well did you know this?
1
Not at all
2
3
4
5
Perfectly
90
Q

A 19 y/o male presents with acute onset of right knee pain, inflammed conjunctiva, mild burning with urination and a history of fever and diarrhea one week prior. Which of the following is the most likely diagnosis?

a. Polyarteritis Nodosa
b. Septic knee joint
c. Reactive Arthritis (Reiter’s Syndrome)
d. Systemic Lupus Erythematosus
e. Ankylosing Spondylitis

A

c. Reactive Arthritis (Reiter’s Syndrome)

How well did you know this?
1
Not at all
2
3
4
5
Perfectly
91
Q

Which one of the following disorders is associated with increased incidence of positivity for HLA-B27?

a. Pseudogout
b. Sjogrens
c. Systemic lupus erythematosus
d. Ankylosing spondylitis

A

d. Ankylosing spondylitis

How well did you know this?
1
Not at all
2
3
4
5
Perfectly
92
Q

Ossification of the annulus and fibrosis of the intervertebral disc and longitudinal ligaments (bamboo spine) appearing on x-ray are classically associated with which of the following?

a. Osteoarthritis
b. Rheumatoid arthritis
c. Degenerative disc disease
d. Reiter’s Syndrome
e. Ankylosing Spondylitis

A

e. Ankylosing Spondylitis `

How well did you know this?
1
Not at all
2
3
4
5
Perfectly
93
Q

A 27-year-old man comes to the office because of a lower back pain that radiates down his buttocks and thighs for 2 years. His medical history includes inflammatory bowel disease and type 1 diabetes mellitus. The pain is worse at night and after long periods of sitting, and gets better with movement and exercise. He also mentions that his knees have been hurting recently. Examination shows limited spinal mobility, sacroiliac joint tenderness, and reduced chest expansion. Laboratory investigations show that HLA-B27 is positive and there are raised inflammatory markers. Which of the following side effect are least likely to occur in the treatment of this condition?

A Gastritis
B Leukopenia
C Myositis
D Renal failure
E Tuberculosis
A

C - Myositis

Ankylosing spondylitis treatments include NSAIDs, glucocorticoids, and disease-modifying antirheumatic drugs; if this treatment fails, anti-TNF antibodies are the next treatment step. Side-effects of treatment include renal failure, leukopenia, and potentially lethal infections, including tuberculosis. In contrast, myositis symptoms are improved with NSAID and glucocorticoid therapy.

How well did you know this?
1
Not at all
2
3
4
5
Perfectly
94
Q

A 35-year-old woman comes to the office because of back pain for 12 years. She describes pain from her neck to tailbone. It is worse at night, sometimes even waking her from sleep. The pain subsides with movement and exercise. She also mentions having intermittent pain at the elbows, wrist, ankles and most recently, the knees. Physical exam shows restricted movements of the neck, the thoracic spine and her lumbar spine. Her ability to expand her chest was reduced, and she was tender at the sacroiliac joints. Which of the following is least likely to be found in her diagnostic work-up?

A Anti-CCP positive
B Elevated C-reactive protein 
C Elevated erythrocyte sedimentation rate
D HLA-B27 positive 
E Sacroiliitis
A

A - Anti-CCP positive

the rest can all be found on diagnostic work-up for ankylosing spondylitis

How well did you know this?
1
Not at all
2
3
4
5
Perfectly
95
Q

A 19-year-old man comes to the emergency room because of difficulty breathing for 6 hours. He reports that earlier that day, he was doing laundry and suddenly became short of breath. He also reports back pain for a year that is worst after waking up in the morning. He reports that his morning stiffness typically lasts for approximately an hour. Over the past few months, he has noticed pain in his hips intermittently but did not have insurance to see a doctor so he ignored it. Chest and spine X-rays are obtained. Which of the following is associated with the most likely diagnosis?

A HLA-B27 positivity
B HLA-DR3 positivity
C Positive ANA
D Positive cyclic citrullinated peptide
E Positive rheumatoid factor
A

A HLA-B27 positivity

How well did you know this?
1
Not at all
2
3
4
5
Perfectly
96
Q

A 17-year-old girl, gravida 2, para 1, comes to the obstetrics and gynecology clinic for routine prenatal care. She has a history of Hashimoto thyroiditis. She is currently 4 weeks’ pregnant. Which of the following complications is the newborn at risk for if the mother’s thyroid hormone concentrations are not properly monitored?

A Anencephaly
B Caudal Regression Syndrome
C Intellectual Disability
D Macrosomia
E Spina Bifida
A

C Intellectual Disability

Congenital hypothyroidism is one of the leading causes of intellectual disability in the world. All newborns in the United States are screened for hypothyroidism.

Congenital hypothyroidism, also known as cretinism, is one of the leading causes of intellectual disability in the world. During pregnancy, the mother has increased iodine requirements to ensure a euthyroid state because of the developing fetus

How well did you know this?
1
Not at all
2
3
4
5
Perfectly
97
Q

A 16-year-old girl comes to the clinic because of worsening fatigue for the past month. She says she has always been the smallest in her class but began puberty at age 14. Her height is now 160 cm (5 ft 3 in). While she keeps up with her class in terms of grades, she feels constantly tired despite having sufficient time for fun and rest. Physical examination shows normal skin color and texture, no periorbital edema, and no visible thyroid on extension of the neck. Her serum thyroxine, thyroid-stimulating hormone, and thyroglobulin are 3 μg/dL, 25 μU/mL, and 3ng/mL respectively. Ultrasound shows no thyroid tissue at the thyroid bed. Computed tomography shows a hyperdense soft tissue mass located at the retroesophageal superior mediastinum. Which of the following is the most likely diagnosis?

A Dyshormonogenic Goiter
B Ectopic Thyroid
C Iodine deficiency
D Lingual thyroid
E Thyroid Agenesis
A
TSH = 25 (very high; normal = 0.4 - 4)
T4 = 3 (low; normal = 4.5 - 11)

Hypothyroidism

B - Ectopic Thyroid

Ectopic thyroid is displacement of thyroid tissue from its normal location at the lower anterior neck. The most common cause of congenital hypothyroidism is thyroid dysgenesis; the most common form of thyroid dysgenesis is ectopic thyroid; the most common form of ectopic thyroid is lingual thyroid.

How well did you know this?
1
Not at all
2
3
4
5
Perfectly
98
Q

A 30-year-old woman comes to the office because of fatigue. She reports dry skin, increased hair loss, and constipation. She says that she has gained more than 4.5-kg (10-lb) over the past few months, and she has missed her period for the last two months. Her respirations are 12/min, her blood pressure is 115/92 mmHg, and her BMI: 28 kg/m2. Obtaining which of the following laboratory values would be the next best step in diagnosing this patient?

A Serum TPOAb (Anti-thyroid peroxidase antibody)
B TSH and free T4
C Human chorionic gonadotropin (hCG)
D T3
E Free T4
A

B TSH and free T4

When working up a patient for hypothyroidism, the first step is to obtain TSH and free T4 laboratory values. The serum TSH would be high, and the free T4 will help determine the degree of hypothyroidism.

How well did you know this?
1
Not at all
2
3
4
5
Perfectly
99
Q

A 36-year-old man comes to the office complaining of feeling cold for the past 7 months. He says that he has been using less air conditioning this summer and is always dressed in layers at work. He reports a 3.6-kg (8-lb) weight gain over the past year and says he notices some blood on the toilet paper occasionally from straining to pass his stools on the toilet. Which of the following is the most appropriate initial test to order?

A Fine needle aspiration biopsy
B Radioiodine scan
C Thyroid stimulating hormone (TSH) level
D Total thyroxine (T4) level
E Total triiodothyronine (T3) level
A

C Thyroid stimulating hormone (TSH) level

The best screening test for suspected hypothyroidism is a TSH level. TSH would be elevated in the case of a primary hypothyroid disease such as Hashimoto thyroiditis.

The best screening test for suspected hypothyroidism is a TSH level. Levels of TSH indicate whether there is primary thyroid disease; TSH would be elevated in the case of a primary hypothyroid disease such Hashimoto thyroiditis, as in this patient. Alternatively, TSH levels would be low in other types of primary thyroid disease, such as Graves disease, in which there is an overproduction of thyroid hormone and thus symptoms of hyperthyroidism.

How well did you know this?
1
Not at all
2
3
4
5
Perfectly
100
Q

A 42-year-old woman presents to her primary care physician with a chief complaint of fatigue. She says that she has been sleeping more than usual. She has gained 20 pounds over the past year and has no energy to exercise. On physical exam, she appears tired and her skin is dry. Her sclerae are white. When you put her hand next to yours, you notice a difference in the hue of her skin (on the left) when compared to your own (on the right). Which of the following is most likely to be increased in this woman’s serum?

A Conjugated bilirubin
B Beta-Carotene
C Retinol
D Zinc

A

B Beta-Carotene

NOT A = This patient’s skin is yellow, which supports the possibility of jaundice, but her sclerae are white. Scleral icterus would suggest hyperbillirubinemia.

Carotonemia is yellowing of the skin associated with beta-carotene excess and can be seen in hypothyroidism.

Beta-carotene builds up in the blood due to lack of thyroxine (T4), which normally speeds the conversion of beta-carotene to retinol

How well did you know this?
1
Not at all
2
3
4
5
Perfectly
101
Q

A 71-year-old woman comes to the emergency department because of altered mental status. She was brought by a patrolman who found her wandering next to the highway. She was unable to answer questions and collapsed in transit. Her temperature is 33°C (91.4°F), pulse is 40/min, respirations are 12/min, blood pressure is 80/50, and oxygen saturation is 85% on room air. Physical examination shows decorticate posturing, incomprehensible speech, eyes opening to pain, dry hair, coarse and waxy skin, and non-pitting edema around the face and all extremities. Electrocardiogram shows sinus bradycardia. Laboratory studies show:

Calcium: 9.0 mg/dL
Hematocrit (female): 34%
Potassium: 4.0 mEq/L
Sodium: 120 mEq/L
TSH: 6.0 μU/mL
Thyroxine (T4): 3.0 μg/dL
Triiodothyronine (T3): 100 ng/dL

Which of the following is the most likely diagnosis?

A Myxedema Coma
B Pheochromocytoma Crisis
C Septic Shock
D Tertiary Hyperparathyroidism
E Thyrotoxic storm
A

normal TSH = 0.4 - 4 (6 = high!)
normal T4 = 4.5 - 11.2 (3 = low!)
normal T3 = 100 - 200
= Hypothyroidism

A Myxedema Coma

Myxedema refers to two distinct concepts:

Severe hypothyroidism that can lead to decreased temperature, blood pressure, breathing, and responsiveness

A form of edema due to deposition of connective tissue components

How well did you know this?
1
Not at all
2
3
4
5
Perfectly
102
Q

A 67-year-old man comes to clinic because of “forgetfulness” for the past 3 months. His daughter brought him, saying he misplaces his keys, loses his bank card, and forgets to turn off stoves. The patient also complains of constipation and weakness for the past 3 days. His temperature is 36.4°C (97.6°F), pulse is 55/min, respirations are 18/min, and blood pressure is 138/90 mm Hg. Physical examination shows dry, cold, and puffy skin with no other abnormalities. Which of the following is the most likely explanation for this patient’s condition?

A Alzheimer disease
B Creutzfeldt-Jakob disease
C Delirium
D Hypothyroidism
E Lewy Body Dementia
A

D Hypothyroidism

Hypothyroidism presents many symptoms akin to dementia. If caught early it can be treated and reversed. Typical signs and symptoms include low heart rate and temperature, cool extremities, cold intolerance, weight gain, fatigue, constipation, and dementia.

How well did you know this?
1
Not at all
2
3
4
5
Perfectly
103
Q

A 25-year-old woman presents to the emergency department with recent mood changes and increased feelings of fatigue. She also reports cold sensitivity. Her symptoms have gradually worsened over the past few months. Her vital signs are normal. Laboratory studies show a normal complete blood count and basic metabolic panel with an elevated TSH. Which of the following lab tests is most appropriate in guiding the management of this patient?

A Serum TPOAb (Anti-thyroid peroxidase antibody)
B Serum free T3 (triiodothyrine)
C Serum free T4 (thyroxine)
D Serum total T4
E Serum total T3
A

C Serum free T4 (thyroxine)

TSH and free T4 levels are the two most useful laboratory studies to obtain in the evaluation of a patient with suspected thyroid disease.

How well did you know this?
1
Not at all
2
3
4
5
Perfectly
104
Q

A 3-month-old infant of immigrant parents is brought to the pediatrics clinic because of intermittent choking, constipation, lethargy, hoarse cry, and a suspicion that she is “floppier” than other babies since her birth. The patient is in the 15th percentile for height and 25th percentile for weight. Physical examination reveals an infant with a protuberant abdomen, dry skin, dry, brittle hair, and a low hairline. Parents report that her birth was normal. Which of the following is the most appropriate next step in management?

A Administration of the flu vaccine
B Karyotype test
C Heptavalent Botulinum Antitoxin
D Return for her 4-month well child check
E Administration of thyroxine
A

E Administration of thyroxine

Congenital hypothyroidism often presents with hypotonia, lethargy, macroglossia, large fontanelles, and dry skin. Early treatment with thyroid hormone is indicated.

How well did you know this?
1
Not at all
2
3
4
5
Perfectly
105
Q

A 35-year-old woman, gravida 3, para 2, comes to the clinic for her initial prenatal visit. She has hypothyroidism secondary to Hashimoto’s thyroiditis that is treated with levothyroxine. She is currently asymptomatic, and her TSH is 2.5 mU/L. Which of the following is the most appropriate management of her hypothyroidism as her pregnancy progresses?

A Decrease her levothyroxine dose
B Discontinue levothyroxine and begin liotrix
C Discontinue levothyroxine until after delivery
D Increase her levothyroxine dose
E Maintain levothyroxine dose

A

TSH normal values = 0.4 - 4.0
TSH values desired if already on Synthroid = 0.4 - 3.0

D Increase her levothyroxine dose

Although the patient is currently asymptomatic and her TSH is within normal limits (0.4-4.0 mU/L), thyroid hormone requirements increase during pregnancy to meet increased metabolic needs.

How well did you know this?
1
Not at all
2
3
4
5
Perfectly
106
Q

Which of the following ADHD medications has a warning that there may be an increased risk for suicide when taking this drug?

a. Guanfacine (Intuniv)
b. Lisdexamfetamine (Vyvanse)
c. Atomoxetine (Strattera)
d. Methylphenidate (Methylin)

A

c. Atomoxetine (Strattera)

How well did you know this?
1
Not at all
2
3
4
5
Perfectly
107
Q

A 25-year-old woman with a history of substance abuse is diagnosed with ADHD. Which of the following is the treatment of choice?

a. Lisdexamfetamine (Vyvanase)
b. Dextroampetamine (Dexedrine)
c. Methylphenidate (Ritalin)
d. Atomoxetine (Strattera)

A

d. Atomoxetine (Strattera)

if there is a hx of substance abuse in household members, can prescribe a non-stimulant therapy

How well did you know this?
1
Not at all
2
3
4
5
Perfectly
108
Q

What are the main neurotransmitters involved in the pathogenesis of ADHD?

a. Serotonin and GABA
b. Dopamine and serotonin
c. Dopamine and norepinephrine
d. Gluatamate and norepinephrine

A

c. Dopamine and norepinephrine

How well did you know this?
1
Not at all
2
3
4
5
Perfectly
109
Q

A 9-year-old boy is brought in for evaluation for his disruptive behavior at school. He is reported to have difficulty remaining seated, blurting out answers to questions, difficulty awaiting turns, talking excessively, intrudes on others by butting into games or activities and does not pay attention to details in his school work. What is the most likely diagnosis?

a. Generalized anxiety disorder with impulsivity
b. ADHD predominantly hyperactive/impulsive
c. ADHD predominantly inattentive
d. Oppositional defiant disorder

A

b. ADHD predominantly hyperactive/impulsive

How well did you know this?
1
Not at all
2
3
4
5
Perfectly
110
Q

Which one of the following is a contraindication to initiation of stimulant medication for the treatment of ADHD?

a. Migraine headaches
b. A family history of myocardial infarction in a maternal grandparent
c. VSD repair as an infant
d. BMI of 18
e. Irritable bowel syndrome

A

c. VSD repair as an infant

How well did you know this?
1
Not at all
2
3
4
5
Perfectly
111
Q

An 8-year-old boy who has just been diagnosed with ADHD comes in to the clinic. The family would like to administer the medication before school, avoid dosing at school (mid-day), he can swallow pills and they would like the medication to last through the school day which is 7 hours. Which of the following is the treatment of choice?

a. Guanfacine (Tenex)
b. Methylphenidate SR (Ritalin SR)
c. Methylphenidate transdermal (Daytrana)
d. Atomoxetine (Strattera)

A

b. Methylphenidate SR (Ritalin SR)

How well did you know this?
1
Not at all
2
3
4
5
Perfectly
112
Q

A 10-year-old boy is diagnosed with ADHD. His parents wish to avoid stimulant medications. Which one of the following medications would be most appropriate first line therapy for this patient?

a. Atomoxetine (Strattera)
b. Guanfancine (Tenex)
c. Clonidine (Catapres)
d. Imipramine (Tofranil)

A

a. Atomoxetine (Strattera)

How well did you know this?
1
Not at all
2
3
4
5
Perfectly
113
Q

A 9-year-old girl is brought in for follow up for ADHD. She has been on amphetamine-dextroamphetamine (Adderall) for 3 months and reports improvement in her ability to concentrate at school. She has also experienced a decreased appetite and weight loss of 6 pounds since starting this medication. Her present BMI is 19. Which of the following is the most appropriate course of action at this time?

a. Discontinue the medication
b. Supplement with a daily multivitamin
c. Have her take the medication an hour earlier in the day
d. Encourage drug holidays on the weekends and school breaks

A

d. Encourage drug holidays on the weekends and school breaks

How well did you know this?
1
Not at all
2
3
4
5
Perfectly
114
Q

Which of the following agents is effective in the treatment of Parkinson’s?

A. penicillin
B. amantadine
C. erythromycin
D. acyclovir
E. cephalexin
A

Answer: B - Amantadine

Amantadine (Symmetrel) is effective (somewhat). It is also used in treating Influenza A specifically. Another choice is selegiline, and levodopa/carbidopa. The other antibiotics listed here are not useful in the treatment of Parkinson’s disease.

How well did you know this?
1
Not at all
2
3
4
5
Perfectly
115
Q

The most common cause of preventable blindness in the United States is:

A. Macular degeneration
B. Cataracts
C. Retinal detachments
D. Glaucoma
E. Trauma
A

D. Glaucoma

The wording is intentionally difficult in this question. Macular degeneration, retinal detachments, and trauma are not preventable. Cataracts are “curable”, but the cataract itself is probably not “preventable.” Chronic glaucoma can be treated to prevent blindness this is a matter of semantics.

How well did you know this?
1
Not at all
2
3
4
5
Perfectly
116
Q

Laboratory report: elevated serum free t4; low serum TSH level. Which of the following is the MOST likely diagnosis?

A. Primary hypothyroidism
B. Secondary hypothyroidism
C. T3 thyrotoxicosis
D. Graves disease
E. Chronic Hashimoto's thyroiditis
A

Answer: D - Graves disease

High T4 (hyperthyroid), with a low TSH, indicates that the pituitary is working normally, and the thyroid is hyperactive i.e. Grave’s disease. Primary hypothyroidism (Hashimoto’s) is low T4 with a high TSH. Secondary hypothyroidism (from pituitary disease) is a low T4 with a low TSH.

How well did you know this?
1
Not at all
2
3
4
5
Perfectly
117
Q

The results of a barium esophagram reveal esophageal webbing in a woman with long-standing iron deficiency anemia. Her diagnosis is:

A. Barrett's esophagus
B. achalasia
C. Plummer-Vinson syndrome
D. Dresslers syndrome
E. Ogilvies syndrome
A

Answer: C - Plummer-Vinson syndrome

Plummer-Vinson is something to know for those who want a 100% on the exam. There is no need to memorize this. More importantly, webbing in the distal esophagus with no other symptoms but dysphagia would be a Schatzki’s ring (much more common). Barrett’s causes GERD symptoms and dysplasia of the esophageal mucosa. Achalasia is a primary neurologic problem of the esophagus causing dysphagia.

How well did you know this?
1
Not at all
2
3
4
5
Perfectly
118
Q

A 41 y/o female presents to PCP complaining of difficulty swallowing her food x last 7 months. Has lost 15 pounds unintentionally and has been tired. Drinks 1 glass of wine/week and 5 pack year history of smoking but quit 20 years ago. Denies taking any medication. Upper endoscopy is ordered. Labs are as follows:

WBC count = 6050 /mm3
RBC count = 2.5 million
Hemoglobin = 10.2 g/dL
MCV = 72
MCHC = 28% hb/cell
RDW = 16.8%
Platelet count = 490,000

Which sign/symptom is most likely in this patient?

a) atrophic glossitis
b) esophageal adenocarcinoma
c) nocturnal cough and dyspnea
d) painful esophageal mucosal lacerations
e) painless esophageal bleeding

A

1) difficulty swallowing food (dysphagia)
2) esophageal webbing on picture
3) MCV = low = microcytic anemia = iron. RDW = high (11.5 - 14.5) = iron deficiency anemia

= a) atrophic glossitis

Patient has plummer vinson’s syndrome! = dysphagia, esophageal webbing, glossitis, iron deficiency anemia

How well did you know this?
1
Not at all
2
3
4
5
Perfectly
119
Q

A 42-year-old woman who recently emigrated from Norway comes to the free clinic because of progressive trouble swallowing for the past few months. She says that when she tries to swallow bread, it often feels like it gets stuck before it reaches her stomach. She also complaints of fatigue and shortness of breath after walking up the stairs to her 2nd-floor apartment that requires her to rest for several minutes to recover. Her current medications include an oral contraceptive pill. Her temperature is 37.0°C (98.6°F), pulse is 88/min, respirations are 17/min, and blood pressure is 110/66 mm Hg. Physical examination shows a thin, pale woman with a smooth, red, and shiny tongue. Laboratory studies show a hemoglobin of 9.1g/dL and a leukocyte count of 9,000mm3. Barium esophagram shows multiple esophageal webs, as shown below. Which of the following is the most likely diagnosis?

A Achalasia
B Esophageal adenocarcinoma
C Gastroesophageal reflux disease
D Plummer-Vinson syndrome
E CREST syndrome
A

D Plummer-Vinson syndrome

Plummer-Vinson syndrome is a condition characterized by the triad of dysphagia, iron-deficiency anemia, and esophageal webs that is often associated with glossitis as well. Patients are at an increased risk of squamous cell carcinoma of the esophagus, and initial treatment is iron-supplementation and relief of dysphagia.

normal hemoglobin = 12-18 (9.1 = LOW)
normal leukocyte = 4,000 - 10,000 (9,000)

iron deficiency anemia
glossitis
dysphagia

How well did you know this?
1
Not at all
2
3
4
5
Perfectly
120
Q

Which of the following disorders is described as a milder pattern of mood swings that does not reach the severity of bipolar disorder but does include brief depressive and manic episodes?

a. Anhedonia
b. Anxiety disorder
c. Dysthymia
d. Cyclothymia

A

d. Cyclothymia

How well did you know this?
1
Not at all
2
3
4
5
Perfectly
121
Q

A 16-year-old girl comes to the pediatrician for a routine check-up. She denies feeling depressed but admits fighting with her mother more often. She reports that she has not menstruated in four months. Physical examination is normal except for excessive growth of lanugo body hair. Which of the following will be normal in this patient?

A Height
B Complete blood count
C ECG
D Lipid panel
E X-ray
A

A - Height

Not CBC - Complete blood count may show signs of anemia and leukopenia. - low platelets + RBCs

Not EKG - Bradycardia, hypokalemia changes; electrolyte imbalances may lead to cardiac abnormalities

not Lipid Panel - could lead to impaired liver function. This is could cause hypercholesterolemia and elevated triglycerides.

not XRAY - osteoporosis

How well did you know this?
1
Not at all
2
3
4
5
Perfectly
122
Q

A 17-year-old female comes to the emergency room because of a suspected colles fracture after tripping and falling in the parking lot at school. Her pulse is 64/min and blood pressure is 104/72 mm Hg. Her height is 162 cm (5 ft 4 in) and weighs 45.4-kg (100-lbs). She confirms her last menstrual period was more than 3 months. Urine pregnancy test shows no sign of pregnancy. Which of the following is her diagnosis?

A Premature ovarian insufficiency
B Ewing Sarcoma
C Bulimia nervosa
D Osteogenesis imperfecta
E Anorexia nervosa
A

E Anorexia nervosa

bradycardia
hypotension
fracture - osteoporosis
amenorrhea

Amenorrhea results from disorders in the hypothalamic-pituitary-ovarian axis in which levels of follicle-stimulating hormone (FSH) and luteinizing hormone (LH) are low despite low levels of estrogen. Due to this, reversion to the prepubertal state occurs, and the LH response to gonadotropin-releasing hormone (GnRH) is blunted. This blunted response is insufficient to maintain menstrual integrity, and amenorrhea results

How well did you know this?
1
Not at all
2
3
4
5
Perfectly
123
Q

A 14-year-old girl comes to the hospital because of anorexia nervosa. She acknowledged that she has not eaten anything for the past 4 days. Physical examination is notable for autonomic dysreflexia with positive chvostek and trousseau signs. She also complains of being too fat. Which of the following is associated with her condition?

A Low levels of chloride
B High levels of chloride
C Low levels of magnesium and calcium
D High levels of sodium
E Low potassium levels
A

C Low levels of magnesium and calcium

Chvostek and Trousseau signs = hypoparathyroidism –> low levels of calcium + magnesium

Anorexia nervosa (AN) is characterized by hypotension, bradycardia, hypothermia, dry skin, hypercarotenemia, lanugo body hair, acrocyanosis, atrophy of the breasts, swelling of the parotid and submandibular glands, peripheral edema, and thinning of the hair.

An increase in blood urea nitrogen (BUN) levels, which reflects a level of dehydration and a decreased glomerular filtration rate (GFR), is present. Electrolyte imbalances are secondary to vomiting, and potassium is most often affected. In cases where hypomagnesemia and hypocalcemia are severe, chvostek and trousseau signs may be positive

Low levels of sodium are associated with anorexia nervosa. This causes low blood pressure and affects renal function. Edema can be observed.

Low potassium levels are seen in anorexia nervosa, but would cause prolonged QT, flattened T waves, and arrhythmias

In anorexia nervosa, chloride levels are low. This can result to hyperirritability, tetany or muscular excitability, slowed respirations, and hypotension secondary to fluid loss.

How well did you know this?
1
Not at all
2
3
4
5
Perfectly
124
Q

A 19-year-old woman comes to the clinic because of amenorrhea. She acknowledged being on a dieting spree with episodes of induced vomiting. Her BMI is 15 kg/m2. Pregnancy test shows negative. Endocrine and coagulation studies are normal. Genital cultures and pelvic ultrasound are non-contributory. Which of the following is her likely diagnosis?

A Anorexia nervosa
B Hyperthyroidism
C Hypothyroidism
D Pituitary adenoma
E Primary amenorrhea
A

A) Anorexia nervosa

amenorrhea
BMI of 15 (normal 18.5 - 24.5)
negative pregnancy test
dieting spree with induced vomiting = binge/purge type of anorexia

How well did you know this?
1
Not at all
2
3
4
5
Perfectly
125
Q

An 18-year-old Caucasian girl comes to the hospital because of poor feeding habits. She explains she tries to avoid extra calories. On review of systems, she denies headache but admits to feeling fatigued and not having a menstrual period for the last 3 months. When asked to remove her baggy clothes for the physical exam, lanugo hair is observed. Which of the following is likely to be found on serum studies?

A Hypercalcemia
B Hyperkalemia
C Hypercarotenemia
D Hypernatremia
E Hypermagnesemia
A

C Hypercarotenemia

Anorexia nervosa (AN) is usually characterized by low levels of body electrolyte and protein. However, due to impaired liver functions, there could be hypercholesterolemia and hypercarotenemia.

How well did you know this?
1
Not at all
2
3
4
5
Perfectly
126
Q

An 18-year-old girl comes to the student health office because of irritableness and concerns about decision-making. On physical examination she appears very thin, with decreased muscle mass and dry skin. She is hypotensive with resting bradycardia. She denies any episodes of induced vomiting. Which of the following is associated with her condition?

A Calluses on hands or fingers
B Dental erosions
C Hirsutism
D Primary amenorrhea
E Secondary amenorrhea
A

E Secondary amenorrhea

Primary amenorrhea is the failure of menses to occur by age 16 years, in the presence of normal growth and secondary sexual characteristics. If by age 13 menses has not occurred and the onset of puberty, such as breast development, is absent, a workup for primary amenorrhea should start

Secondary amenorrhea = lack of period DUE TO anorexia

How well did you know this?
1
Not at all
2
3
4
5
Perfectly
127
Q

An adolescent female comes to the dentist because of acidic erosion of her teeth (photograph is shown below). She was observed to be emaciated with dry skin, hypercarotenemia, and lanugo body hair. She was also hypotensive with dry mucous membranes. An ECG showed prolonged QT interval. Which of the following electrolyte and acid/base disturbance is associated with her illness?

A Hyperkalemic metabolic acidosis
B Hyperkalemic metabolic acidosis and acute compensatory respiratory alkalosis
C Hypokalemic hypochloremic metabolic alkalosis
D Low serum calcium leading to arrhythmia
E No electrolyte or acid/base disturbance

A

C Hypokalemic hypochloremic metabolic alkalosis

How well did you know this?
1
Not at all
2
3
4
5
Perfectly
128
Q

An 18-year-old woman comes to the hospital because of poor feeding habits. She explains she tries to avoid extra calories. On review of systems, she denies headache but admits to feeling fatigued and not having a menstrual period for the last 3 months. When asked to remove her baggy clothes for the physical exam, lanugo hair is observed. Which of the following is ideal in managing her condition?

A Administering olanzapine
B Cognitive-Behavioral Therapy (CBT)
C Prescribing bupropion
D Prescribing orlistat
E Prescribing sertraline
A

avoids eating
fatigue
amenorrhea
lanugo

= anorexia nervosa

ANSWER = CBT (COGNITIVE BEHAVIORAL THERAPY)

Anorexia nervosa (AN) is an eating disorder which is mainly treated by family therapy and psychotherapy management. If this fails, olanzapine could be prescribed.

olanzapine (Zyprexa) = antipsychotic - weight gain
buproprion (Wellbutrin) = antidepressant
orlistat = weight loss drug (X)
sertraline (Zoloft) = antidepressant

How well did you know this?
1
Not at all
2
3
4
5
Perfectly
129
Q

A 26-year-old woman comes to the clinic because of amenorrhea. On examination, the woman has a BMI of 16. There is no abdominal tenderness or guarding. Vaginal and bimanual exams are grossly normal. A pregnancy test is negative. Serum levels of FSH, LH, TSH, prolactin are low. A basic metabolic panel returns results consistent with hypokalemia, hypochloremia, and compensated anion gap metabolic alkalosis. Which of the following is her likely diagnosis?

A Renal disease
B Cervical stenosis
C Pituitary adenoma
D Anorexia nervosa
E Androgen tumor
A

D Anorexia nervosa

BMI 16 = moderate
amenorrhea
hypokalemia, hypochloremia, metabolic alkalosis

Anorexia nervosa (AN) may present with low levels of FSH, LH, TSH, and prolactin. However, if bulimic episodes are also involved, these hormones may be normal. The patient’s history usually gives more direction.

How well did you know this?
1
Not at all
2
3
4
5
Perfectly
130
Q

A 15-year-old female comes to the clinic for her regular check-up. She is 165 cm (5 ft 5 in) tall and weighs 36.8 kg (85 lbs). She acknowledges taking low low-calorie diet with possibility of skipping meals. She has not had a period in 3 months, and she is worried that she might be pregnant. Serum beta-human chorionic gonadotropin is negative. Which of following is most appropriate in management?

A Discuss a nutrition plan and counseling for her anorexia nervosa

B Discuss a nutrition plan and counseling for her bulimia nervosa

C Tell her that the pregnancy test is probably false negative because she has not menstruated

D Performing endocrine work-up for secondary amenorrhea.

E Prescribing oral contraceptives

A

A Discuss a nutrition plan and counseling for her anorexia nervosa

restrictive anorexia nervosa
amenorrhea

How well did you know this?
1
Not at all
2
3
4
5
Perfectly
131
Q

A 16-year-old boy comes to the hospital because of extreme dieting and heavy exercise. The boy complains of being scared of gaining weight despite being told that his BMI of 16 is dangerous for his health. He has wasted features and fine downy hair. His family members are really concerned about him. What is the next best step in management?

A Olanzapine
B Family therapy
C Sertraline
D Haloperidol 
E Amiodarone
A

B Family therapy

restrictive anorexia nervosa

The first line approach is family therapy, followed by cognitive behavioral therapy.

How well did you know this?
1
Not at all
2
3
4
5
Perfectly
132
Q

A 6-year-old girl comes to the office because of behavioral changes that concern her mother. The mother says that her daughter has repeatedly been sent to the principal’s office because she disrupts the class by excessive talking, interrupting other children, not raising her hand and not staying in her seat. She does not follow instructions. At home, she can not sit still and has a difficult time completing her usual chores, often disregarding anything her parents tell her. She estimates this behavior started about a year ago and thought it would just be a phase. Which of the following is most commonly associated with the most likely diagnosis?

A Autism
B Chromosomal abnormalities
C Low IQ
D Oppositional defiant disorder
E Rett syndrome
A

The most likely diagnosis is attention deficit hyperactivity disorder (ADHD). These children have trouble paying attention, are easily distracted and have difficulty following instructions. These children also must demonstrate these symptoms in at least two settings.

Oppositional defiant disorder (ODD) is a common co-morbidity. Children with ODD display hostile behavior towards authority figures. The other choices are not commonly associated with ADHD.

How well did you know this?
1
Not at all
2
3
4
5
Perfectly
133
Q

A 7-year-old boy comes to the office because has been failing to do schoolwork and his mother is concerned about his ability to focus. He has difficulty paying attention in school, staying quietly seated during class, and often interrupts without waiting for his turn to speak. Which of the following criteria must be present in order to diagnose attention deficit hyperactivity disorder?

A Evidence of social skill impairment
B Overwhelming fear and worry in response to schoolwork
C Signs of impulsivity at school only
D Sudden rapid motor tics that persist for over a year
E Symptoms present for at least 6 months

A

E Symptoms present for at least 6 months

How well did you know this?
1
Not at all
2
3
4
5
Perfectly
134
Q

A 28-year-old woman comes to the office because she wants to be able to concentrate better at work. She says that she cannot get work done because she is easily distracted. She recently got into legal trouble for fighting with a stranger in a movie theater. She reports trouble concentrating in the past which resulted in poor grades in school, and has been taking Ritalin for this condition. Which of the following is a characteristic of her diagnosis?

A Below-average intelligence
B Decreased size of frontal lobes
C Onset before age 5
D Poor social interactions
E Symptoms always seen in adulthood
A

Attention deficit hyperactivity disorder (ADHD) is a developmental disorder characterized by limited attention span and poor impulse control. It is associated with a decrease in frontal lobe volume or metabolism, but does not result in a lower intelligence level.

How well did you know this?
1
Not at all
2
3
4
5
Perfectly
135
Q

A 40-year-old woman was just recently diagnosed with attention deficit hyperactivity disorder (ADHD). She says that she can become very focused when at work, but when not at work her thoughts are very scattered. When asked if she has experienced this in the past, she reports trouble concentrating the same way ever since she was a young student in elementary school. She has recently starting taking methylphenidate for her symptoms. In order to provide patient education on her new treatment, which of the following is true about this treatment?

A Hypotension is a common side effect
B Chronic use can lead to tardive dyskinesia
C Appetite suppression is a common side effect
D Bupropion is more effective in adults with ADHD than methylphenidate
E Sedation is a common side effect

A

C Appetite suppression is a common side effect

NOT A) Methlyphenidate is a CNS stimulant, and thus works to increase the levels of catecholamines in the synaptic cleft. Hypertension and tachycardia are common side effects of this stimulant medication.

NOT B) Tardive dyskinesia is characterized by uncontrolled stereotypical movements in the lower face, and can be seen as a result of long-term antipsychotic use.

NOT D) Stimulant medications and non-stimulant medications, like anti-depressants, are generally considered equally effective. However, stimulants and atomoxetine are still considered first-line treatment, which remains the same in children and adults

NOT E) CNS stimulants like methylphenidate and dextroamphetamine work by increasing the levels of catecholamines in the synaptic effect. Thus, one of its clinical uses is treating narcolepsy.

How well did you know this?
1
Not at all
2
3
4
5
Perfectly
136
Q

A mother brings in her 7-year-old son because she is worried about his behavior after his teacher called home. The boy has not been doing his assignments and frequently tells his teachers that he forgets about them. In addition, he tends to have a difficult time sitting still in class and will disrupt the other children. The behavior has been going on for about 8 months, but it is now getting worse so this was brought to the mother’s attention. His mother was surprised to hear about these issues since she has not noticed bad behavior at home. However, she does state that she needs to repeat herself several times when asking her son to complete his chores. What is the most likely diagnosis?

A Schizoid personality disorder 
B Attention deficit hyperactivity disorder 
C Conduct disorder     
D Oppositional defiant disorder     
E Antisocial personality disorder
A

B Attention deficit hyperactivity disorder

How well did you know this?
1
Not at all
2
3
4
5
Perfectly
137
Q

A 9-year-old boy comes to the office due to concern over his repeated difficulties in school. His mother explains that over the past 6 months, she has repeatedly met with his teacher due to his conduct at school. His teacher said that he constantly speaks loudly without raising his hand, does not clean up his assigned area, and does not finish his homework on time. During the office visit, the patient is constantly grabbing objects and is very irritable. The patient was prescribed methylphenidate. What is the mechanism of action of this drug?

A Monoamine oxidase inhibitor     
B 5-HT2 receptor antagonist     
C D2 receptor antagonist     
D Cholinesterase inhibitor     
E Blocks reuptake of dopamine and norepinephrine
A

Methylphenidate = Focalin = Stimulant =

E Blocks reuptake of dopamine and norepinephrine

How well did you know this?
1
Not at all
2
3
4
5
Perfectly
138
Q

A 6-year-old boy comes to the psychiatric department because of disruptive behavior that has lasted 8 months. His mother is concerned about his difficulties with schoolwork. She notes that he has always been a very active child and is easily distracted. He has been sent to the principal’s office multiple times for disrupting class and impulsive behavior towards his peers. Although the first-line pharmacotherapy for this condition is a stimulant, the mother is hesitant about using it. Which non-stimulant medication can also be used to treat this condition?

A Methylphenidate
B Lorazepam
C Fluoxetine
D Atomoxetine
E Dextroamphetamine/Amphetamine
A

D Atomoxetine

How well did you know this?
1
Not at all
2
3
4
5
Perfectly
139
Q

A 10-year-old girl comes to the office because she has had trouble concentrating in school this year. Her mother reports that she has not been doing her assignments. She tends to have a difficult time sitting still in class, following instructions, and disrupts the other children. She is not disruptive at home, but she will forget to do chores often according to her mother. Which of the following treatments should be recommended for this child?

A) Atomoxetine
B) Clonidine
C) Fluoxetine
D) Methylphenidate
E) Parental Counseling
A

D) Methylphenidate

Stimulant drugs are considered the first-line treatment for ADHD in children

Methylphenidate (or Ritalin) is usually the first line stimulant drugs used as medical therapy

How well did you know this?
1
Not at all
2
3
4
5
Perfectly
140
Q

Which of the following is most likely to be found on the laboratory evaluation of a patient admitted for treatment of anorexia nervosa?

a. Normal BUN
b. Ketonuria
c. Hypernatremia
d. Hyperkalemia
e. Decreased liver function tests (LFT)

A

b. Ketonuria

How well did you know this?
1
Not at all
2
3
4
5
Perfectly
141
Q

Which of the following is most likely to be a long-term complication associated with anorexia nervosa, even if the patient is able to regain ideal body weight after treatment?

a. Congestive heart failure
b. Pericardial effusion
c. Constipation
d. Superior artery mesenteric syndrome
e. Osteopenia

A

e. Osteopenia

How well did you know this?
1
Not at all
2
3
4
5
Perfectly
142
Q

When admitting a patient for further workup and treatment of anorexia nervosa, what laboratory finding would you most likely see?

a. Metabolic acidosis
b. Leukopenia
c. Hyperkalemia
d. Increased levels of FSH
e. Decreased serum bicarbonate level

A

b. Leukopenia

How well did you know this?
1
Not at all
2
3
4
5
Perfectly
143
Q

A 16-year-old girl comes to the office because of decreased mood and tiredness. Her mother says that she has not been doing any of her schoolwork, and often displays a sudden loss of interest in her extracurricular activities. Her pulse is 90/min and her blood pressure is 110/75. Her BMI is 22. Physical examination shows decaying enamel on teeth and drying skin. Thickened skin is found around the knuckles of her right hand. A complete metabolic panel would most likely show which of the following electrolyte disturbances?

A Hypercalcemia     
B Hyperkalemia     
C Hypocalcemia     
D Hypokalemia     
E Hyponatremia
A

D Hypokalemia

would see hyponatremia and hypocalcemia, but patient would have different symptoms

Although dry skin can be a sign of hypocalcemia, it is often accompanied by neurological symptoms such as headache and muscle cramping. It can be associated with many causes, including hypoparathyroidism, vitamin D deficiency, and acute renal failure.

Hyponatremia is most common electrolyte abnormality in hospitalized patients. It indicates excessive extracellular fluid relative to sodium in the body and can lead to nausea, vomiting, lethargy, and seizures.

How well did you know this?
1
Not at all
2
3
4
5
Perfectly
144
Q

A 19-year-old woman comes into the office because of abdominal pain and lack of energy. She says she has not been participating in activities at her college, and often feels overweight. However, her current BMI is 21. Six months ago, her BMI was 23. Physical examination shows tachycardia, erosion of dental enamel and callus formation over her knuckles. Which of the following acid-base disorders would most likely be seen in this patient?

A Metabolic acidosis     
B Metabolic alkalosis     
C Mixed respiratory and metabolic acidosis     
D Respiratory acidosis     
E Respiratory alkalosis
A

B Metabolic alkalosis

How well did you know this?
1
Not at all
2
3
4
5
Perfectly
145
Q

A 20-year-old woman comes to the emergency department because of suspected aspiration following an episode of choking. She reports abdominal pain. Physical examination shows erosions of the teeth, petechiae, and calluses on the knuckles of her right hand. At a BMI of 22.6, she weighs 140 lbs (63.5 kg) with a height of 5’6”. Her pulse is 90/min and blood pressure is 104/76. Based on the clinical findings, which of the following is the most likely diagnosis?

A Anorexia nervosa
B Bulimia nervosa
C Gastroesophageal reflux disease
D Major depressive disorder
E Premature ovarian insufficiency
A

B Bulimia nervosa

How well did you know this?
1
Not at all
2
3
4
5
Perfectly
146
Q

A 15-year-old girl comes to the office because of lethargy, irregular menses, and decreased mood. She has recently dropped out of all extracurricular activities at school and says she feels unattractive and overweight. Physical examination shows ragged-appearing enamel over the front teeth and thickened skin over her knuckles. She has a height and weight in the 95th and 85th percentile, respectively. Based on these findings, which is the most likely diagnosis?

A Anorexia nervosa
B Body dysmorphic disorder
C Bulimia nervosa
D Cushing syndrome
E Hypothyroidism
A

C Bulimia nervosa

How well did you know this?
1
Not at all
2
3
4
5
Perfectly
147
Q

A 17-year-old girl presents at her pediatrician’s office accompanied by her mother for a normal check-up. She has recently been forcibly vomiting after meals. She states that she has recently been very anxious in school and that vomiting is the only thing in life that she can control. Her vitals show no abnormalities and BMI is 22.1 kg/m2. In addition to family and cognitive behavioral therapy, which of the following is the most appropriate pharmaceutical interventions?

A Bupropion
B Clonazepam 
C Fluoxetine
D Olanzapine
E Ondansetron
A

C Fluoxetine

How well did you know this?
1
Not at all
2
3
4
5
Perfectly
148
Q

A 25-year-old woman with no significant past medical history comes to the office for a routine physical. She is married with two children and complains of feeling physically and emotionally drained by her demanding career in politics, as well as her dwindling relationship with her famous husband. Although she is concerned about her weight, her current BMI is 20. Her pulse is 98/min, temperature is 99.7 o F (37.6 oC), blood pressure is 115/76, and respirations is 12/min. Physical examination reveals bilateral parotid enlargement, tooth enamel erosion, and scarring on the knuckles. A complete metabolic panel will most likely find which of the following electrolyte imbalances?

A Decreased bicarbonate
B Hypercalcemia
C Hyperchloremia
D Hypokalemia
E Hyponatremia
A

D Hypokalemia

How well did you know this?
1
Not at all
2
3
4
5
Perfectly
149
Q

A 23-year-old man comes to the university health center because of his partner’s concern about his vomiting habits. The patient says that he feels guilty after eating and often feels overweight. Physical examination shows a BMI of 22 kg/m2, pulse is 90/min, and blood pressure is 110/60 mm Hg. His teeth enamel show signs of deterioration and knuckles on his hands have calluses. Which of the following is the appropriate next step in management?

A Behavioral weight loss therapy
B Couple's therapy
C Fluoxetine
D Haloperidol
E Pregabalin
A

C Fluoxetine

How well did you know this?
1
Not at all
2
3
4
5
Perfectly
150
Q

Which of the following agents has a mechanism of action that does not involve beta-receptors?

A. epinephrine
B. albuterol
C. propranolol
D. prazosin
E. ritodrine
A

D - Prazosin

prazosin is an alpha-blocker. Ritodrine and epinephrine cause beta sympathetic stimulation. Albuterol is a beta agonist. Propranolol is a beta blocker

beta receptors = heart + lungs
beta 1 = heart
beta 2 = lungs

How well did you know this?
1
Not at all
2
3
4
5
Perfectly
151
Q

A 19-year-old woman with no significant past medical history comes in for a pre-college physical examination required by her school. She has no complaints, and her review of systems is negative. Her temperature is 37.0°C (98.6°F), blood pressure 120/80 mm Hg, pulse 80/min, and respirations 10/min. Physical examination reveals a single nodule on the left lobe of the thyroid gland. Her thyroid-stimulating hormone is within normal limits.

Which of the following is the most appropriate next step in management?

CT scan of the thyroid gland
Fine-needle biopsy
Repeat examination in 6 months
Thyroid uptake scanning

A

Fine-needle biopsy

The correct answer is B. The first test for a thyroid nodule is TSH; if normal, the most appropriate next step is FNA biopsy - which can be done safely and without complication. An ultrasound can be used to differentiate between solid and cystic structure and can be used to guide the needle aspiration. The FNA biopsy provides a definitive diagnosis in most cases and is the most helpful test in guiding management.

CT scan (choice A) is important in the workup of a patient who has a goiter and is complaining of obstructive symptoms such as shortness of breath or difficulty swallowing. It has no role in the initial workup of a thyroid nodule. If the patient has thyroid cancer, the CT scan of the thyroid can help stage the cancer.

Serial physical examinations (choice C) are not acceptable management. Thyroid cancer is curable in its early stages, so efforts should be made to rule out carcinoma in these patients

PEARL: A patient who has a thyroid nodule should have a full thyroid function evaluation performed. The first test performed should be the TSH; a suppressed level suggests hyperthyroidism and a hot nodule, which lessens the possibility for cancer. The next test should be a fine-needle biopsy (if the TSH is not suppressed). Histology will identify whether the patient has cancer and also will identify the type of cancer that the patient has. A thyroid scan can help to identify the nodule and whether it is hyper- or hypofunctioning.

How well did you know this?
1
Not at all
2
3
4
5
Perfectly
152
Q

A 58-year-old woman is found on a routine medical check-up to have a serum calcium level of 11.8 mg/dL, where the upper limit of normal is 9.5 mg/dL. Repeated determinations confirm values between 10.9 and 12.2 mg/dL, and she is found to have elevated concentrations of parathyroid hormone. She is asymptomatic, has no pertinent family history, and has no evidence of renal stones or bone disease. She is offered the option of elective parathyroidectomy, but she declines and elects to have close medical follow-up.

In addition to monitoring serum calcium levels, which of the following therapeutics should be given?

A

d

How well did you know this?
1
Not at all
2
3
4
5
Perfectly
153
Q

A 6-year-old male is brought into your office for evaluation of his gait. You note that when ambulatory, the medial aspects of his knees are far apart and his feet are unusually close together. Which of the following best describes this condition?

A. cubitus valgus
B. cubitus varus
C. genu valgus
D. genu varum
E. Legg-Calve-Perthes disease
A

Answer: D = Genu varum

This is genu varum (bow legs). Valgus deformity forms an “L” in the knock-kneed child and has an “L” in the middle of it (try it, it works)

How well did you know this?
1
Not at all
2
3
4
5
Perfectly
154
Q

A female presents complaining of a grayish, watery vaginal discharge. The KOH prep does not reveal any hyphae. Clue cells are identified on the wet mount. A paucity of WBCs are noted. What diagnosis is most likely?

A. Gonorrhea
B. Chlamydia
C. Candidiasis
D. bacterial vaginosis
E. Trichomoniasis
A

D. bacterial vaginosis

Clue cells mean BV

FISHY, grey, scant, THIN, STICKY

PH > 4.5 (BACTERIA = BASIC)

Wet-prep shows epithelial cells with bacteria coating on surface = “clue cells”

KOH testing causes the release of amine odor (+ whiff test)

First Line Treatment:
Metronidazole: 500 mg PO b.i.d. × 7 days*****
Metronidazole gel 0.75%: 5 g intravaginally daily × 5 days
Clindamycin 2% cream: 5 g intravaginally qhs × 7 days

How well did you know this?
1
Not at all
2
3
4
5
Perfectly
155
Q

You note polygonal, purple, popular lesions involving the flexor surfaces of a patient’s wrists. The patient has no other lesions. The lesions appear to follow a linear pattern of distribution. What is this phenomenon?

A. Auspitz sign
B. Koebner’s phenomenon
C. Darier’s sign
D. Gorlin’s sign
E. Gottron’s papules
A

B. Koebner’s phenomenon

Koebner’s phenomenon is described - it suggests psoriasis = refers to skin lesions appearing on lines of trauma / irritation

Auspitz sign is also present in psoriasis where the scales bleed when removed.

Darier’s sign is in allergic patients who develop urticaria when the skin is rubbed.

Gottron’s papules are purple papules on the fingers seen in dermatomyositis.

Gorlin’s sign is seen in Ehlers-Danlos when a patient can touch the tip of the nose with his tongue

How well did you know this?
1
Not at all
2
3
4
5
Perfectly
156
Q

Which of the following is most likely on history of a patient with schizoid personality disorder?

a. Lives alone and is single
b. Employed in a highly competitive sales job
c. Incarceration
d. Paranoid ideations

A

a. Lives alone and is single

avoid social interaction because they aren’t interested in getting to know others. Results in isolation; they want to be isolated. Find less pleasure in physical contact - like sex or holding hands. Less likely to seek out relationships than the average person
- may also have a flat affect or emotional blunting- don’t show positive or negative emotions

How well did you know this?
1
Not at all
2
3
4
5
Perfectly
157
Q

After watching a particular movie 5 months ago, a young man becomes infatuated with an actress in the film and believes the movie is really about their destiny to be together. He begins to stalk her and sees a conspiracy unfolding that is keeping them apart. When she continually ignores his phone calls and poems he believes he will win her love and catch her attention by assassinating the current president. He believes there is no chance that his plot will fail. He suffers from no visual or auditory hallucinations. Socially, he has few relationships and is awkward and paranoid, which causes him social anxiety. These fanciful beliefs influence his behaviors and he begins to obsess about his plan. Based on this information, which of the following is the most likely diagnosis?

A Schizoid personality disorder
B Schizophrenia
C Schizotypal personality disorder
D Paranoid personality disorder
E Schizophreniform disorder
A

C Schizotypal personality disorder

Schizotypal personality disorder is a cluster A personality disorder characterized by odd behavior and magical thinking. They lack delusions or hallucinations, unlike schizophrenia.

People with schizotypal PD often exhibit ideas of reference, are socially withdrawn/awkward/anxious or paranoid, and may have abnormal perceptual experiences and constricted affect. They may also dress in an odd manner. Many of these characteristics manifest in this patient.

  • Ideas of Reference - belief that everything, including innocuous events or coincidences in the world, directly relate to ones own DESTINY
How well did you know this?
1
Not at all
2
3
4
5
Perfectly
158
Q

A 42-year-old man comes to the office for blood work because he thinks his fiancé has been “poisoning” him for the last 6 months. He thinks she is planning on killing him in order to inherit his money after they get married. When asked how he knows this, he says he hears his family members whispering about it and they are unaware he can hear them. He says that he has to constantly check to see if his fiancé’s spies are watching him. He also says that people at his work have been commenting secretively about how thin and gaunt he looks. He does not eat or sleep much because he is so busy watching his back. Which of the following is the most likely diagnosis?

A Antisocial personality disorder
B Borderline personality disorder
C Paranoid personality disorder
D Schizoid personality disorder     
E Schizotypal personality disorder
A

C Paranoid personality disorder

Patients with paranoid personality disorder hold extremely improbable and odd beliefs that affect their functioning, but are able to function well in areas that are outside their paranoid suspicions.

How well did you know this?
1
Not at all
2
3
4
5
Perfectly
159
Q

A 35-year-old man comes to the office for an annual visit. He says he has been under a lot of stress lately at home and at work and is afraid that he will lose his job since his co-workers are trying to sabotage him. He does not get along with his co-workers and believes it is because he does a better job. At home, he suspects that his wife is cheating on him because he does not make enough money to support her. He denies any auditory and visual hallucinations, special powers, ideas of reference, or belief that others can hear his thoughts. His wife says he has always had a difficult time confiding in others and would hold grudges against friends and co-workers. Which of the following is the most likely explanation for this man’s symptoms?

A Narcissistic personality disorder     
B Paranoid personality disorder     
C Paranoid schizophrenia     
D Schizoid personality disorder     
E Schizotypal personality disorder
A

B Paranoid personality disorder

Patients with paranoid personality disorder are suspicious of others, display pervasive distrust, and often interpret motives as malicious. They tend to read hidden threatening meanings into remarks and react angrily to perceived insults. The paranoid thoughts of these patients are merely suspected and not believed with certainty.

How well did you know this?
1
Not at all
2
3
4
5
Perfectly
160
Q

A 45-year-old man comes to the physician because of concerns about his prostate after experiencing frequent urination during the day and night. He very rarely sees his primary care physician but has been a patient there for several years. He is a quiet man and works as a laboratory technician usually during the overnight shift. He is single and has never had a relationship before. He has no close friends, but he expresses no desire to have friends. There is no evidence of hallucinations or a thought disorder. Which of the following is statements most accurately explains the patient?

A He most likely has a Cluster A personality disorder

B He most likely has a Cluster B personality disorder

C He most likely has a concomitant substance abuse disorder

D He most likely has a genetic association with anxiety disorders

E He most likely has a pattern of disregard for the rights of other people

A

A He most likely has a Cluster A personality disorder

= Schizoid personality disorder

Patients with a schizoid personality disorder will be detached from others and have little desire for close relationships. People with this disorder prefer to be alone and take pleasure in few activities.

How well did you know this?
1
Not at all
2
3
4
5
Perfectly
161
Q

A 35-year-old man comes to the office for an annual visit. He says he has been under a lot of stress lately at home and at work. He does not get along with his co-workers and believes it is because he does a better job. He suspects that his wife is cheating on him with the neighbor because he does not make enough money to support her. He denies any auditory and visual hallucinations, special powers, ideas of reference, or belief that others can hear his thoughts. His wife says that he has always had a difficult time confiding in others and would hold grudges against friends and co-workers. Which defense mechanism is this man most likely to display?

A Acting out     
B Denial
C Dissociation
D Projection
E Regression
A

D Projection

Paranoid personality disorder is a cluster A personality disorder. Projection is the major defense mechanism seen in this disorder and is characterized by attributing thoughts or feelings to others.

Projection is the major defense mechanism seen in this personality disorder, in which patients take an unacceptable internal impulse, wish, or desire and attributes it to an external source. For example, a man may think about cheating on his wife so he blames his wife for having an affair. Another example would be a boy who talks about his stuffed animal as having certain feelings, which are really what the boy feels. Projection is an immature defense mechanism and paranoia tends to result from the use of projection

How well did you know this?
1
Not at all
2
3
4
5
Perfectly
162
Q

A 30-year-old woman comes to the clinic for an annual checkup. She is dressed in mismatching clothing and believes that she has telepathic abilities. Her family has previously stated that she has been superstitious since she was a child, and she lacked close friendships due to her peculiar behavior. Which of the following is this patient’s most likely personality disorder?

A Avoidant personality disorder      
B Borderline personality disorder      
C Histrionic personality disorder     
D Schizoid personality disorder      
E Schizotypal personality disorder
A

E Schizotypal personality disorder

Schizotypal personality disorder is a cluster A disorder characterized by eccentric beliefs, magical thinking, and constricted affect. These patients often lack close relationships.

Schizotypal personality disorder is characterized by at least five of the following:

1) odd beliefs or magical thinking
2) perceptual experiences
3) odd thinking and speech
4) suspiciousness of others
5) constricted affect
6) odd behavior
7) lack of close friends
8) social anxiety.

These patients often have BIZARRE ATTIRE, SPEECH AND BEHAVIOR. They may wear MISMATCHING CLOTHING and say UNUSUAL / MADE-UP WORDS. They often have IDEAS OF REFERENCE in which they interpret events that are completely unrelated as having specific meaning toward themselves. The treatment of choice, as with all personality disorders, is psychotherapy.

  • Ideas of Reference - belief that everything, including innocuous events or coincidences in the world, directly relate to ones own destiny
How well did you know this?
1
Not at all
2
3
4
5
Perfectly
163
Q

You’re examining a 35-year-old man who is exhibiting acute psychotic behavior. He also has delusions and hallucinations. He has a history of a hypertrophic cardiomyopathy as well. Which of the following medications would you be very hesitant to give this man?

(A) Haloperidol (Haldol)
(B) Olanzapine (Zyprexa)
(C) Lithium carbonate (Lithobid)
(D) Oxazepam (Serax)
(E) Diazepam (Valium)
A

(A) Haloperidol (Haldol)

Haloperidol (Haldol) is a typical antipsychotic that can cause cardiac arrhythmias, specifically by prolonging the QT interval. You wouldn’t give it to someone with a hypertrophic cardiomyopathy. A good choice to use for this patient would be olanzapine (Zyprexa).

  • Lithium carbonate is used for bipolar disorder
  • Oxazepam and diazepam are used for anxiety and panic disorders. These other meds may be used adjunctively; although the person still needs to be given a medication to control his or her psychotic behavior).
How well did you know this?
1
Not at all
2
3
4
5
Perfectly
164
Q

You’re seeing a 35-year-old woman for evaluation of major depressive disorder. You want to begin treatment with an antidepressant. Which of the following conditions do you need to exclude before you treat her?

(A) Panic disorder
(B) Seasonal affective disorder
(C) Borderline personality disorder
(D) Bipolar disorder
(E) Dementia
A

(D) Bipolar disorder

In anyone you think has major depressive disorder, make sure you consider that the person may have bipolar disorder. By giving a bipolar person an antidepressant, you can unmask a manic episode

Choice (A), panic disorder, wouldn’t be a concern here, and Choice (B), seasonal affective disorder, makes no sense. Nothing in the question suggests that her depression is related to seasons of the year (for example, better in June than in January). Note that Choice (C), borderline personality disorder, can be associated with depression. The woman in the question is typically too young to have dementia, Choice (E), and nothing in the question makes you think that she does.

How well did you know this?
1
Not at all
2
3
4
5
Perfectly
165
Q

You’re seeing a 25-year-old man in the ER who was recently started on metoclopramide (Reglan). He presents with a very high fever and a change in mental status. On physical examination, his muscles are rigid. His body temperature is 40.0°C (104°F). You obtain a stat CPK level, and it is 50,000. This man is in severe trouble. What is the most likely diagnosis?

(A) Serotonin syndrome
(B) Tyramine reaction
(C) Neuroleptic malignant syndrome (NMS)
(D) Drug overdose
(E) Bacterial infection
A

(C) Neuroleptic malignant syndrome (NMS)

This man has neuroleptic malignant syndrome (NMS), which you usually see after someone takes a neuroleptic or antipsychotic. This person took metoclopramide (Reglan). He presents with fever and a change in mental status. His muscles are rigid because of the medication. A person with neuroleptic malignant syndrome has an elevated CPK level.

Choice (A) is wrong because serotonin syndrome (SS) occurs with medications that can cause excessive serotinergic activity. Examples of such medications include SSRIs, tricyclics, certain over-the-counter medications, and anti-Parkinson’s medications. Symptoms can include confusion, fever, and tachycardia. You usually see Parkinson’s and hyperactive reflexes with SS, as compared to the muscle rigidity you seen in neuroleptic malignant syndrome.

A tyramine reaction, Choice (B), concerns the interaction of a MAO inhibitor with anything that contains tyramine, such as wine and cheese. Choice (D), a drug overdose, is nonspecific, and the question gives no evidence of a bacterial infection, Choice (E).

How well did you know this?
1
Not at all
2
3
4
5
Perfectly
166
Q

You’re going to school with a colleague who swears that he has every medical condition you are studying. In school, you’re currently studying the GI system. He’s having some diarrhea and fears he may have colon cancer. He beseeches the physicians to allow him to have a colonoscopy. This behavior could be an example of what psychiatric condition?

(A) Malingering
(B) Factitious disorder
(C) Somatoform disorder
(D) Hypochondriasis
(E) Social phobia
A

(D) Hypochondriasis

How well did you know this?
1
Not at all
2
3
4
5
Perfectly
167
Q

You are evaluating a 55-year-old man who comes to the clinic complaining of a fever. You take his temperature, and it is 39.4°C (103°F). You find out that this person may have somehow heated the thermometer with a lighter to induce a false reading. This behavior could be an example of what psychiatric condition?

(A) Malingering
(B) Factitious disorder
(C) Somatoform disorder
(D) Hypochondriasis
(E) Social phobia
A

(B) Factitious disorder

This person is actively trying to mess with the equipment to cause the healthcare provider to think a disease or medical condition is present when it really isn’t. Therefore, he has factitious disorder, Choice (B). This condition differs from Choice (A), malingering, in which the person is making up symptoms or acting sick. With malingering, the person isn’t trying to sabotage medical equipment or do something illegal. Concerning Choice (C), a somatoform disorder is a mental disorder causing physical symptoms; a somatoform disorder is not a syndrome that’s purposely made up. With Choice (D), hypochondriasis, the affected person thinks that he or she has every medical condition. With Choice (E), the person is afraid of a particular aspect of society, such as when someone is afraid to go on stage or be seen in public places.

How well did you know this?
1
Not at all
2
3
4
5
Perfectly
168
Q

What’s the most immediate treatment needed in a patient suffering from neuroleptic malignant syndrome?

(A) Intravenous diuretics
(B) Intravenous haloperidol (Haldol)
(C) Intravenous metoclopramide (Reglan)
(D) Warming blanket
(E) Dantrolene sodium
A

(E) Dantrolene sodium

The immediate choice for the treatment of neuroleptic malignant syndrome (NMS) is Choice (E) — you need to do something to stop the muscle rigidity, so you give dantrolene sodium. Bromocriptine can also be given.

Concerning Choice (A), the treatment of choice is intravenous fluids, not diuretics. You don’t want to dehydrate this person. Choices (B) and (C) would be contraindicated: You would not give a neuroleptic to someone with neuroleptic malignant syndrome. Concerning Choice (D), you’d use a cooling blanket, not a warming blanket, because the person would be hyperthermic.

How well did you know this?
1
Not at all
2
3
4
5
Perfectly
169
Q

A 46 year-old-man presents to a primary care physician with a chronic cough. He reports that he has not seen a physician in 25 years, but he has not been sick during this time. He lives in a cabin in the woods, and although he does not have any friends, he reports that he does not feel lonely. He is well-groomed, dressed normally, and does not have any bizarre beliefs. He describes his mood as “fine”, and his speech is fluid, albeit limited in amount. His thought process is linear, and his thought content is appropriate. He denies any visual or auditory hallucinations. Which of the following is true about this patient’s psychiatric condition?

A It has a high correlation with schizophrenia
B The patient should be treated with an antidepressant
C No specific treatment is needed at this time
D The patient is likely very responsive to criticism and praise
E The patient likely suffers from a co-morbid substance use disorder

A

C No specific treatment is needed at this time

= Schizoid personality disorder

This patient’s clinical presentation is consistent with schizoid personality disorder (SPD), which does not require treatment, unless the patient presents with a co-morbid Axis I disorder, such as depression, psychosis, or anxiety.

dressed normally / well groomed / no bizarre beliefs / not lonely when alone / no friends / fluid speech = not schizotypal

No hallucinations = no schizophrenia

How well did you know this?
1
Not at all
2
3
4
5
Perfectly
170
Q

Frances, an only child, is a 20-year-old receptionist in a wholesale dress house who feels that she hasn’t been discovered yet by the fashion world. She has done some showroom modeling for customers and is always daydreaming that she will be swept off her feet by the next wealthy man who comes through the door. Eating in the local restaurants, she always takes the opportunity to complain about the clumsy waiters, the inept service, or the poor quality of the food. Frances engages in a rich fantasy life in which she is adored and loved by crowds of onlookers and where she gets even with all the people whom she perceives have been mean to her.

A. Histrionic Personality Disorder
B. Narcissistic Personality Disorder
C. Borderline Personality Disorder
D. Dependent Personality Disorder
E. Immature Personality Disorder
A

B. Narcissistic Personality Disorder

How well did you know this?
1
Not at all
2
3
4
5
Perfectly
171
Q

Matt is an active, physically strong, and exceptionally tall 18-year-old 12th grader who stays out late at night with his friends, despite his parents forbidding it. He ran away from home several times when he was younger and is truant from school frequently. His grades are poor and he has been left back and placed in special education classes. The school psychologist gave him a diagnosis of Conduct Disorder when he was 1C. When he attends school now, he threatens and intimidates students and teachers and seems to enjoy getting into fights. He has attacked one student with a brick and boasts that he carries a knife and has a gun hidden on school grounds. Matt has never expressed feelings of remorse for his actions. A good talker, he often engages in conning his teachers and enjoys the attention it seems to get from the other kids. A favorite weekend activity in which Matt has been engaging for the past several years is shoplifting. Your diagnosis would be:

A. Oppositional Defiant Disorder
B. Conduct Disorder
C. Antisocial Personality Disorder
D. Disruptive Behavior Disorder Not Otherwise Specified (NOS)
E. Attention-Deficit/Hyperactivity Disorder (NOS)

A

C. Antisocial Personality Disorder

How well did you know this?
1
Not at all
2
3
4
5
Perfectly
172
Q

A 32-year-old Hispanic man comes to your general practice office for the first time. In the course of taking his history, the man mentions several times that he feels exploited by his employees and that he doubts their loyalty to his company. “In fact,” he says, “I have even overheard them talking about how they would change the company if I were no longer in charge.” He says he is consequently reluctant to confide in others, especially in his business partner, and he even wonders if he should be telling you all this since you have just met. Based on this information alone, you might suspect that his man is suffering from which personality disorder?

A.  Paranoid Personality Disorder
B.  Schizoid Personality Disorder
C.  Schizotypal Personality Disorder
D.  Antisocial Personality Disorder
E.  Histrionic Personality Disorder
A

A. Paranoid Personality Disorder

How well did you know this?
1
Not at all
2
3
4
5
Perfectly
173
Q

A 30-year-old woman is brought to the emergency department following a minor car accident. Physical examination is unremarkable except for a small laceration above her eyebrow. As the physician is talking to the woman she becomes very emotional and theatrical when describing the car accident, but would not provide further details about the accident other than talking about how it was a “near death experience.” As the physician proceeds with the examination, the patient becomes less preoccupied with her “near death experience” and very provocative toward the examiner. Her toxicology screen is negative. Her past psychiatric history is negative for depression and any suicide attempts. She is single and lives alone but states that she has been in several intimate relationships. She works in theater and has been in several local plays. Which of the following is the most likely diagnosis?

A Borderline personality disorder      
B Dependent personality disorder 
C Histrionic personality disorder      
D Narcissistic personality disorder 
E Schizotypal personality disorder
A

C Histrionic personality disorder

Histrionic personality disorder is characterized by a pervasive pattern of excessive emotionality and attention seeking and includes features such as inappropriate sexual behaviors, shallow or exaggerated expression of emotion, and consistent use of physical appearance to draw attention to self.

How well did you know this?
1
Not at all
2
3
4
5
Perfectly
174
Q

A 32-year-old man comes to the emergency department because of multiple non-lethal stab wounds. He is currently imprisoned for murder and armed robbery. After his condition stabilizes, he insists on staying in the hospital until he is fully cured. He threatens to harm the attending physician if she endorses his return to prison. Which of the following is his most likely diagnosis?

A Adjustment disorder with mixed features
B Antisocial personality disorder
C Bipolar disorder, manic
D Borderline personality disorder
E Narcissistic personality disorder
A

B Antisocial personality disorder

Antisocial personality disorder is characterized by a pervasive pattern of disregard and violation of the rights of others and the rules of society

This patient’s history of criminal activity, evidence of recent physical altercation, and acting out with threat of harm toward the attending physician is most suggestive of antisocial personality disorder. Patients with antisocial personality disorder demonstrate a pervasive lack of empathy and remorseless disregard toward the rights and well-being of others in their behavior. They often become involved in criminal activities and act in an impulsive and irresponsible fashion.

How well did you know this?
1
Not at all
2
3
4
5
Perfectly
175
Q

A 25-year-old woman is brought to the emergency department after telling her roommate that she wants to just “go away” following an argument with her boyfriend. Her roommate states that the patient has few close friends and often believes that her relationships are more intimate than they actually are. She is also easily influenced by her friends and often is perceived as being shallow. She denies depressed mood, grandiosity, weight changes, sleep disturbances or suicidal ideation prior to the argument. What other trait is most likely present in this patient?

A A grandiose sense of self-importance
B A style of speech that is excessively impressionistic and lacking in detail
C Needs others to assume responsibility for most areas of her life
D Transient, stress related paranoia
E Views herself as socially inept

A

B A style of speech that is excessively impressionistic and lacking in detail

= HISTRIONIC PERSONALITY DISORDER

CLINICAL MANIFESTATIONS
⦁ Self-absorbed
⦁ “temper tantrums”
⦁ efforts to draw attention to themselves with the need to be the center of attention
⦁ often inappropriate, sexually provocative, seductive with shallow or exaggerated emotions
⦁ seek reassurance + praise often
⦁ may believe their relationships are more intimate than they actually are
⦁ can be suggestible = easily influenced by others or circumstances

How well did you know this?
1
Not at all
2
3
4
5
Perfectly
176
Q

A 28-year-old man comes to the emergency department because of a recent suicide attempt. He was found after taking an unknown amount of sleeping pills. He recently lost his job, and has feelings of low self-worth. Six months ago, he was seen by a psychiatrist for episodes of self-mutilation due to feelings of being “empty” and poor self-esteem. Over the last three years, he has accumulated debt from excessive gambling, and has been forced to sell his home and car. What defense mechanism is this patient most likely to display during his hospital stay?

A Dissociation     
B Sublimation     
C Splitting     
D Acting out     
E Displacement
A

C Splitting = Borderline Personality Disorder

Patients with borderline personality disorder commonly use the defense mechanism of splitting. Splitting is defined as labeling people as all good or all bad. For example, he may only express his concerns to his nurses, but may view all doctors as mean and non-understanding of his needs.

Borderline personality disorder is a pattern of abnormal behavior with extreme fear of abandonment, and unstable relationships with other people. This pattern of behavior typically begins in early adulthood. Substance abuse, depression, and eating disorders often co-exist with borderline personality disorder

How well did you know this?
1
Not at all
2
3
4
5
Perfectly
177
Q

A 20-year-old woman comes to the clinic because of feelings of emptiness and the desire to self-harm. Physical examination and laboratory tests show no abnormalities. A clinical assessment is performed and she is diagnosed with borderline personality disorder. Which of the following groups of characteristics would most likely be seen in this patient?

A Unstable self-image, impulsivity, suicidal behavior
B Arrogance, feelings of grandiosity, lack of empathy
C Lack of personal relationships, promiscuity, splitting behavior
D Attention seeking, inappropriate sexual seduction, rapidly shifting emotions
E Intense interpersonal relationships, rapidly shifting emotions, submissive

A

A Unstable self-image, impulsivity, suicidal behavior

Patients with borderline personality disorder are unstable and have intense interpersonal relationships, frantic efforts to avoid abandonment, unstable self-image, impulsivity that leads to self-damaging behavior, such as sexual promiscuity or substance abuse, suicidal behavior, rapidly shifting affect, intense anger, or psychotic features.

How well did you know this?
1
Not at all
2
3
4
5
Perfectly
178
Q

An 18-year-old woman comes to the emergency department because her mother states that “she just can’t deal with her anymore.” The patient is reserved and reluctant to answer questions. Her mother says her daughter regularly abuses alcohol, and has many sexual partners. The patient says her most recent boyfriend is “the love of her life and would do anything” for her. She does not have a job and is home schooled, due to an inappropriate relationship with a former teacher. Physical examination shows numerous cuts on her forearm. Which of the following is most likely present in this patient?

A Obsessive compulsive personality disorder
B Histrionic personality disorder
C Schizoid personality disorder
D Borderline personality disorder
E Narcissistic personality disorder
A

D Borderline personality disorder

Borderline = hallmarked by a pattern of instability in relationships, self-image, identity, behavior, and affects often leading to self-harm and impulsivity. Substance abuse is common. Self cutting is also very common.

How well did you know this?
1
Not at all
2
3
4
5
Perfectly
179
Q

A 25-year-old woman comes to the emergency department because of cuts she made with a razor blade to her left forearm and right hip. She says her boyfriend broke up with her recently, and she has been feeling very lonely and empty. She reports frequent cutting with a razor blade on herself when feeling this way in the past. She lives with a roommate, but reports constant fighting with her. Which of the following is the most likely diagnosis?

A Bipolar disorder type II
B Borderline personality disorder
C Dysthymic disorder
D Major depressive disorder
E Schizoid personality disorder
A

B Borderline personality disorder

Borderline personality disorder is characterized by instability of interpersonal relationships, self-image, and emotions, as well as by impulsivity across a wide range of situations, causing significant impairment or subjective distress.

How well did you know this?
1
Not at all
2
3
4
5
Perfectly
180
Q

A 23 year-old man comes to the psychiatry clinic, accompanied by his mother, because she is concerned after seeing cut marks on his arms. His mother says that his life is chaotic and unstable. He often brings nice girls home whom he “likes a lot” to meet his family, only to come home days later, slam his bedroom door shut, and yell that he “hates all of them” and “just wants to be alone”. He has difficulty keeping jobs because of his angry outbursts, but also has no trouble finding a new one because he can be very friendly and professional in interviews. Which of the following is the most likely diagnosis?

A Schizotypal personality disorder
B Antisocial personality disorder
C Avoidant personality disorder
D Histrionic personality disorder
E Borderline personality disorder
A

E Borderline personality disorder

Borderline personality disorder is characterized by instability of interpersonal relationships, self-image, and emotions, as well as by impulsivity across a wide range of situations, causing significant impairment or subjective distress.

While this patient’s ability to charm in interviews could point to antisocial personality disorder, and his emotional outbursts could point to histrionic personality disorder, the constellation of symptoms (self-mutilation, mood instability, and splitting with regard to girls) point to borderline personality disorder.

How well did you know this?
1
Not at all
2
3
4
5
Perfectly
181
Q

A 23-year-old woman is urgently reviewed by the psychiatric crisis team because of a suicide attempt an hour ago. Her mother states that she found her daughter on the floor cutting her wrists because her boyfriend broke up with her today. Medical history includes childhood sexual abuse and multiple previous admissions for suicide attempts. The patient states that she is in a good mood today because the man delivering her hospital food gave her an extra jello cup and she believes that he is in love with her. She states that he is “so much better than her ex-boyfriend”. Examination shows the patient sitting up in bed applying makeup. There are multiple scars on her wrists from previous suicide attempts. Which of the following personality disorders is this patient most likely exhibiting?

A Dependent
B Narcissistic
C Histrionic
D Borderline
E Avoidant
A

D Borderline

Borderline personality disorder is a DSM-V diagnosis characterized by seeking approval from others for self-identity, impairments in empathy and intimacy, and pathological personality traits such as grandiosity and attention seeking.

Patients with borderline personality disorder have instability in their interpersonal relationships, self-image, and mood. They also often exhibit impulsive behavior. The gender ratio for this disease is 3:1 with more females being affected. The onset of the disease typically is in adolescence and early adulthood. There is usually a history of childhood trauma associated with borderline personality disorder. Patients typically have comorbid diseases such as depression, anxiety, substance abuse, and eating disorders. They typically have a history of suicide attempts or threats which are meant to bring on attention or get what they desire. On physical exam there is often multiple burns or markings from past suicide attempt. They also tend to see people as all “good” or all “bad,” and this is a psychiatric phenomenon known as splitting.

How well did you know this?
1
Not at all
2
3
4
5
Perfectly
182
Q

A 32-year-old female is caught by the police because of identity theft. When confronted by the police, she lies about the accusations and tries to run. She also tries to escape custody by putting the lives of the police officers in danger. Which of the following personality disorder is she displaying?

A Schizotypal personality disorder
B Borderline personality disorder
C Schizoaffective personality disorder
D Schizoid personality disorder
E Antisocial personality disorder
A

E Antisocial personality disorder

Antisocial personality disorder is always characterized by a history of conduct disorder before the age of 15. If left untreated, this will cause antisocial personality disorder.

The DSM-V defines antisocial personality disorder as failure to conform to social norms with respect to lawful behaviors, as indicated by repeatedly performing acts that are grounds for arrest; deceitfulness, as indicated by repeated lying, use of aliases, or conning others for personal profit or pleasure; impulsivity or failure to plan ahead; irritability and aggressiveness, as indicated by repeated physical fights or assaults; reckless disregard for the safety of self or others; consistent irresponsibility, as indicated by repeated failure to sustain consistent work behavior or honor financial obligations; lack of remorse, as indicated by being indifferent to or rationalizing having hurt, mistreated, or stolen from another; the individual is at least age 18 years; there is evidence of conduct disorder with onset before age 15 years; and the occurrence of antisocial behavior is not exclusively during the course of schizo­phrenia or bipolar disorder.

How well did you know this?
1
Not at all
2
3
4
5
Perfectly
183
Q

A 17-year-old girl comes to the emergency department with her parents because she inflicted multiple cuts on her forearms once her boyfriend left her. Her primary physician is notified, he arrives to her room and she says: “I am so happy that you are here, since you are the only person in this world that gets me.” The next day, the doctor goes to follow-up the patient’s progress. As he tries to talk to her, the patient says: “I will not confide in you anymore because you do not care at all about what is happening to me.” She also adds that, “The nurses are fantastic and they take really good care of me.” What type of defense or coping mechanism is this patient most likely using?

A Denial
B Displacement 
C Projection
D Regression
E Splitting
A

E Splitting

Splitting is a primitive coping mechanism and the key symptom for patients of borderline personality disorder, such as the one described in this clinical vignette. Splitting is an ego mechanism that fluctuates between idealization and devaluation of objects, actions, and persons.

Splitting is an ego mechanism that fluctuates between idealization and devaluation of objects, actions, and persons. Individuals who use splitting as a coping mechanism tend to think in extremes. In psychoanalytic theory, people with borderline personality disorder are not able to integrate the good and bad images of both self and others. This primitive coping mechanism is the key symptom for patients of borderline personality disorder such as the one described in this clinical vignette. Splitting follows “All or None” phenomenon. The doctor is very good one day, and becomes very bad another day. It’s most commonly seen in borderline personality disorder.

How well did you know this?
1
Not at all
2
3
4
5
Perfectly
184
Q

A 27-year-old woman and her boyfriend of 2 years are referred to couples counseling. The boyfriend is thinking of proposing, however there has been strain on their relationship because of the patient’s inappropriate behavior. The boyfriend states that she is constantly seeking his attention, and “throws a fit” in public if she feels he is not. In addition, she dresses like a “high school cheerleader” and flirts with all of his friends and even his own brother. Most recently she accompanied him to a work dinner and started talking openly about their sex life, which caused him a great deal of embarrassment. When asked about these allegations, the patient started crying profusely and stated that her boyfriend hates her and is ashamed of her. In effort to calm her down, the therapist patted her arm, after which she began flirting with him. Which of the following is the most likely diagnosis?

A Borderline personality disorder
B Dependent personality disorder
C Histrionic personality disorder
D Schizoid personality disorder
E Schizotypal personality disorder
A

C Histrionic personality disorder

Histrionic personality disorder is characterized by a pervasive pattern of excessive emotionality and attention seeking and includes features such as inappropriate sexual behaviors, shallow or exaggerated expression of emotion, and consistent use of physical appearance to draw attention to self.

How well did you know this?
1
Not at all
2
3
4
5
Perfectly
185
Q

A 25-year-old woman comes to the psychiatrist because of a referral by her family physician. She has been treated for a myriad of oral and genital sexually transmitted infections over the past five years, and she says she has had several hundred sexual partners in her lifetime. When the psychiatrist enters the room, she immediately jumps out of her chair and gives him a hug, telling him she is excited to see him. She is dressed provocatively, and during the interview, she strokes the psychiatrist’s shoulder and winks at him. She tells him that she frequently goes out alone to the local bars, dances on the tables, and often performs sexual favors for male customers in the restrooms, suggesting that she would do the same for him. Which of the following is the most likely diagnosis based on the patient’s presentation and medical history?

A Antisocial personality disorder
B Avoidant personality disorder
C Borderline personality disorder
D Histrionic personality disorder
E Narcissistic personality disorder
A

D Histrionic personality disorder

Histrionic personality disorder is characterized by attention-seeking behavior, excessive emotionality and excitability, attention seeking, sexual provocation, and excessive concern with appearance.

How well did you know this?
1
Not at all
2
3
4
5
Perfectly
186
Q

A 12-year-old comes to the hospital because of a repetitive pattern of aggressive behavior. He has had episodes of peer-fights, theft, and vandalization without remorse for the past year. He was caught torturing his neighbor’s cat. Compared to his peers, which of the following is he most likely to develop as he gets older?

A Oppositional defiant disorder
B Delusional disorder
C Conduct disorder
D Antisocial personality disorder
E Anxiety disorder
A

D Antisocial personality disorder

AS HE GETS OLDER!
patient has conduct disorder right now (12 = can’t have antisocial yet) - but can develop antisocial in the future

How well did you know this?
1
Not at all
2
3
4
5
Perfectly
187
Q

A 30-year-old woman comes to the emergency department because of an attempt of suicide. She took an entire bottle of acetaminophen after breaking up with her boyfriend. She is tearful and shouts that she has nothing to live for since she lost the love of her life. She was in the relationship for three weeks. She is convinced that he left her for her best friend. One year ago, she was seen by a psychiatrist for episodes of self-mutilation due to feelings of being lonely and having no self-worth. Over the last five years, she has been accumulating debt, and had recently needed to sell her home and move in with her sister. Despite this debt, she continues to buy new clothes, and goes out to the bar most nights of the week. Which of the following is the most likely explanation for her behavior?

A Dependent personality disorder     
B Narcissistic personality disorder     
C Antisocial personality disorder     
D Histrionic personality disorder     
E Borderline personality disorder
A

E Borderline personality disorder

Patients with borderline personality disorder have unstable moods and relationships. These patients tend to be impulsive and it is common for them to have a history of multiple suicide threats or attempts.

How well did you know this?
1
Not at all
2
3
4
5
Perfectly
188
Q

A 25-year-old medical student comes to the psychiatric clinic because of difficulties working in groups over the past year. While in the waiting room, he demands to be seen by the chair of psychiatry himself because he has “a reputation to uphold.” During the visit, he explains that he refuses to cooperate on any group projects with other students because he insists that he can do the work better himself. He further notes that he wants to specialize in experimental neurosurgery and can’t have other students dragging down his grades. He reports that he often enters professors offices or the faculty lounge uninvited to share personal anecdotes and “fascinating” stories from his research. He also states that he has no issues with exploiting his classmates for his own gain because medical school is “inherently competitive.” Which of the following disorders does this patient most likely have?

A Narcissistic personality disorder
B Borderline personality disorder
C Paranoid personality disorder
D Histrionic personality disorder
E Antisocial personality disorder
A

A Narcissistic personality disorder

Narcissistic personality disorder is a cluster B disorder. These patients have an inflated sense of self, sense of entitlement, and lack of regard for others.

How well did you know this?
1
Not at all
2
3
4
5
Perfectly
189
Q

A 23-year-old woman comes to the clinic because she has frustrations about not maintaining long-term relationships with men. She says that they are always amazing in the beginning, but in the end they all are jerks. She mentions her use of multiple drugs, and her many sexual encounters, sometimes with people she does not even know. Physical examination shows scars on her wrists. What defense mechanism is she most likely to use in the clinic?

A Displacement
B Reaction formation
C Acting out
D Splitting
E Projection
A

= Borderline Personality Disorder

D Splitting

190
Q

A 45-year-old woman comes to the office because of chronic headaches. As soon as the physician enters the room, the patient starts angrily describing how the nurse “did not care at all.” She says that all nurses are cold, and just want money. Then she calms down and says she is excited the doctor is there because all doctors really care about the patients. Which of the following is most likely the defense mechanisms described here?

A Splitting     
B Projection    
C Dissociation     
D Identification     
E Displacement
A

A) Splitting

191
Q

An 8-year-old girl comes to the emergency department because she is a victim of physical abuse. After the case was notified to the authorities by one of their neighbors, she was removed from home and taken into the custody of child protection services. Which of the following disorders is this patient most at risk for in adulthood?

A Asperger syndrome
B Attention deficit hyperactivity disorder
C Borderline personality disorder
D Childhood disintegrative disorder
E Delusional disorder
A

C Borderline personality disorder

Borderline personality disorder is a pattern of abnormal behavior with extreme fear of abandonment, and unstable relationships with other people. This pattern of behavior typically begins in early adulthood. Victims of child abuse are at risk of developing borderline personality disorder.

192
Q

An adult patient who uses inappropriate sexually seductive behavior in day to day interactions and demonstrates a pervasive pattern of excessive emotionality, and attention-seeking behavior in which he or she is uncomfortable in situations where they are not the center of attention best describes which of the following personality disorders?

a. Dependent
b. Antisocial
c. Schizotypal
d. Histrionic

A

d. Histrionic

193
Q

While interviewing a patient you note that he has an irritating sense of self-importance and an over-inflated opinion of himself. He is quite boastful and alludes to his many talents. He has difficulty in focusing attention on anything other than himself and seems to have an abnormal need for approval and admiration. These characteristics are most compatible with which personality disorder?

a. Histrionic
b. Antisocial
c. Borderline
d. Narcissistic

A

d. Narcissistic

194
Q

A 32-year-old man is accused of murdering four people. He has a history of impulsivity and refusal to conform to social norms. Psychiatric examination reveals an intelligent man who seems to show no remorse. He denies any wrongdoing and appears self-centered and insincere. Which of the following personality disorders is most likely?

a. Dependent
b. Antisocial
c. Avoidant
d. Histrionic

A

b. Antisocial

195
Q

Which of the following may be a symptom of more pervasive disorders such as OCD?

a. Body dysmorphic disorder
b. Illness anxiety disorder (hypochondriasis)
c. Histrionic personality disorder
d. Conversion disorder
e. Somatization disorder

A

a. Body dysmorphic disorder

196
Q

A 45-year-old retail clerk comes to the clinic for an evaluation of depression. She claims that since high school, she has never felt happy because she considered herself to be an “outcast.” Her constantly depressed mood is accompanied by fatigue, lack of interest, difficulty concentrating, and pessimism. She is convinced that all of her high school classmates disliked her, and she has a hard time finding friends. Her only “good” time is when she is at home alone, listening to music, watching her favorite show on TV, or playing games on the internet. During the past year, she has tried several therapies to help her get over her shyness and depressed moods. She denies agoraphobia, public anxiety, or panic attacks. Which of the following is the most likely diagnosis?

A Adjustment disorder 
B Avoidant personality disorder
C Schizoid personality disorder
D Schizotypal personality disorder
E Social phobia
A

B Avoidant personality disorder

Avoidant personality disorder is defined by staying away from social situations, though these patients do desire relationships.. This is due to fear of rejection and hypersensitivity to criticism.

197
Q

An 18-year-old woman who is a college student, presents to the outpatient psychiatric clinic with multiple scabs on her arms. She has a history of attention deficit hyperactivity disorder (ADHD). She states that for the two months, she has been having thoughts of driving herself and her brother off a cliff. She states that her brother, who is 10 years older than her, is alive and well. However, she can’t get these images of her brother out of her head, and reports that it is causing her mental distress that she relieves by scratching and picking at her arms. Which of the following is the most likely diagnosis?

A Anti-social personality disorder
B Generalized anxiety disorder
C Major depressive disorder
D Obsessive-compulsive disorder
E Schizophrenia
A

D Obsessive-compulsive disorder

Obsessive-compulsive disorder is an ego dystonic disorder that is characterized by recurrent obsessions and subsequently alleviate their anxiety with compulsions.

198
Q

A 40-year-old woman comes to the psychiatric outpatient clinic because of decreased mood since the breakup with her boyfriend 6 months ago. She reports that she has been dating him for about 5 years but he left because she was “too clingy” and got tired of her frequent calls. She states that she has lived at home since dropping out of college because she does not like to be alone. She says that she has been calling her mother at work frequently since the breakup because she feels helpless, depressed and lonely. Which of the following is the most likely diagnosis?

A Borderline personality disorder
B Dependent personality disorder
C Histrionic personality disorder
D Narcissist personality disorder
E Paranoid personality disorder
A

B Dependent personality disorder

Dependent personality disorder is a personality disorder that is characterized by a pervasive psychological dependence on other people. These patients depend on others to meet their emotional and physical needs.

199
Q

A 34-year-old man comes to the office for a follow up appointment. He tells you that he is doing well, except that he seems to be spending most of his time organizing various things around his house. He states that in the last week, he has organized his bookshelf, his kitchen, and has color-coded his wardrobe. He enjoys doing these tasks but states that there seems to be no end to his organizing. He states he has always been this way, and that when his possessions are not organized, he gets very frustrated. His symptoms have affected his ability to maintain relationships with his family and friends. Which of the following is the most likely diagnosis?

A Generalized anxiety disorder
B Normal behavior
C Obsessive compulsive disorder
D Obsessive Compulsive Personality Disorder
E Depression
A

D Obsessive Compulsive Personality Disorder

Obsessive compulsive disorder is characterized by obsessive thoughts that provoke anxiety, which is relieved by performing a compulsion. In contrast, patients with obsessive compulsive personality disorder are in harmony with their personality, and enjoy following their own rules for order.

200
Q

A 32-year-old man is brought to the psychiatrists office by his girlfriend because of trouble getting to work on time for a year. Before he goes to work in the morning, he repeatedly returns to the bedroom to make sure the lights are turned off. He also must lock the door four times before he feels comfortable leaving for work. You ask the patient about these behaviors and he says that he wishes he could stop them, but he cannot because he is so nervous about the lights being left on, and the house being unlocked. Which of the following medications is the most appropriate initial management?

A Bupropion
B Fluoxetine
C Lorazepam
D Amitriptyline
E Ziprasidone
A

B Fluoxetine

Obsessive compulsive disorder is treated with selective-serotonin reuptake inhibitors like fluoxetine, as well as clomipramine, which is a tricyclic antidepressant.

201
Q

A 30-year-old male presents to the clinic for treatment on the advice of his girlfriend. He reports a history of extreme shyness and has been the “silent one” since childhood. He has very few friends and works as a computer technician. He does not hang out with his coworkers outside of work since he is afraid they will not like him. Which of the following is most likely associated with the patient’s condition?

A This patient has a Cluster C personality disorder

B The patient is likely to have a first degree relative with schizophrenia

C This patient has a Cluster A personality disorder

D This patient has a personality disorder not otherwise specified (NOS)

E The patient is likely to use splitting as a defense mechanism

A

A This patient has a Cluster C personality disorder

= avoidant personality disorder

Avoidant personality disorder is a cluster C disorder where patients desire relationships with others. However, their fear of rejection causes them to avoid social settings.

202
Q

A 30-year-old is brought to the office by her husband because she has been continuously washing her hands in fear of contamination for 6 months. She currently washes her hands >30 times a day. She states that she was recently fired from her job because she took too many breaks to wash her hands. She admits to her behavior being irrational but she also admits that she cannot control or stop it. Which of the following is the most appropriate initial treatment?

A Ziprasidone
B Imipramine
C Fluoxetine
D Buproprion
E Lithium
A

= OCD

C Fluoxetine

Obsessive-compulsive disorder is characterized by the presence of obsessions, compulsions, or both. It is treated with selective-serotonin reuptake inhibitors like fluoxetine, as well as clomipramine, which is a tricyclic antidepressant.

203
Q

An 85-year-old man is brought to your office by his daughter because of an inability to “throw anything away” for 2 years. She states that their house is overrun with items and that insects have begun to make homes in them. The father replies that he can get rid of things but only if he is able to sell them or find a way to trade them for fair value. He explains that he grew up during the great depression and that his father was a banker who lost everything. The father always told the patient to invest in physical goods, as “that is the only thing that retains its value in a depression.” Which of the following is the most likely diagnosis?

A Hoarding disorder
B Major depressive disorder
C Obsessive compulsive disorder
D Schizophrenia
E Obsessive compulsive personality disorder
A

A Hoarding disorder

Hoarding disorder is characterized by persistent difficulty discarding or parting with possessions, regardless of their actual value. The affected individual perceives a need to save these items and feels distress at the notion of discarding them.

204
Q

A 46-year-old man comes to the office for a psychological evaluation after having repeated fights with his coworkers. When asking the patient about his life, he mentions that he is the oldest brother from all family members and that his father used to work in the army. Currently, the patient claims to enjoy strict workout routines and also adores making lists for all his daily activities, asserting that he doesn’t go to bed until they’re finished. When asking the patient about his current problem, he claims that his coworkers don’t ever clear their workspace, he says it’s all cluttered and he finds it very hard to work around them. In fact, he repeatedly washes his hands just by greeting them. Looking at his previous evaluations, he has an immaculate profile and an outstanding job performance. Which of the following defense mechanisms is most likely to be associated with his condition?

A Displacement
B Rationalization 
C Projection 
D Reaction Formation
E Undoing
A

E Undoing

Undoing is a common defense mechanism seen in patients with obsessive-compulsive personality disorder. This mechanism consists of performing the reverse of unacceptable behavior in an attempt to counteract it, such as repeated hand washing.

This condition is more common in first born males (i.e., oldest brother), who had a harsh discipline upbringing (i.e., his father working in the army). An ordinary defense mechanism seen in these patients is undoing, which consists of performing the reverse of unacceptable behavior in a counteraction attempt. A good example for this is repeated hand washing. Hand washing is an undoing mechanism that helps them fix their impulse once they’ve done the unacceptable behavior, such as, shaking hands or opening door knobs.

205
Q

A 30-year-old man was recently diagnosed with OCD. He has a history of rash and difficulty breathing on 2 different SSRIs in the past. Which of the following would be a reasonable alternative to trial?

a. Bupropion (Wellbutrin)
b. Venlafaxine (Effexor)
c. Clomipramine (Anafranil)
d. Sertraline (Zoloft)

A

c. Clomipramine (Anafranil)

= TCA = 2nd line after SSRIs

Venlafaxine (Effexor) = 3rd line

206
Q

Which of the following may be a symptom of more pervasive disorders such as OCD?

a. Body dysmorphic disorder
b. Illness anxiety disorder
c. Histrionic personality disorder
d. Conversion disorder
e. Somatization disorder

A

a. Body dysmorphic disorder

207
Q

Conversion disorder is best described as:

a. Reports of multiple physical symptoms without an identifiable medical basis
b. Preoccupation with the possibility that normal sensations are symptoms of serious disease
c. Possibly a symptom of more pervasive disorders such as OCD
d. One or more symptoms or deficits affecting motor or sensory functions that suggest a neurological or other general medical condition

A

d. One or more symptoms or deficits affecting motor or sensory functions that suggest a neurological or other general medical condition

208
Q

Which of the following is most unique to conversion disorders?

a. Symptoms are gradual in onset
b. Not related to past emotional trauma or stressors
c. Easily diagnosed
d. The symptom or deficit is due to malingering
e. “La belle indifference”

A

e. “La belle indifference”

209
Q

A patient at a Christmas party on a cruise ship develops a terrible headache. His blood pressure is 224/158 mm Hg. Which of the following agents is this patient most likely taking regularly?

tricyclic antidepressants
barbiturates
benzodiazepines
monoamine oxidase inhibitors
phenothiazines
A

monoamine oxidase inhibitors

This is a typical scenario for MAOI induced malignant hypertension from eating the “wrong” foods, which are frequently found on a Christmas buffet, (cheese, sausages, red wine, etc).

210
Q

A 50-year-old woman who has been treated with sertraline for major depressive disorder for
more than two years comes to the office because she has had weakness, cold intolerance,
constipation, and weight gain during the past six months. Physical examination shows dry,
coarse skin as well as bradycardia, hypothermia, and swelling of the hands and feet. Which of
the following laboratory studies is the most appropriate to determine the diagnosis?

(A) Liver function testing
(B) Measurement of serum electrolyte levels
(C) Measurement of serum estrogen level
(D) Measurement of serum sertraline level
(E) Measurement of serum thyroid-stimulating hormone level

A

(E) Measurement of serum thyroid-stimulating hormone level

This question tests the examinee’s ability to determine the laboratory study that is most likely to
specify the diagnosis. The correct answer is Option (E), measurement of serum thyroid
stimulating hormone level. Hypothyroidism is suspected on the basis of the patient’s symptoms
of depression, weakness, constipation, and weight gain as well as the physical findings of
bradycardia, hypothermia, swelling of the hands and feet, and dry, coarse skin. Measurement of
serum thyroid-stimulating hormone level is the study that will either confirm or refute this
suspected diagnosis

211
Q

A 35-year-old man with bipolar disorder, most recent episode mixed, comes to the clinic for
routine follow-up examination. His condition has been difficult to control and has required
treatment with multiple medications during the past two years. The patient says his mood has
been stable with his current regimen, but for the past three months, he has had tremor
primarily affecting his hands. Which of the following medications is the most likely cause of this
patient’s tremor?

(A) Carbamazepine 
(B) Gabapentin 
(C) Lamotrigine 
(D) Lithium carbonate 
(E) Topiramate
A

(D) Lithium carbonate

The correct answer is Option (D), lithium carbonate - approved by the Food and Drug Administration for management of bipolar I disorder and is the most commonly used pharmacologic therapy for that condition. Tremor, as noted in the patient’s history, is a common adverse effect of lithium carbonate. Such tremor is seen in the fingers of the outstretched hands, usually at a frequency of 8 to 10 Hz, and is sometimes related to peak serum lithium levels. Dividing the dose of lithium carbonate and decreasing caffeine intake can decrease the tremor. In the case of a severe coarse tremor, lithium carbonate toxicity (serum lithium level greater than 2.5 mEq/L) must be ruled out

212
Q

A 19-year-old woman who is a college student is brought to the clinic by her roommate because
she has been acting strangely during the past six months. During the past month, the patient has
been describing how another person’s thoughts have been entering into her mind. The patient’s
grades have been slipping, and she does not talk as much as she did previously. The roommate
says that when the patient does talk, she strays from the topic and is hard to follow. During the
interview, the patient says a television reporter told her that the government had a special
message for her and she should listen to the radio for further instructions. Which of the
following conditions is the most likely cause of this patient’s symptoms?

(A) Acute stress disorder 
(B) Delusional disorder 
(C) Schizoaffective disorder 
(D) Schizophrenia 
(E) Schizophreniform disorder
A

(D) Schizophrenia

The correct answer is Option (D), schizophrenia. The patient has had classic symptoms
of schizophrenia for at least six months, including hallucinations (voices speaking to her), social
dysfunction affecting grades and friendships, and disorganized speech. Although the patient is
somewhat young for schizophrenia (peak incidence in women is between 25 and 35 years), her
symptoms meet the DSM criteria for the condition.

Option (C), schizoaffective disorder, is plausible but can be ruled out based on the fact that there is no
evidence of concurrent manic or depressive symptoms during the schizophrenic episode
described.

Option (E), schizophreniform disorder, is also plausible but can be ruled out based on
the six-month duration of symptoms. To meet criteria for schizophreniform disorder, symptoms
must persist for at least one month but fewer than six months

213
Q

A 37-year-old man comes to the office after he experienced what he says was a nervous
breakdown. The patient says that after he recently declared bankruptcy, losing his home and his
business, he became very depressed. During this time, he began to hear voices telling him that
he was useless and should kill himself. The patient says his symptoms stopped after approximately one week. He has had no similar episodes. Medical history includes no psychiatric conditions. Physical examination shows no abnormalities, and results of laboratory studies are within normal limits. Which of the following is the most likely diagnosis?

(A) Borderline personality disorder 
(B) Brief psychotic disorder 
(C) Major depressive disorder with psychotic features 
(D) Schizophrenia 
(E) Schizophreniform disorder
A

(B) Brief psychotic disorder

This question tests the examinee’s ability to distinguish between disorders that may present with
psychotic symptoms. The correct answer is Option (B), brief psychotic disorder. Essential features
of brief psychotic disorder include sudden onset of one or more of the following symptoms:
delusions, hallucinations, disorganized speech, disorganized behavior, and catatonia. Brief
psychotic disorder is also characterized by duration of symptoms for at least one day but less
than one month followed by return to premorbid level of functioning. The disorder is not
associated with a medical condition or the effect of illicit substances such as hallucinogens.

Option (A), borderline personality disorder, is incorrect because although it may present with
stress-related paranoid ideation, this condition is transient. Option (C), major depressive disorder
with psychotic features, is incorrect because a diagnosis of major depressive disorder requires
the presence of symptoms meeting diagnostic criteria for most of the day, nearly every day, and
for at least two weeks. Option (D), schizophrenia (6 months+), and Option (E), schizophreniform disorder (> 1 month, < 6 months), are also incorrect because the short duration of the patient’s symptoms (one week) does not meet criteria for these conditions.

214
Q

A 28-year-old man comes to the office because he has had severe restlessness, anxiety, and
generalized myalgia during the past three days. The patient says he has not slept for the past
two days. Physical examination shows dilation of the pupils, excessive lacrimation, diaphoresis,
and piloerection. The most likely cause of this patient’s condition is withdrawal of which of the
following substances?

(A) Cocaine 
(B) Inhalants 
(C) Methamphetamines 
(D) Modafinil 
(E) Opioids
A

(E) Opioids

The correct answer is Option (E), opioids.
Insomnia, dilation of the pupils, piloerection, diaphoresis, lacrimation or rhinorrhea, and myalgia
are classic symptoms of opioid withdrawal.

Option (A), cocaine, is incorrect because symptoms of cocaine withdrawal include fatigue, vivid
dreams, increased appetite, and psychomotor retardation or agitation. Option (B), inhalants, is
incorrect because withdrawal syndrome from inhalants is characterized by disturbed sleep,
irritability, hallucinations, and delusions, nausea and vomiting, diaphoresis, and tachycardia.
Option (C), methamphetamines, is incorrect because withdrawal from methamphetamines is
characterized by any two of the following: lethargy, fatigue, psychomotor retardation or
agitation, craving for stimulants, increased appetite, insomnia or hypersomnia, and bizarre or
unpleasant dreams. Option (D), modafinil, is incorrect because withdrawal of this medication,
which is used to treat narcolepsy, would cause sleepiness (not insomnia).

215
Q

A 48-year-old man comes to the office for annual physical examination. The patient has smoked
one pack of cigarettes daily for the past 20 years and says he wants to quit. He has tried to quit
several times, but every time he does, he has depressed mood, difficulty sleeping, anxiety,
restlessness, and increased appetite. The patient says these symptoms are so severe that he is
unable to function well at work. Which of the following is the most appropriate therapy to aid in
smoking cessation for this patient?

(A) Alprazolam  
(B) Diazepam  
(C) Nicotine transdermal patches  
(D) Nicotinic acid 
(E) Varenicline
A

(C) Nicotine transdermal patches

The correct answer is Option (C), nicotine transdermal patches. Nicotine transdermal patches
used as an aid in smoking cessation are not likely to exacerbate a patient’s symptoms of
depression, anxiety, or sleep disturbance

Option (A), alprazolam, and Option (B), diazepam, are incorrect because although they may be
plausible therapies for anxiety, these medications are not approved for smoking cessation.
Option (D), nicotinic acid, is incorrect because this medication, which is used in conjunction with
low-fat diet to manage hypercholesterolemia, is neither approved nor effective for smoking
cessation. Option (E), varenicline, is a plausible choice because it is approved for smoking
cessation. However, therapy with varenicline is not advisable in patients who have psychiatric
disorders because it has the potential to exacerbate agitation and depression

216
Q

A 55-year-old woman comes to the office because she has a 20-year history of repetitive actions
and is tired of wasting time by repeating these actions. The patient says she repeatedly checks
the doors in her house to make sure they are locked, washes her hands several times per hour,
and checks her electrical appliances three times each morning before she leaves for work.
Medical history includes second-degree atrioventricular block. Which of the following
medications is the most appropriate initial therapy for this patient’s psychiatric disorder?

(A) Clomipramine 
(B) Clonazepam 
(C) Methylphenidate  
(D) Quetiapine  
(E) Sertraline
A

(E) Sertraline

This question tests the examinee’s ability to recognize the signs and symptoms of obsessive
compulsive disorder and to choose the most appropriate therapy. The correct answer is Option
(E), sertraline. Sertraline is a selective serotonin reuptake inhibitor (SSRI), and all of the SSRIs
available in the US have been approved by the Food and Drug Administration for management of
obsessive-compulsive disorder. Although SSRIs may cause transient problems with sleep,
gastrointestinal disturbances, headache, restlessness, and anxiety, these adverse effects are less
problematic than those associated with other medications, such as clomipramine.

217
Q

A 20-year-old woman who is a college student comes to the health services center because she
has had symptoms of depression for the past three months, since she was a victim of date rape
at a party. The patient says she was heavily intoxicated when the incident occurred and has little
memory of the event, but she was embarrassed and ashamed when she awoke at the scene and
realized what had happened. She did not seek medical care at that time. The patient says she
has not told her friends about the incident, and she has continued to attend classes and work
part time. However, she says she constantly feels sad and anxious, has become tearful and
withdrawn, and has had difficulty sleeping because of frightening nightmares. This patient most
likely has which of the following psychiatric conditions?

(A) Acute stress disorder 
(B) Adjustment disorder, unspecified 
(C) Generalized anxiety disorder 
(D) Major depressive disorder 
(E) Post-traumatic stress disorder
A

(E) Post-traumatic stress disorder

This question tests the examinee’s ability to discriminate between behavior disorders based on
the salient points in the patient’s history. The correct answer is Option (E), post-traumatic stress
disorder. The patient described meets the criteria for post-traumatic stress disorder, which
include the following: involvement in a traumatic event, duration of symptoms for more than one month, and significant effect on daily life due to depression, anxiety, nightmares, and social
withdrawal.

Option (D), major depressive disorder, is incorrect because it is
characterized by symptoms not exhibited by the patient described. Characteristic symptoms of
major depressive disorder include, but are not limited to, the following: depression, anhedonia,
significant weight loss without dieting, weight gain (more than 5% of body weight within one
month), insomnia or hypersomnia, psychomotor agitation or retardation, fatigue, loss of energy,
recurrent thoughts of death, and suicidal ideation

218
Q

A 42-year-old man is referred to the office for mental health evaluation. The patient says he has
a long-standing fear of criticism and rejection. He also has feelings of inadequacy and refuses to
try new activities because of his fear of embarrassment. The patient has held the same entry
level position at his workplace for the past 14 years despite several opportunities for promotion.
The patient is unmarried and has only had one intimate relationship, which lasted only a few
weeks. He has few friends other than those in his model airplane club. Which of the following
personality disorders is the most likely diagnosis?

(A) Avoidant 
(B) Dependent 
(C) Paranoid 
(D) Schizoid 
(E) Schizotypal
A

(A) Avoidant

The correct answer is Option (A), avoidant. Avoidant
personality disorder is characterized by fears of rejection, disapproval, and/or criticism;
unwillingness to be involved with people unless assured of being liked; restraint in intimate
relationships; preoccupation with social criticism or rejection; inhibition in new situations
because of feelings of inadequacy; and unwillingness to engage in new activities because of
possibility of embarrassment.

219
Q

A 23-year-old man comes to the office because he has pain in his right hand. The nurse says the
patient was irritable and reluctant to answer many intake questions, and he refused to put on an examination gown. On questioning, the patient is initially charming and glib, but after he is
assured that the information he shares will not be relayed to his probation officer, he admits
that he was involved in a fistfight. He says proudly that “the other guy is in a lot worse shape.”
This patient most likely has which of the following psychiatric conditions?

(A) Antisocial personality disorder 
(B) Bipolar disorder, most recent episode hypomanic  
(C) Conduct disorder 
(D) Oppositional defiant disorder 
(E) Paranoid personality disorder
A

(A) Antisocial personality disorder

220
Q

A 46-year-old man is brought to the emergency department by ambulance because he has been
having visual hallucinations for the past four hours. The patient’s wife says he has had diarrhea,
agitation, and confusion for the past two days. Two weeks ago, he was fired from his job
because of excessive absences. He has been unable to find a new job and has been asking his
friends for money for several days. The patient has a 25-year history of drinking one pint of
whiskey daily. Temperature is 38.5°C (101.3°F), pulse rate is 126/min, and blood pressure is
162/102 mmHg. Which of the following is the most appropriate initial management of this
patient’s condition?

(A) Admission to the hospital for monitoring and social services evaluation
(B) Discharge and referral to an outpatient alcohol treatment center
(C) Intramuscular injection of haloperidol and discharge
(D) Intravenous infusion of ethyl alcohol and discharge
(E) Oral administration of chlordiazepoxide and admission to the hospital

A

(E) Oral administration of chlordiazepoxide and admission to the hospital

This question tests the examinee’s ability to determine the appropriate initial management
based on the salient findings in the history and physical examination. The correct answer is
Option (E), oral administration of chlordiazepoxide and admission to the hospital. The history
and physical examination findings in this patient represent delirium tremens due to chronic
alcohol use. Therefore, oral administration of chlordiazepoxide and admission to the hospital is
the most appropriate initial management because of the potential for seizure activity

221
Q

A 35-year-old woman is brought to the emergency department by ambulance after police were
called to her home by a neighbor, who witnessed rape and robbery by two intruders. The
patient is fully alert and oriented to person, place, and time. She cannot remember anything
about the incident. Which of the following is the most likely cause of this finding?

(A) Depersonalization/derealization disorder (depersonalization disorder) 
(B) Dissociative amnesia 
(C) Fugue state 
(D) Traumatic brain injury 
(E) Volitional memory loss
A

(B) Dissociative amnesia

Dissociative amnesia is characterized by inability to recall important information, usually of a traumatic or stressful nature, that is too extensive to be considered ordinary forgetfulness.

Depersonalization/derealization disorder (depersonalization disorder) is characterized by feelings of detachment as though one is in a dream, and fugue state involves confusion regarding
personal identity. Option (D), traumatic brain injury, is incorrect because the patient has not
sustained a head injury. Option (E), volitional memory loss, is incorrect because the patient’s
memory loss is due to her ability to suppress or repress selective memories. In addition, volitional
memory loss is not a DSM diagnosis.

222
Q

A 26-year-old man with a history of involuntary eye blinking and snorting comes to the office
because he is distressed about increasing frequency of these tics over the past six months. The
patient says the episodes currently occur numerous times during the day. Medical history
includes development of eye blinking at 12 years of age and development of snorting at 16 years
of age. Psychotherapy and exercises initially reduced the frequency of these episodes. Because
the patient is embarrassed by his tics, his performance at work has decreased and he now
declines most invitations to social gatherings. Which of the following medications is the most
appropriate therapy for this patient?

(A) Carbamazepine 
(B) Fluvoxamine 
(C) Methylphenidate 
(D) Risperidone 
(E) Venlafaxine
A

This question tests the examinee’s ability to recognize the signs and symptoms of Tourette
disorder and to choose the most appropriate therapy. The correct answer is Option (D),
risperidone. A tic is a rapid, recurrent, nonrhythmic, stereotyped movement of vocalization.

Tourette disorder is characterized by vocal or motor tics, both of which may be present during
the course of the illness. Onset of tics occurs before 18 years of age, and symptoms persist for at
least one year, occurring many times nearly every day. The tics cause marked distress and
impairment in occupational and social functioning. Compelling evidence exists that the
dopamine system is involved in tic disorders — pharmacologic agents that antagonize dopamine
suppress tics, and agents that increase dopamine, such as methylphenidate, exacerbate tics.
Risperidone is a dopamine antagonist and is, therefore, the most appropriate therapy for
Tourette disorder.

TREATMENT FOR TOURETTE’S = ANTIPSYCHOTIC

Option (A), carbamazepine, is incorrect because it is indicated for neuralgias and seizures, not for
Tourette disorder, and it does not affect dopamine transmission. Option (B), fluvoxamine, is
incorrect because it is a selective serotonin reuptake inhibitor, which increases dopamine levels.
Option (C), methylphenidate, is incorrect because it is a stimulant, and stimulants have been
reported to exacerbate preexisting tics. Option (E), venlafaxine, is incorrect because it is a
serotonin-norepinephrine reuptake inhibitor and does not decrease dopamine transmission.

223
Q

A 19-year-old college student comes to the health clinic for evaluation because she has been
purging food during the past semester. The patient appears well developed and well nourished.
Physical examination shows mild enlargement of the parotid glands. In addition to complete
blood cell count and urinalysis, measurement of which of the following levels is most
appropriate?

(A) Fasting blood glucose 
(B) Serum amylase
(C) Serum electrolytes 
(D) Serum liver enzymes 
(E) Serum thyroid-stimulating hormone
A

(C) Serum electrolytes

Patients who purge are at increased risk for hypokalemia and/or hypochloremic
alkalosis because of vomiting. The most appropriate laboratory study to test for these conditions
is measurement of serum electrolyte levels

224
Q

A 5-year-old boy is brought to the clinic by his mother because she is worried about changes in
his behavior during the past few months. The patient’s mother says he has had episodes that
occur sporadically at night during which he sits up in bed screaming, crying incoherently, and
intensely frightened. She says it is initially difficult to calm him down, and he is usually sweating,
breathing rapidly, and tremulous. The patient eventually calms down after 10 to 15 minutes and then falls asleep. Which of the following questions is most appropriate to confirm the suspected
diagnosis of sleep terror disorder in this patient?

(A) Do his arms and legs jerk during sleep? 
(B) Does he have nightmares? 
(C) Does he to go to sleep late? 
(D) Does he watch violent movies? 
(E) Does he wet the bed?
A

(B) Does he have nightmares?

Sleep terror disorder is characterized by arousal with screams or crying accompanied by fear and signs of autonomic arousal, such as tachycardia, sweating, and rapid breathing.

Patients with sleep terror disorder have no recollection of a dream or the episode, which causes distress or impairment in social or occupational functioning.

225
Q

A 19-year-old college student comes to the health clinic for evaluation because she has been
purging food during the past semester. The patient appears well developed and well nourished.
Physical examination shows mild enlargement of the parotid glands. In addition to complete
blood cell count and urinalysis, measurement of which of the following levels is most
appropriate?

(A) Fasting blood glucose 
(B) Serum amylase
(C) Serum electrolytes 
(D) Serum liver enzymes 
(E) Serum thyroid-stimulating hormone
A

(C) Serum electrolytes

Patients who purge are at increased risk for hypokalemia and/or hypochloremic
alkalosis because of vomiting. The most appropriate laboratory study to test for these conditions
is measurement of serum electrolyte levels

226
Q

A 5-year-old boy is brought to the clinic by his mother because she is worried about changes in
his behavior during the past few months. The patient’s mother says he has had episodes that
occur sporadically at night during which he sits up in bed screaming, crying incoherently, and
intensely frightened. She says it is initially difficult to calm him down, and he is usually sweating,
breathing rapidly, and tremulous. The patient eventually calms down after 10 to 15 minutes and then falls asleep. Which of the following questions is most appropriate to confirm the suspected
diagnosis of sleep terror disorder in this patient?

(A) Do his arms and legs jerk during sleep? 
(B) Does he have nightmares? 
(C) Does he to go to sleep late? 
(D) Does he watch violent movies? 
(E) Does he wet the bed?
A

(B) Does he have nightmares?

Sleep terror disorder is characterized by arousal with screams or crying accompanied by fear and signs of autonomic arousal, such as tachycardia, sweating, and rapid breathing.

Patients with sleep terror disorder have no recollection of a dream or the episode, which causes distress or impairment in social or occupational functioning.

227
Q

An 8-year-old girl is brought to the pediatric clinic by her parents because of a 6 month history of hair-pulling which has left a bald spot on her head. Her parents deny that she is eating her hair. They state that she appears to be pulling on her hair compulsively. Which of the following is the most appropriate first-step for treatment?

A Phenelzine
B Clozapine
C Clomipramine
D Fluvoxamine
E Cognitive behavioral therapy
A

E Cognitive behavioral therapy

Trichotillomania is an impulsive disorder in which patients experience anxiety which is relieved by the pulling of their own hair. First line therapy of choice is cognitive behavioral therapy.

228
Q

Which disorder type does autism spectrum disorder belong in?

a. Conduct
b. Neurodevelopmental
c. Dissociative
d. Somatoform

A

b. Neurodevelopmental

229
Q

A 6-year-old boy comes to the office with his mother because of behavioral problems for 6 months. He has been sneaking into his classroom during recess and stealing food and money from his classmates. He has also been dismissed from physical education class permanently because he regularly becomes too aggressive during sports. Examination shows he is friendly and polite; however, his pockets are filled with tongue depressors. Which of the following is the most likely diagnosis?

A. Antisocial personality disorder
B. ADHD
C. Autism spectrum disorder
D. Conduct Disorder
E. Oppositional defiant disorder
A

D. Conduct Disorder

Conduct disorder is characterized by physical aggression and kleptomania in a child. These behaviors are often referred to as “antisocial behaviours.” It is often seen as the precursor to antisocial personality disorder, which is not diagnosed until the individual is 18 years old

230
Q

A 15-year-old boy is referred to a child psychologist because of worsening behavior and constant disruption in class. He has received multiple reprimands in the past 6 months for not doing the homework that his teacher assigned, and refuses to listen to the classroom instruction. He was previously well behaved and one of the top students in his class. He denies any recent major life events or changes at home. Which of the following is the most likely diagnosis?

A Antisocial personality disorder     
B Attention deficit hyperactivity disorder     
C Conduct disorder     
D Major depressive disorder      
E Oppositional defiant disorder
A

E Oppositional defiant disorder

Oppositional defiant disorder is a pattern of angry/irritable mood, argumentative/defiant behavior or vindictiveness lasting at ≥6 months and not due to substance abuse or a psychotic disorder.

ODD is often co-morbid with ADHD, and can sometimes lead to the development of conduct disorder. ADHD must first be ruled out in children with ODD, and treatment consists of therapy for the patient and parent management training

231
Q

A 13-year-old boy has been suspended 5 times this year for arguing with teachers. He has presented a pattern of negativism and hostility that has lasted for about 8 months. When asked about the suspensions, he admits that he loses his temper easily and often blames the principal for not being fair to him. He usually finds an argument before finishing his homework. At home, he goes out of his way to annoy his siblings. He gets furious if his legal guardian finds out about it and confiscates his smartphone. Which of the following is a characteristic behavior of this disorder?

A Violating the rights of others
B Hostile and disobedient behavior towards authority
C Destruction of property and theft
D Physical aggression
E Killing and/or harming small animals
A

OPPOSITIONAL DEFIANT DISORDER
= B) Hostile and disobedient behavior towards authority

a / c / d / e = conduct disorder

Oppositional defiant disorder is characterized by a pattern of angry/irritable mood, argumentative/defiant behavior or vindictiveness lasting at least 6 months. Presentation includes recurrent loss of temper, arguing, refusing to comply with rules, deliberately annoying others, blaming others for behavior, and resentfulness

232
Q

A mother brings in her 10-year-old son because she is worried about his behavior. His teacher called home with complaints about his behavior toward her and other teachers in the school. The boy will yell out during teaching sessions and refuse to participate in class activities. He seems to get along with his classmates and will participate in recess with his friends. The mother also notes that her son tends to have behavioral issues at home. Many times he will act out when asked to do his chores and frequently gets in arguments with his mom. What is the most likely diagnosis?

A Antisocial personality disorder  
B Attention deficit hyperactivity disorder        
C Conduct disorder     
D Oppositional defiant disorder  
E Schizoid personality disorder
A

D Oppositional defiant disorder

not antisocial (have to be 18+) = Antisocial personality disorder is a cluster B personality disorder. Patients with antisocial personality disorder show a disregard for others and typically have violations with the law. begins in childhood as conduct disorder

not conduct = Individuals with a conduct disorder have numerous antisocial activities such as lying, stealing, running way, physical violence, sexually coercive behaviors, and destruction of property. Children and adolescence with conduct disorder will have defiant behavior toward everyone. In oppositional defiant disorder the poor behavior is directed toward authority figures.

not ADHD = not having problems listening, difficulty paying attention, easily distracted, forgetful, talking excessively and/or fidgeting

not schizoid = Schizoid personality disorder is a cluster A personality disorder. Patients will be detached from others and have little desire for close relationships. People with this disorder prefer to be alone and take pleasure in few activities. This would not be diagnosed until adulthood, similarly to antisocial personality disorder.

233
Q

A 20-year-old Caucasian man comes to the office because of abnormal social behavior for the past year. He says he has been forced to come for a court-mandated psychiatric evaluation after assaulting his neighbor with a knife. He says his neighbor deserved it and the patient shows no remorse for the incident. He has a long history of violence and has many criminal convictions related to drugs, theft, and sexual assault. Which of the following is the most likely diagnosis?

A Antisocial personality disorder     
B Avoidant personality disorder     
C Borderline personality disorder     
D Conduct disorder    
E Schizoid personality disorder
A

A Antisocial personality disorder

Can’t be conduct - patient is > 18

Antisocial personality disorder can be diagnosed in patients >18 years old. It is characterized by aggressive behavior toward people and animals, destruction of property, lack of remorse, illegal activity, and lying.

234
Q

A 10-year-old boy comes to the office because of inappropriate behavior towards his classmates for 3 months. He was recently caught trying to touch the genital area of several female classmates, as well as trying to coerce them into the bathroom against their will. He has a history of attention-deficit hyperactivity disorder (ADHD) and motor tics which included involuntary shoulder shrugging, blinking, and head thrusting, all of which have worsened over the past year. Which of the following is his most likely diagnosis?

A Anti-social personality disorder 
B Borderline personality disorder     
C Conduct disorder     
D Oppositional defiant disorder
E Tourette syndrome
A

C Conduct disorder

Conduct disorder is a childhood disorder characterized by anti-social behavior including stealing, fighting, destroying property, and forced sexual activity. It is more common in children with attention deficit hyperactivity disorder.

has ADHD + Tourette’s = more likely to have ODD and Conduct disorder

patient is 10 = cannot be a personality disorder (not A or B)

Tourette syndrome does not account for his inappropriate aggressive sexual behavior. A diagnosis of Tourette syndrome requires the presence of tics for >1 year and must begin before the patient is 21 years old.

235
Q

A 14-year-old boy comes to the office to be evaluated for his behavior. His neighbor recently called the police because he was seen setting fire to the toolshed in the backyard. The first time he was caught setting a fire was in his old house three years ago. His mother subsequently found a collection of lighters in a cabinet in his room. She says that lately, her son has been having a difficult time coping with the loss of his father who was killed on active duty a few months ago. Her son has since kept to himself, but is planning to be a volunteer firefighter when he becomes eligible. Which of the following disorders belongs to the same diagnostic category as this patient’s most likely diagnosis?

A Antisocial personality disorder
B Conduct disorder
C Kleptomania
D Schizophreniform disorder
E Passive-aggressive personality disorder
A

C Kleptomania

Pyromania is a disorder of impulse control along with kleptomania and intermittent explosive disorder. Patients with pyromania fail to resist impulses to deliberately start fires or engage in fire-related activity. These acts induce euphoria and emotional release.

236
Q

A 10-year-old child is sent to the school psychologist because he refuses to comply with the class rules. He gets upset at the teacher regularly when he is told to complete a homework assignment in class. Sometimes he refuses to complete them altogether. Several of his teachers have reported that he intentionally creates noises in class to interrupt the class. He tells the psychologist that the teacher and his classmates are at fault. What is the most likely diagnosis?

A Antisocial personality disorder
B Attention-deficit hyperactivity disorder
C Conduct disorder
D Obsessive-compulsive disorder
E Oppositional defiant disorder
A

E Oppositional defiant disorder

Oppositional defiant disorder is characterized by a pattern of angry/irritable mood, argumentative/defiant behavior or vindictiveness lasting at least 6 months. Presentation includes recurrent loss of temper, arguing, refusing to comply with rules, deliberately annoying others, blaming others for behavior, and resentfulness.

237
Q

A 15-year-old boy comes to the office because of recently being caught by his parents beating a stray dog in an alley near his home a day ago. He has been known to vandalize the local convenience store and has run away from home twice over during the past two years. He has also been dismissed from physical education class permanently because he regularly becomes too aggressive during sports. He has a history of attention deficit hyperactivity disorder. Which of the following is the most likely comprehensive diagnosis?

A Antisocial personality disorder     
B Attention deficit hyperactivity disorder     
C Conduct disorder     
D Oppositional defiant disorder
E Schizoid personality disorder
A

C Conduct disorder

Conduct disorder involves a number of problematic behaviors. These include oppositional and defiant behaviors, and antisocial activities such as lying, stealing, running away, physical violence, and sexually coercive behavior.

238
Q

Which of the following symptoms is generally not characteristic of GAD?

A awakening with apprehension and unrealistic concern about future misfortune

B worry out of proportion to the likelihood or impact of feared events

C a 6-month or longer course of anxiety and associated symptoms

D association of the anxiety with depression

E anxiety exclusively focused on health concerns

A

E - anxiety exclusively focused on health concerns

When health concerns become the focus of worry, a diagnosis of hypochondriasis or another somatoform disorder becomes more likely.

239
Q

Which of the following pharmacologic agents is not recommended in the treatment of GAD?

A venlafaxine
B buspirone 
C benzodiazepines
D selective serotonin reuptake inhibitors (SSRIs)
E clozapine
A

E clozapine

Clozapine is an antipsychotic agent used in the treatment of patients with schizophrenia. It has no role in the treatment of GAD, and its use in the United States is restricted.

240
Q

A 57-year old woman comes to the office because of difficulty sleeping that has been ongoing for years. She says that she has always had difficulty sleeping, but due to a recent change in jobs, the lack of sleep has been affecting her more, and she feels tired throughout the day. She attributes her sleep problems to uncontrollable worry about her children, her job, the family’s financial situation, and her relationship with her husband. She denies using alcohol or any other psychotropic medications and has no chronic medical conditions. Which of the following is the most necessary before a diagnosis of generalized anxiety disorder can be made?

A An independent cause of insomnia
B Family history of generalized anxiety disorder
C History of post-traumatic stress disorder
D History of difficulty concentrating
E Recent loss of a loved one

A

D History of difficulty concentrating

Generalized anxiety disorder is defined as excessive anxiety and worry occurring most days, lasting >6 months, that interfere with daily life, are impossible to control, are not caused by other mental health conditions, and three of the following symptoms: restlessness, fatigue, trouble concentrating, irritability, muscle tension, or sleep problems.

241
Q

A 5-year-old girl comes to her pediatrician’s office for a well-child check. Her mother says that while the patient enjoys to play and talk at home, she gets uncomfortable and does not answer her teacher verbally when she gets asked a question in school. She will instead nod her head or point to the answer. While playing with her classmates, the patient is talkative. The mother says that the patient has “always been shy.” Audiometry examination shows normal hearing bilaterally. Which of the following is the most likely diagnosis?

A Hearing loss
B Pervasive developmental disorder
C Post traumatic stress disorder
D Schizoid personality disorder
E Selective mutism
A

E Selective mutism

Selective mutism is characterized by a child’s inability to speak in certain situations and people. These patients act normally in other settings and people.

242
Q

A 61-year-old man comes to the office because of uncontrollable feelings of fear and sadness for no apparent reason and increasing fatigue for eight months. He says that these feelings occur most days, at home and at work, and have begun to affect his quality of life. He is otherwise in good health and he uses no medications or recreational drugs, including alcohol and tobacco. Examination shows no abnormalities. Which of the following is the most likely diagnosis?

A Generalized anxiety disorder
B Specific phobia
C Panic disorder
D Agoraphobia
E Culture-bound syndrome
A

A Generalized anxiety disorder

Generalized anxiety disorder is characterized by excessive, exaggerated fear and worry about everyday life events with no obvious reasons for worry lasting longer than 6 months. Patients with this condition are at increased risk of developing other conditions such as major depressive disorder and bipolar disorder.

243
Q

A 20-year-old woman comes to the office because of uncontrollable worry and apprehension since beginning university nine months ago. These feelings persist throughout each day and she cannot associate it with a particular event or trigger. She becomes consumed with planning and scheduling, becoming obsessed with everything she has to work on in the short-term. She was recently prescribed a benzodiazepine to help cope with her symptoms. Which of the following neurotransmitter alterations is most closely associated with the diagnosis?

A Decreased γ-aminobutyric acid (GABA) levels
B Decreased acetylcholine levels
C Decreased norepinephrine, serotonin, and dopamine levels
D Decreased serotonin levels
E Increased dopamine levels

A

A Decreased γ-aminobutyric acid (GABA) levels

While some studies show decreased serotonin in patients with anxiety, those studies are somewhat inconclusive, and the link between anxiety and GABA is far stronger.

Decreased levels of norepinephrine, serotonin, and dopamine are the findings in a patient with depression

244
Q

A 56-year-old Caucasian man comes to the office because of a longstanding history of chronic fatigue, sleep disturbances, and difficulty concentrating. His medical history is only relevant to alcohol abuse disorder and chronic back pain, to which he currently takes acetaminophen, oxycodone, and hydrocodone. Upon medical interrogation, the patient claims that his life is out of control, and expresses extreme concern about medication side effects and eventually becoming unable to take care of himself. Physical examination is noncontributory and additional laboratory studies are all within normal ranges. At the end of his visit, the patient is diagnosed with generalized anxiety disorder. Which of the treatments is most likely to be contraindicated in this patient?

A Alprazolam
B Buspirone
C Fluoxetine
D Pregabalin
E Venlafaxine
A

A Alprazolam

Alprazolam is a short-acting benzodiazepine used for the treatment of acute anxiety. This drug is contraindicated in susceptible individuals (i.e., alcohol use disorder, concomitant opioid use) because of the high risk of respiratory depression.

245
Q

A young woman comes to the office, sits down, and stares at the floor looking as though she might cry. She does not give a verbal response to any questions, but occasionally shrugs or nods her head. She has the medical intake form given to her by the receptionist in her hand, on which are her current medications, which include oral contraceptive pills, topiramate, sumatriptan, and buspirone. Which of the following is her most likely psychiatric diagnosis?

A Bipolar disorder
B Somatization disorder
C Generalized anxiety disorder
D Panic disorder
E Major depressive disorder
A

C Generalized anxiety disorder

Buspirone is used as an alternative treatment for patients with generalized anxiety disorder who cannot tolerate or have failed to respond to first line treatments –selective serotonin reuptake inhibitors (SSRIs) and serotonin-norepinephrine reuptake inhibitors (SNRIs).

246
Q

A 23-year-old man presents to the ED with increased anxiety and restlessness for the past 6 hours. Blood pressure is 168/102 mmHg, pulse is 112/min and regular, temperature is 101.2F. Physical examination reveals a resting tremor and DTR’s are 3+. Which of the following is the most likely etiology?

a. Benzodiazepines
b. Amphetamines
c. Opioids
d. Marijuana

A

b. Amphetamines

247
Q

With what disorder might a patient see their Physician Assistant for several times with different, and wide ranging complaints with no organic or objective findings, before diagnosing the disorder?

a. General anxiety disorder
b. PTSD
c. Complicated grief
d. Panic disorder

A

a. General anxiety disorder

248
Q

A 12-year-old boy is brought to the clinic for an annual physical. His parents state that they have noticed the patient becoming more anxious and quiet over the past year. Some days, he requests to stay home from school and refuses to give an explanation as to why. Medical history shows that he has always been on-track with his developmental milestones, though shyness is mentioned in several previous encounter notes. His temperature is 37.0°C (98.6°F), pulse is 85/min, respirations are 16/min, and blood pressure is 124/82 mm Hg. Physical examination shows a well-nourished young male of appropriate pubertal development. When the physician asks to speak alone with patient, the patient confides that he often feels scared at school when attempts to speak to anyone other than his close friends. Which of the following is the most likely diagnosis?

A Agoraphobia
B Culture bound anxiety
C G​eneralized anxiety disorder
D Panic disorder
E Selective mutism
A

E Selective mutism

Selective mutism (SM) is an anxiety disorder in which a person who is normally capable of speech does not speak in specific situations or to specific people.

249
Q

A 7-year-old girl is brought to the office because her mother is worried about her performance in school for the past year. The patient’s teacher says that the girl refuses to speak when called upon in class. She has a close group of two friends that she talks to and plays with at recess, but will not speak to other classmates when instructed to work on group projects. She does well on homework assignments and written tests, but her grades have begun to suffer due to this behavior in class. Family history is noncontributory. She is seen speaking with her mother in the patient waiting room, but during the patient interview, she avoids eye-contact with the physician and appears to be trembling. Child psychology consultation leads to a diagnosis of selective mutism. Which of the following is the mechanism of action of the recommended pharmacological approach to treatment?

A Binds to and inhibits both the norepinephrine transporter and serotonin transporter
B Binds to and inhibits monoamine oxidase A (MAOA)
C Binds to and inhibits the norepinephrine transporter
D Binds to and inhibits the serotonin transporter
E Inhibits the secretion of hydrogen ions

A

selective mutism

treatment = SSRI = Prozac (Fluoxetine)

D Binds to and inhibits the serotonin transporter (SSRI)

250
Q

A 25-year-old man comes to the emergency department because of a sore arm and difficulty using his hand. He has been evaluated multiple times for multiple somatic complaints, but medical evaluation has been consistently unrevealing. He also has a history of cocaine abuse, for which he was admitted to various substance abuse rehabilitation facilities. Often, his trips to the hospital coincided with scheduled court appearances after violating probation. Physical examination does not show obvious injury or focal abnormalities, and an X-ray of the arm is normal. A urine toxicology screen is negative. The patient was then told that he was to be discharged from the emergency department, at which point he stated that his pain was too great for him to leave the hospital and he demanded admission. Which of the following is the most likely diagnosis?

A Conversion disorder    
B Dissociative identity disorder    
C Factitious disorder    
D Malingering    
E Rheumatoid arthritis
A

D Malingering

Malingering is characterized by a patient’s conscious effort to deceive caregivers for secondary gain, including financial gain or to avoid school or work. An absence of any positive lab findings that coincide with a patient’s complaints should be suspected.

patient using hospital visits to avoid court

251
Q

A 15 year-old boy is referred to your psychiatry clinic because of repeated incidents of illness induced by ingestion of his own feces, some of which have resulted in extended hospital stays. Your conversation with him reveals that he is regularly bullied at school, and it is apparent that he would do almost anything to avoid having to go to school. Which of these terms best describes the direct cause of the feces-ingesting behavior?

A Avoidant personality disorder
B Conversion disorder
C Factitious disorder
D Illness anxiety disorder
E Malingering
A

E Malingering

Malingering is when a patient consciously attempts to deceive caregivers for secondary gain. For example, patients may exaggerate symptoms for financial gain, or to avoid school or work.

252
Q

A 13-year-old girl comes to the office because of “debilitating” right-sided knee, hip and back pain. She reports falling last week while skateboarding home from school. She has not been to school since. Her leg is atraumatic, and the anterior and posterior drawer signs are negative. She has full range of motion in her ankles, knees, and hips. Her gait is normal with adequate muscle strength bilaterally. Her reflexes are all intact and symmetric. A review of her chart reveals a hospitalization for abdominal pain of unknown origin, and multiple school excuses for various ailments. When asked how school is going, she refuses to give an answer, but her mother expressed concerned about bullies. Which of the following is the most likely presenting problem?

A Conversion disorder     
B Hypochondriasis     
C Malingering     
D Munchausen's disease     
E Somatization disorder
A

C Malingering

Malingering should be considered in patients who present with evidence of secondary gain (school avoidance, bullying) and complaints of physical ailments without clear physical, laboratory, or imaging findings.

253
Q

A 48-year-old male presents with crushing chest pain for 45 minutes. He is diaphoretic, nauseous and short of breath. His EKG demonstrates 4 mm of ST segment depression in leads I, aVL, V5 and V6. What is the most likely diagnosis?

A. esophageal spasm
B. hiatal hernia
C. acute lateral wall subendocardial myocardial ischemia
D. acute inferior wall myocardial subendocardial ischemia
E. acute anterior wall myocardial transmural ischemia

A

C. acute lateral wall subendocardial myocardial ischemia

It is obvious this patient is having an AMI. The ECG lead abnormalities suggest occlusion of the high lateral and low lateral walls of the LV - typically supplied by the circumflex - there are no Q waves - so subendocardial. Inferior AMI is leads II, III, avF. Anterior wall AMI is leads +/- VI, with V2-V4, and maybe V5 & V6.

254
Q

A 17-year-old male is stabbed in the anterior chest with an ice pick. He quickly becomes hypotensive, tachycardic and obtunded. His breath sounds are equal, his neck veins are distended, and his chest x-ray is normal. What condition is most likely?

A. Simple pneumothorax
B. Tension pneumothorax
C. Pericardial Tamponade
D. aortic laceration distal to the origin of the left common carotid artery
E. inferior vena cava laceration
A

C. Pericardial Tamponade

This is the typical clinical presentation of tamponade (hypotension, JVD). He was stabbed and probably has a mediastinal bleed (so, I think the mediastinum would be widened on his chest film — but the “normal CXR” is what rules out the pneumothorax - another common problem in a stabbing injury.

255
Q

An elderly patient is recovering from a pneumonia caused by Influenza. Suddenly, the patient’s condition deteriorates. You order a chest x-ray that demonstrates an air-fluid level, suggestive of an abscess. You empirically begin treatment for:

A. Streptococcal pneumoniae
B. Chlamydia
C. Haemophilus influenzae
D. Staphylococcus aureus
E. Legionella pneumophila
A

Answer: D
Staphylococcus aureus

Empyema (lung abscess) is almost always staph aureus. While strep pneumonia is the most common CAP, when a complication like an empyema occurs, it is almost always staph.

256
Q

An 18-year-old female presents after completing her freshman year at college. Two months ago, she developed “bronchitis” which has persisted. She has become increasingly tired and has frequent night sweats. A chest x-ray today reveals a left mediastinal mass. What is the next step in her evaluation?

A. repeat the chest x-ray in six months
B. ultrasound of the left hemithorax
C. bone scan
D. CT scan of the chest
E. proceed directly to bronchoscopy with biopsy
A

D. CT scan of the chest

Any patient presenting with mediastinal nodes should have a CT. This patient may have sarcoid or Hodgkin’s or another type of cancer - CT is indicated.

257
Q

A female patient presents for an evaluation for primary amenorrhea. She is noted to be short of stature and have short fourth metacarpals. What diagnosis do you suspect?

A. XO
B. XXY
C. XXX
D. XXXY
E. Trisomy 21
A

Answer: A
XO

This is the genotype for Turner’s syndrome. XXY is the genotype for Klinefelter’s (which is a tall, mildly retarded male). Trisomy 21 is Down’s. The other two are not common genotypes.

258
Q

A patient with known hyperlipidemia, taking clofibrate, presents to your office complaining of severe myalgias and weakness. He has noted a dark color to his urine. The laboratory calls with a panic value CPK of 8500. He was recently started on a new medication, four days earlier. Which of the following is most likely the new medication?

lovastatin
hydrochlorothiazide
dexamethasone
phenytoin
ampicillin
A

Answer: A
Lovastatin

Statin + fibrate = suspicion for rhabdomyolysis in a patient with myalgias, ALWAYS order a CPK. (statin alone, or fibrate alone may cause it too, but the combination is most likely)

259
Q

A patient at a Christmas party on a cruise ship develops a terrible headache. His blood pressure is 224/158 mm Hg. Which of the following agents is this patient most likely taking regularly?

A. tricyclic antidepressants
B. barbiturates
C. benzodiazepines
D. monoamine oxidase inhibitors
E. phenothiazines
A

Answer: D
Monoamine oxidase inhibitors

This is a typical scenario for MAOI induced malignant hypertension from eating the “wrong” foods, which are frequently found on a Christmas buffet, (cheese, sausages, red wine, etc).

260
Q

A 51-year-old male with renal artery stenosis and an elevated BUN and creatinine, is newly diagnosed as hypertensive. Which of the following agents should be avoided?

A. hydrochlorothiazide-triamterene
B. prazosin
C. nifedipine
D. verapamil
E. furosemide
A

Answer: A
Hydrochlorothiazide-triamterene

This is a potassium-sparing diuretic and should be avoided in anyone with renal disease. ACEI should be avoided as well (but none of these are ACEI’s).

261
Q

A 6-year-old female presents complaining of right ear pain. The tympanic membrane cannot be well visualized. She complains of pain when the tragus is manipulated. Several periauricular lymph nodes are easily palpable. Which of the following is the most likely diagnosis?

A. acute otitis media
B. acute otitis externa
C. eustachian tube dysfunction
D. Ramsey-Hunt syndrome
E. perforated tympanic membrane
A

Answer: B
Acute otitis externa

OE is described here. OM would not cause preauricular nodes and tragal tenderness. Eustachian tube dysfunction would cause middle ear pain (like when you’re up in a plane). Ramsay-Hunt is herpes zoster of the ear canal and would be visible. A perforated TM would cause middle ear, not external ear pain (typically).

262
Q

A 34-year-old woman comes to the clinic because of fatigue, low energy, and a depressed mood. She says that she has felt this way for most of her life. She feels depressed most of the time but denies any recent stresses or significant losses in her life. She also says that she is doing well at work and that she recently received a promotion. She has no interests other than her job and says that her self-esteem is very low. She denies suicidal thoughts but says that she does not care if she dies. She has had no sleep disturbance, change in appetite, or difficulty concentrating. She is taking no medications and denies substance abuse. Results of a recent medical evaluation required by her employer were all normal, including a physical examination, EKG, CMP, CBC, urinalysis, and TSH level. Which of the following is the most likely diagnosis?

A Bipolar II disorder
B Adjustment disorder with depressed mood
C Major depressive disorder
D Hypothyroidism
E Bipolar I disorder
F Persistent depressive disorder (dysthymia)

A

F Persistent depressive disorder (dysthymia)

fatigue, low energy, and a depressed mood

She says that she has felt this way for most of her life

this patient has persistent depressive disorder (dysthymia). The DSM-V defines this disorder as a condition in which patients have a depressed mood for most of the day, for more days than not, for at least two years. While depressed, the patient must also have two or more of the following characteristics: a change in appetite, a change in sleep, low energy or fatigue, low self-esteem, poor concentration or difficulty making decisions, and/or feelings of hopelessness. There must be no history of a manic or hypomanic episode, substance abuse, a chronic psychotic disorder, or an organic cause. It tends to have an earlier onset and a more chronic course than major depressive disorder which tends to more episodic.

263
Q

A 48-year old woman comes to the office for a routine medical examination. Her medical history is relevant for osteoarthritis, type II diabetes, and a right leg amputation 4 years ago. She currently works as a supermarket cashier. When asking the patient about her life, she claims that her husband doesn’t care for her anymore and that the last time they engaged sexual intercourse was more than 3 years ago. She additionally mentions she can’t sleep and often feels worthless after waking up, she can’t even remember the last time she laughed. Physical examination is noncontributory. Despite all her feelings, she manages to play cards with her friends twice a week. Her temperature is 36.1°C (96.9°F), pulse is 84/min, respirations are 19/min, and blood pressure is 110/70 mm Hg. After finishing her medical visit, she seems sad and indifferent to all indications. Which of the following is the most likely diagnosis?

A Major depressive disorder 
B Persistent depressive disorder 
C Seasonal affective disorder 
D Type I bipolar disorder 
E Type II bipolar disorder
A

B Persistent depressive disorder

can’t sleep = sleep irregularities
worthless = hopelessness

Persistent depressive disorder is a mood disorder characterized by chronic and mild depressive symptoms that last for ≥2 years. Unlike depression, patients with a persistent depressive disorder are functional at a suboptimal level.

264
Q

A 35-year-old man comes to the emergency department because he has felt depressed and very tired. He says that he has been feeling this way for several years, but now it has started to affect his marriage. He has also felt a decreased appetite but has not had any changes in weight. He denies suicidal or homicidal ideations and admits to low self-confidence and difficulty finding pleasure. However, these feelings have not interfered with his work. His medical history is non-contributory. His temperature is 37ºC (98.6ºF), pulse is 74/min, respiratory rate is 12/min, and blood pressure is 119/87 mmHg. Physical examination shows no abnormalities. Which of the following is the most likely diagnosis?

A Cyclothymic disorder      
B Adjustment disorder with depressed mood      
C Dysthymic disorder      
D Major depression      
E Bipolar disorder
A

C Dysthymic disorder

Dysthymic disorder condition has a duration of at least 2 years in adults and 1 year in adolescents and children. It is manifested as a depressed mood, or irritable mood in children, for most of the day, occurring more days than not.

The DSM-V defines this disorder as a condition in which patients have a depressed mood for most of the day, for more days than not, for at least two years. While depressed, the patient must also have two or more of the following characteristics: a change in appetite, a change in sleep, low energy or fatigue, low self-esteem, poor concentration or difficulty making decisions, and/or feelings of hopelessness.
To diagnose dysthymia, any major depressive episodes must not have occurred in the first 2 years of the illness (the first 1 year in children) and history of mania should not exist.

265
Q

A 43-year-old man comes to the office because of difficulty falling asleep for the past four years. During this time, he endorses depressed mood, and low energy. His friends describe him as a reserved individual. He has recently been in trouble with his employer up for not showing up to work. Which of the following is the most likely disorder faced by the patient?

A Adjustment disorder 
B Bipolar disorder
C Cyclothymia
D Dysthymia
E Major depressive disorder
A

D Dysthymia

difficulty falling asleep = sleep irregularities
low energy = fatigue
depressed mood
4 years = > 2 years in adults

Dysthymia is characterized by depressive symptoms that do not meet criteria for major depressive disorder and last for at least two years. Treatment begins with selective serotonin reuptake inhibitors but can also include serotonin norepinephrine reuptake inhibitors, tricyclic antidepressants, and monoamine oxidase inhibitors.

266
Q

A patient who exaggerates an illness in order to get out of work would be an example of which of the following?

a. Factitious disorder
b. Malingering
c. Hypochondriac
d. Munchausen’s

A

b. Malingering

267
Q

A 25-year-old woman who is in the surgical ward after an appendectomy is being evaluated for an erythematous excoriation with a serosanguinous crust on her right antecubital fossa where an IV line was placed. The nurse says that the patient has been picking at that area continuously despite being urged to stop. Physical examination shows dozens of hypertrophic, hyperpigmented scars on her chin, cheeks, and forearms. The patient says that she has caused them herself since her teenage years because she has not been able to break the stress habit, even after several infections. Which of the following is the most likely diagnosis?

A Methamphetamine abuse
B Atopic dermatitis
C Excoriation disorder
D Scabies
E Delusional parasitosis
A

C Excoriation disorder

Excoriation disorder is repetitive picking, scratching, rubbing, digging, or squeezing of skin, resulting in visible damage and impairment in social functioning.

Excoriation disorder (skin picking disorder) is defined in the DSM-V as repetitive picking, scratching, rubbing, digging, or squeezing of skin, resulting in visible damage and impairment in social functioning. It is grouped with trichotillomania and body dysmorphic disorder.

268
Q

A 12-year-old girl is brought to her pediatrician’s office because of an elongated nose. She reports that her nose is too big, long, and ugly. She describes it as “disgusting” and is obsessed with getting a rhinoplasty. Her parents state that she has been harassing them for months for the operation and constantly checks her nose in the mirror. The patient states that she cannot sleep at night and spends hours looking over fashion magazines. She says that she cannot stand the sight of her own nose and will try to break it if her parents will not pay for her rhinoplasty. Head, ear, eyes, and neck examination shows an average sized nose with no obvious functional defects. Which of the following is the most likely diagnosis?

A Anorexia nervosa
B Body dysmorphic disorder
C Malingering
D Delusional disorder
E Social anxiety disorder
A

B Body dysmorphic disorder

Body dysmorphic disorder is a preoccupation with an imagined or slight defect in appearance, sufficient to cause the patient significant distress or interfere with their functioning in some way.

Body dysmorphic disorder is a DSM-V diagnosis characterized by a perceived flaw with the patient’s body which they find “repulsive” and “disgusting.” They are preoccupied with a minor defect with their appearance. Typically, they will describe their imperfection out of proportion to physical examination. This imperfection will occupy their time and they will spend hours distressing over their flaw (i.e. examining their nose in the mirror or analyzing the noses of fashion models). These patients will often go to the extremes to fix their defect and can demand plastic surgery to fix their flaw. Body dysmorphic disorder is more common in women than men and generally affects patients between the age of 15 to 20. An selective serotonin re-uptake inhibitor may be helpful in alleviating the symptoms in these patients. Surgery should not be given to these patients as they rarely help correct the perceived imperfections in these patient’s appearance

269
Q

A 6-year-old boy is brought to the emergency department after a sudden loss of consciousness. His medical history is relevant for attention deficit disorder, to which he currently takes methylphenidate. The mother, who is a nurse, mentions that the boy was jumping in a playground when he suddenly collapsed to the floor. Upon hospital admission, initial laboratory evaluation revealed no abnormalities, except for low blood glucose. After glucose administration, the child recovers completely. Exhaustive studies are requested to evaluate his hypoglycemia all of which are negative. When you inform the mother that his discharge is going to be soon, she expresses anger, telling you that she will take her child to another hospital. Next morning, the patient’s blood glucose is 50 mg/dl (2.7 mmol/L). Physical examination shows dilated pupils and tachypnea. Which of the following is the most appropriate next step in management?

A Contact child welfare services
B Initiate IV glucose infusion
C Protect the child 
D Report the mother for child abuse
E Try to contact the boy's father
A

B Initiate IV glucose infusion

Munchausen syndrome by proxy is a form of child abuse. Here, the patient is experiencing an acute episode of hypoglycemia, therefore, management is to initiate an IV glucose infusion to resolve his medical needs. After this resolution, management should be directed towards the child’s protection.

The patient in the vignette is experiencing Munchausen syndrome by proxy, a rare type of factitious disorder in which the caregiver fabricates or induces an illness in another person (in this case, via secret insulin injections by the mother) to meet their emotional needs. The diagnosis of this syndrome should be first suspected when there are discrepancies between clinical findings and medical history. Also, if a patient does not improve despite extensive workup and optimal therapy or if the patient improves, but, worsens only when the caregiver is present.

270
Q

A 6-year-old girl is brought to the emergency department because of a 48 hour history of vomiting. The patient has been seen in the same emergency department 3 times within the past 6 months for the same complaint. The patient’s past medical history is significant for asthma. The vomiting subsides shortly after admission to the hospital for intravenous hydration. Workup and imaging reveals no etiologies for her vomiting.

During her hospital stay, the mother seems very eager to interact with the the medical team and is very involved in the care of her child. The mother stays in her daughter’s room during every admission. During a visit from the father, he notes that the patient only has these symptoms of vomiting while in the custody of her mother. After ensuring proper fluid and electrolyte balance, what is the next best step in the management of this patient?

A Colonoscopy
B Repeat computerized axial tomography scan
C Consult child abuse services
D Technetium-99 scan
E Administration of fluvoxamine
A

C Consult child abuse services

271
Q

A 5-year-old girl is brought in to the ED by her mother after collapsing at home. The mother states the child has been having diarrhea with at least 12 loose stools for the last 3 days. Examination reveals the child to be responsive, but lethargic and dehydrated. The child is admitted to the hospital and the mother is very involved in the child’s care. While her mother is present the child’s condition improves slightly but continues to have multiple loose stools. When the mother had to leave to go to work, the child responded well to treatment without any further episodes of diarrhea. What is the most likely diagnosis?

a. Malingering
b. Münchausen by proxy
c. Depression
d. Münchausen syndrome
e. Factitious disorder

A

b. Münchausen by proxy

272
Q

A 9-year-old boy is brought to the emergency room with shortness of breath. His mother reports that his symptoms began two days ago and that this is the first time that her son has ever been sick. On exam, he is pale, diaphoretic, and lethargic, and his glucose is found to be 20 mg/dL. When looking up the patient’s chart, the physician finds that the patient has been brought into the emergency room four times in the past year with the same symptoms, and his C-peptide levels at all visits have been low. After administration of glucose, what is the most appropriate next step?

A Discharge to home with follow-up at with an endocrinologist
B Refer the patient’s mother to a psychiatrist
C Contact the patient’s outpatient pediatrician for follow-up diabetes testing
D Contact the hospital ethics board
E Contact child protective services

A

E - Contact child protective services

This clinical presentation is most consistent with Factitious disorder imposed on another (Munchausen syndrome by proxy). This is considered a form of child abuse, and providers should contact child protective services in suspected cases. Factitious disorder imposed on another is a psychiatric condition in which a patient consciously reports false symptoms, or induces symptoms, with the goal of playing the “sick role.” Patients may inflict significant self-harm in the process of creating symptoms, such as injecting insulin or swallowing food that is known to be contaminated

273
Q

A 34-year-old woman comes to her physician for the 6th time this month with persistent fatigue, pain in the buttocks, and severe concern about the possibility of being ill. She regularly presents to her physician’s office with diseases she has found on the internet and asks if the physician thinks she might have them. Her concern about illness and the persistence of her symptoms have caused her to reduce activities she considers “risky” and change jobs several times in the past two years. Medical examinations are consistently unremarkable. Which of the following is the most likely diagnosis?

A Factitious disorder
B Functional neurological symptom disorder
C Generalized anxiety disorder
D Illness anxiety disorder
E Somatic symptom disorder
A

E Somatic symptom disorder

Somatic symptoms disorder in the DSM-V describes patients who are worried about having a serious illness and manifest somatic symptoms. In the DSM-IV it was known as hypochondriasis.

The patient in this question most likely has somatic symptom disorder, as evidenced by preoccupation with being ill as well as the presence of symptoms. In the DSM-V, hypochondriasis has been eliminated as a diagnosis due to its perception as a pejorative term. The diagnosis has been further divided into those patients that manifest somatic symptom (somatic symptom disorder) and those that do not (illness anxiety disorder). This patient manifests some somatic symptoms (persistent pain in the buttocks and fatigue) in addition to illness anxiety interfering with normal social functioning, and thus best fits the diagnostic criteria of somatic symptom disorder.

274
Q

A 45-year-old man comes to the office for a check-up. He complains of increasing stool frequency from every other day to every day over the past year. He says that he has colon cancer and would like to be evaluated for treatment. He admits having been worked up for colon cancer by several physicians who could not find the cancer despite him constantly telling them it is there. Family and medical history is noncontributory. Cardiopulmonary examination is normal and abdominal examination shows positive bowel sounds and a soft non-tender non-distended abdomen with no palpable masses. Stool occult blood testing is negative. Which of the following is the most likely diagnosis?

A Conversion disorder
B Factitious disorder     
C Illness anxiety disorder
D Malingering     
E Pain disorder
A

C Illness anxiety disorder

Illness anxiety disorder is characterized by misinterpretation of minor bodily symptoms as a serious illness. Patients often self diagnose themselves and do not believe the physician’s negative work-up. Preoccupation with fear of death can cause a great deal of distress or interfere with a person’s ability to perform important activities.

Illness anxiety disorder was previously known as hypochondriasis, however this is not a possible diagnosis in the DSM-V. Another similar DSM-V diagnosis to illness anxiety disorder is somatic symptom disorder which is characterized by excessive preoccupation with a chronic symptom.

275
Q

A 45-year-old man is brought to the emergency department because of chest pain for the past 3 hours. The patient has been seen several times in the last year for similar symptoms, however, each time cardiac evaluation is normal. History reveals no cardiac risk factors. The patient says he barely leaves his house and had to quit his job because he fears his symptoms of chest pain will begin and he will not be able to reach a hospital. After thorough evaluation, all tests are negative. The patient becomes frustrated and storms out of the hospital demanding a second opinion. Which of the following is the most likely diagnosis?

A Adjustment disorder     
B Conversion disorder     
C Somatic symptom disorder
D Panic disorder     
E Post-traumatic stress syndrome
A

C Somatic symptom disorder

Somatic symptom disorder is a DSM-V diagnosis characterized by excessive preoccupation with minor bodily symptoms. Patients often self diagnose themselves and do not believe the physician’s negative work-up. Preoccupation with or fear of their symptomatology can often inhibit a patients ability to perform important activities, such as work, school activities, or family and social responsibilities. The preoccupation must be present for at least six months.

276
Q

A 28-year-old woman comes to the clinic because of ongoing diffuse low back pain for the past 4 months. She rates the pain as 5 on a 10-point scale. An MRI 1 months ago showed a slight disc bulge. She also reports intermittent headache, nausea, and diarrhea for the past 8 months. She states she has been taking significant amounts of time off work and she lies awake at night worrying about her symptoms. Her records show a history of knee pain, and intermittent weakness in her right arm both of which were investigated thoroughly with no cause identified. Which of the following is the most likely diagnosis in this patient?

A Body dysmorphic disorder     
B Factitious disorder     
C Fibromyalgia     
D Hypochondriasis     
E Somatic symptom disorder
A

E Somatic symptom disorder

Unlike the previous criteria for diagnosis of somatization disorder, which required the presence of symptoms across four groups (pain, sexual, gastrointestinal & neurosensory) there are now no specific symptom requirements for somatic symptom disorder. Under the DSM-V criteria the symptoms generally are present for at least 6 months, may or may not be explained by a medical condition, but must cause significant dysfunction or distress, and be associated with excessive thoughts, feelings, or behaviors.

277
Q

A 25-year-old woman comes to the clinic because of abdominal, shoulder, and head pain for the past year. This is her first time being seen at this clinic, but she brings copies of her medical charts from 3 other clinics, all within the past year. She states that the three other physicians were unable to diagnose her, “even though they ran all kinds of laboratory tests and scans.” She states that she may be fired from her job for missing too many days at work to attend medical visits. She does not like to take pain medication and really wants to be diagnosed “before it is too late.” Temperature is 37.0°C (98.6°F), pulse is 70/min, respirations are 10/min, and blood pressure is 126/78 mm Hg. Physical examination shows no abnormalities. Routine laboratory studies are within normal limits. Which of the following is the most likely diagnosis?

A Factitious disorder
B Malingering
C Illness anxiety disorder
D Conversion disorder
E Somatic symptom disorder
A

E Somatic symptom disorder

Somatic symptom disorder is characterized by a variety of complaints regarding physical symptoms that result in significant social or occupational distress. Individuals may complain primarily of pain or fatigue in different body parts, and often visit several physicians, resulting in expensive and wide-ranging laboratory studies, hospitalizations, procedures, and potentially surgeries.

278
Q

A 30-year-old woman comes to the office because of abdominal pain. She tells you that she thinks that she “may” have cancer. Family history is non-contributory and previous work-up by several physicians in the past year have been negative. The patient undergoes several diagnostic tests which are also found to be negative. She calls the office the next day asking if cancer diagnosis is “truly” negative and tells you that the pain is causing her to miss work. Which of the following is the most likely diagnosis?

A Conversion disorder
B Factitious disor​der
C Illness anxiety disorder
D Malingering​
E Pa​in disorder
A

C Illness anxiety disorder

Illness anxiety disorder is characterized by misinterpretation of minor bodily symptoms as a serious illness. Patients often self diagnose themselves and do not believe the physician’s negative work-up. Preoccupation with fear of death must cause a great deal of distress or interfere with a person’s ability to perform important activities.

279
Q

A 21-year-old woman comes to the emergency department because of a sudden loss of sight and inability to walk. She complains of pain when seeing bright light and prefers to wear sunglasses. On physical examination, she is not able to track the fingers and has no peripheral vision, but does display a response when touching her eyelashes. A magnetic resonance imaging study is not able to determine a diagnosis. She is told that the etiology of her sudden blindness or difficulty to walk is unknown. She replies indifferently, stating that maybe she’ll get better one day. She requests breakfast and and appears unusually apathetic. She appears to be unable to put her feelings into words. Which of the following is the term that describes this condition?

A Dysthymia
B Alexithymia
C Aphasia
D Pure alexia
E Dyslexia
A

B Alexithymia

Alexithymia describes a patient that cannot put their feelings into words. This often affects those with somatization disorder or conversion disorder.

280
Q

A 25-year-old woman comes to the urgent care clinic because of she felt a sudden onset of left lower extremity paralysis that lasted for a few minutes when she was at home. The patient is in moderate distress and states that this morning her partner of five years abruptly ended their relationship and will not answer her calls. She reports feeling depressed and confused as to why this happened. She denies thoughts of harming herself or others at this time. Neurologic and musculoskeletal examination of the patient is normal. Laboratory studies were also within normal limits. Which of the following options is the best first line of treatment for this patient?

A Diazepam
B Fluoxetine
C No treatment necessary
D Psychiatric admission
E Psychotherapy
A

conversion disorder

Conversion disorder is a somatic symptom disorder characterized by neurological deficits (sensory or motor) preceded by conflicts or other stressors. Common complaints are weakness, paralysis, tremors, psychological seizures, and sensory disturbances (aphonia, deafness, blindness).

Treatment consists of education (1st line) about the illness and cognitive behavioral therapy (2nd line) therapy. Medication with a serotonin-norepinephrine reuptake inhibitor (SNRI) is the third line treatment.

281
Q

A 27-year-old woman presents to her gynecologist for follow-up for irregular, painful menstrual cycles. She has undergone a complete workup, which is negative. She is very distressed about her condition and states that she thinks it’s probably related to whatever is causing her bouts of generalized “weakness,” which have been occurring about once a month for the last year and associated with a headache. She has a past medical history of several food intolerances and irritable bowel syndrome, both of which cause her significant abdominal pain and are managed by a gastroenterologist. She states that she’s recently had episodes of chest pain, lasting 20-30 minutes, unrelated to exercise, and requests to be referred to both a cardiologist and a neurologist. On exam, she is a thin, athletic woman who appears healthy. What is the most appropriate next step in treatment?

A Discharge her from care
B Establish a primary care provider
C Send her directly to the ER
D Refer her to a psychiatrist
E Refer her to both a cardiologist and a neurologist
A

B) Establish a primary care provider

This patient has somatic symptom disorder. One of the most important steps in treatment for this disorder is regular (at least monthly) follow-up with a consistent primary physician the patient trusts.

282
Q

A 25-year-old comes to the office because of general feelings of sadness. She has been feeling sad, run-down, and easily annoyed for the past three days. She gave birth to a healthy baby girl a week ago via spontaneous vaginal delivery. Right after her daughter was born, she was overjoyed. However, now, even when her daughter is asleep, she finds she is unable to concentrate on work or get any rest herself. These symptoms have not compromised the care of her daughter. Examination shows some elements of depression, but she answers questions appropriately and her hygiene is good. Which of the following is the most appropriate next step in management?

A Reassurance and support with a follow-up appointment in 1-2 weeks.
B Recommend emergency admission
C Recommend starting a selective serotonin reuptake inhibitor (SSRI).
D Recommend starting a tricyclic antidepressant (TCA).
E Refer to cognitive behavioral therapy.

A

A Reassurance and support with a follow-up appointment in 1-2 weeks.

Postpartum blues affects 70-80% of postpartum women, characterized by mild depressive symptoms that typically resolve within two weeks. Roughly 10% go on to develop the more severe and longstanding postpartum depression.

This patient has postpartum blues. This condition affects 70-80% of postpartum women. These women experience mild depressive symptoms such as sadness, insomnia, difficulty with concentration, and increased irritability. They are still able to care for their newborn and themselves. Typically, the postpartum blues begin 2-4 days after giving birth and resolve within two weeks. No treatment is necessary aside from reassurance and support. Approximately 10% of women will go on to develop postpartum depression. These women have more severe and longer lasting depressive symptoms that may interfere with their ability to care for their child. Postpartum depression can be treated with antidepressant medication and/or cognitive behavioral therapy. It is too early to recommend these therapies to this patient, but she should be closely followed to ensure that her symptoms resolve in the expected time period.

283
Q

A 35-year-old woman comes to the office because she has been “feeling down” for the past few months. She has not been sleeping well and has lost 6.8-kg (15-lb) over the past three months because of lack of appetite. She also says that she previously enjoyed going to the movie theater and attending a book club with friends, but she does not want to anymore. When questioned about suicidal thoughts, she reports having occasional thoughts of hurting herself, but then says “oh, but I would never actually kill myself.” Her past medical history includes seizures, but she is off medication as she has been seizure free for a number of years. Which of the following has the most similar neurotransmitter dysregulation as the one seen in this patient?

A Alzheimer disease     
B Cocaine intoxication     
C Obsessive compulsive disorder     
D Schizophrenia     
E Tourette disorder
A

C Obsessive compulsive disorder

Major depressive disorder is believed to be mediated primarily by serotonin and SSRIs increase serotonin and cause a decrease in symptoms. Serotonin is also thought to play a key role in obsessive compulsive disorder.

This patient is having an episode of major depression, and based off of her symptoms, meets DSM-V criteria for major depressive disorder (MDD). To diagnose MDD, a patient must have a major depressive episode lasting >2 weeks, with at ≥5 of the following symptoms: depressed mood, anhedonia, weight or appetite changes, sleep disturbances, psychomotor agitation or retardation, loss of energy, feelings of guilt or worthlessness, decreased concentration, and suicidal ideations.

This patient meets criteria because she has weight loss with decreased appetite, depressed mood, anhedonia (loss of enjoyment in normally pleasurable activities), sleep disturbances, and suicidal ideations. Initial treatment of choice for most patients with MDD is a selective serotonin reuptake inhibitor (SSRI).

Both major depressive disorder and obsessive compulsive disorder are thought to be due to a decrease in serotonin levels. Both disorders are treated with selective serotonin reuptake inhibitors as first line medical management.

284
Q

A 30-year-old woman comes to the office because of alternating constipation and diarrhea. This is her ninth visit to the office in the past four months. Her previous visits have been because of fatigue, sleep disturbances, chronic headaches, dyspepsia, and neck pain. When asked if she has been experiencing any stressors in her life that might be contributing to her poor health, the patient suddenly bursts into tears. She states that her husband moved out several months ago and has filed for a divorce. Which of the following factors most strongly suggests a depressive episode rather than a somatoform disorder?

A Absence of sexual or reproductive symptoms
B Concurrent diagnosis of a panic disorder
C Current age
D Headaches that improve with sumatriptan
E Presence of mood symptoms beginning before somatic symptoms

A

E Presence of mood symptoms beginning before somatic symptoms

Major depressive disorder can be characterized by somatic symptoms. The presence of mood symptoms beginning before somatic symptoms would suggest that depression is the primary underlying condition.

285
Q

A 16-year-old girl comes to the clinic because she has not eaten a lot for two days. Her mother has become worried by her daughter’s change in behavior over the past month and brought her to the clinic. She says that her daughter barely speaks to her and sleeps >15 hours a day. She has not expressed any interest in spending time at school or with friends and has isolated herself. These symptoms have worsened over the past few days. Physical examination shows a thin, disheveled patient with dark circles around her eyes and fingernail scratches on her arms. Which of the following should be evaluated as the most appropriate next step in management?

A Depressive episode
B Eating disorder
C Psychotic episode
D Substance abuse
E Suicidal ideation
A

E Suicidal ideation

Although this patient likely requires treatment for depression, the risk of suicide or self-harm should be assessed first. Not all depressed patients display suicidal ideation or self-destructive behavior.

In adolescents with major depressive disorder, suicidal ideation or behavior is an alarming feature that should prompt urgent evaluation and treatment. This may include inpatient hospitalization.

286
Q

A 30-year-old man comes to the clinic because of recurrent thoughts of suicide. During the last month, he has slept poorly, gained 4.5-kg (10-lb), performed poorly at work, and was recently involved in a physical altercation in a bar. He has lost interest in all activities and says “I want to die” during the end of the interview. Physical examination shows superficial lacerations over his volar wrist. The patient failed to respond to fluoxetine and cognitive behavioral therapy in the past. The patient was recently placed on escitalopram, but no improvement has yet been noted. Which of the following is the most likely underlying etiology of this patient’s condition?

A Interference of GABA synthesis from the substantia nigra
B Interference of acetylcholine synthesis from the nucleus accumbens
C Interference of dopamine synthesis from the basal nucleus of Meynert
D Interference of norepinephrine synthesis from the ventral tegmentum
E Interference of serotonin synthesis from the raphe nucleus

A

E Interference of serotonin synthesis from the raphe nucleus

Patients with major depressive disorder have an underlying interference in the production or activity of neurotransmitters. These include serotonin, a neurotransmitter primarily synthesized in the raphe nucleus.

others are not properly matched with their synthesis sites

287
Q

A 40-year-old woman comes to the clinic because she has been feeling depressed for the past three months. She has been having trouble falling asleep and has lost 9-kg (20-lb) during this time. She also says that she used to love playing tennis and going for walks, but just does not have the energy to do either of those activities anymore. When questioned about suicidal thoughts, she says she has occasional thoughts of hurting herself, but she does not have a plan in mind to actually do so. Which of the following neurotransmitter alterations best explains the cause of this patient’s symptoms?

A Decreased acetylcholine     
B Decreased dopamine     
C Decreased serotonin     
D Increased dopamine     
E Increased norepinephrine
A

C Decreased serotonin

Major depressive disorder has a biological etiology of neurotransmitter dysfunction. It is characterized by decreased activity of serotonin, norepinephrine (NE), and dopamine (DA) at the level of the synapse, changes in GABA and glutamate, and changes in brain circuitry. Depressed serotonin activity is thought to be the most significant factor.

288
Q

A 26-year-old woman comes to the office because she has felt increasingly sad and tired for a week. She gave birth to a healthy baby boy one month ago via an uncomplicated spontaneous vaginal delivery. She really enjoyed taking care of her son for the first week after he was born, but feels things have become increasingly difficult since then. She also says that she keeps starting tasks around the house, but constantly forgets what she is doing, or loses interest in the project at hand and that she hasn’t slept in days. Examination shows she appears disheveled and is slightly malodorous. Her affect is flat, and she is slow to answer questions, but does reply appropriately. Which of the following is the most likely diagnosis?

A Birth-related post-traumatic stress disorder
B Normal post childbirth mood changes
C Postpartum blues
D Postpartum depression
E Postpartum psychosis
A

D Postpartum depression

Postpartum depression affects approximately 10% of postpartum women. It is characterized by severe depressive symptoms that last and worsen for more than two weeks after delivery.

Postpartum depression (PPD) affects approximately 10% of women. A history of depression either prior to, or during pregnancy is the primary risk factor. PPD is characterized by changes in sleep, energy level, appetite, weight, and concentration. Additionally, mothers with PPD may feel overwhelmed and unable to care for the baby or feel that they are not bonding with their baby. Recommended treatment for PPD includes psychotherapy and pharmacotherapy with selective serotonin reuptake inhibitors.

289
Q

A 38-year-old woman comes to the office because she is having difficulty coping with her 4-week-old son and is concerned that she is not bonding with him. She also reports increased irritability, lack of sleep, decreased appetite, low mood, and poor concentration. Medical history includes an episode of major depression in her early twenties, but is otherwise non-contributory. Examination shows a disheveled appearing woman, who answers questions in a quiet voice and avoids eye contact. Which of the following treatments should most likely be avoided?

A Selective serotonin reuptake inhibitors
B Anti-psychotics
C Counseling
D Serotonin/norepinephrine reuptake inhibitors
E Tricyclic antidepressants

A

B Anti-psychotics

Postpartum depression (PPD) affects approximately 10% of women. It is characterized by changes in sleep, energy level, appetite, weight, and concentration. Additionally, mothers with PPD may feel overwhelmed and unable to care for the baby, or feel that they are not bonding with their baby.

Anti-psychotics are not indicated for postpartum depression, and therefore should be avoided. They may be used in the treatment of postpartum psychosis, which is characterized by onset within two weeks of childbirth of a combination of psychotic symptoms (e.g. hallucinations and delusions) and symptoms of a mood disorder. Women with postpartum psychosis are at increased risk of harming their baby. Hospitalization may be appropriate and be considered on a case-by-case basis.

290
Q

A 28-year-old woman comes to the office because of difficulty coping with her 6-week-old baby and is concerned that she is not bonding properly with him. She also says she has a bad mood, irritability, poor sleep, decreased appetite, and poor concentration. She had an episode of major depression in her early twenties, but she has been feeling well for over 5 years. Which of the following statements best describes the situation?

A This is a normal reaction to childbirth and does not require further treatment or follow up
B This patient has childbirth-related post traumatic stress disorder and should seek out treatment
C This patient has postpartum blues, which should resolve on its own
D This patient has postpartum depression and should seek out treatment
E This patient has postpartum psychosis and should be hospitalized immediately

A

D This patient has postpartum depression and should seek out treatment

Postpartum depression (PPD) affects approximately 10% of women. A history of depression either prior to, or during pregnancy is the primary risk factor. PPD is characterized by changes in sleep, energy level, appetite, weight, and concentration. Additionally, mothers with PPD may feel overwhelmed and unable to care for the baby, or feel that they are not bonding with their baby. Recommended treatment for PPD includes psychotherapy and pharmacotherapy with selective serotonin reuptake inhibitors.

291
Q

A 25-year-old man comes to the psychiatry clinic for a follow up after being diagnosed with unipolar depression. Various trials of selective serotonin reuptake inhibitors and serotonin-norepinephrine reuptake inhibitors have shown to be ineffective. Psychiatric evaluation shows various atypical features to his depressive episodes. Which of the following features would most likely be present during one of this patient’s depressive episodes?

A Emotional detachment, muteness, depersonalization
B Hurried speech, delusions, hallucinations
C Improved mood in reaction to positive events, hyperphagia, sleeping at least two hours more when depressed
D Inability to focus, need for control, excessive worrying
E Social or occupational conflicts, talkativity, risky behaviors

A

C Improved mood in reaction to positive events, hyperphagia, sleeping at least two hours more when depressed

Central features of atypical depression include mood reactivity, hypersomnia, and hyperphagia. Treatment of choice is selective serotonin reuptake inhibitors.

292
Q

A 35-years-old man comes to the clinic because of a one-month history of fatigue and insomnia. He explains that he cannot concentrate like before and that he is always tired. The patient says that he has gone through a recent divorce and that he only sees his kids on Saturdays. He has also lost most of his savings investing in a pyramid scheme. Physical examination shows poor posture, unkempt hair, and that his shirt is dirty. He states that other patients with real diseases should be examined before him. He then tells the nurse that she looks like a character from his favorite medical television series, but that he doesn’t enjoy watching it anymore. He has poor eye contact. His family medical history is notable for his father’s suicide when the patient was 14. Which of the following is the most likely diagnosis?

A Borderline personality disorder
B Dysthymia
C Generalized anxiety disorder
D Major depressive disorder (MDD)
E Post-traumatic stress disorder
A

D Major depressive disorder (MDD)

Common symptoms of major depressive disorder include persistent low mood, anhedonia, sleep and appetite changes, and reduced concentration and energy. Family history, recent stress, or illness are risk factors for depression.

293
Q

A 12-year-old Iranian boy is brought to the pediatrician’s office because his mother found a suicide note under his bed. His medical history is relevant for post-traumatic stress disorder due to a terrorist attack 2 years ago. His family recently immigrated to the United States and he just started the seventh year in a new school. When asking the boy about his life at the new school, he bursts into tears claiming that all boys at school call him “Muzzie” and that he was recently called to detention, because someone framed him placing toy grenades and plastic guns under his desk. He claims none of his teachers believed his justification and now nobody wants to be friends with him. A 20-item Child Depression Inventory reveals a score equivalent to severe depression. The physical exam is noncontributory. Which of the following is the most likely cause of this patient’s current condition?

A Recent immigration to a new country  
B Major depressive disorder  
C Bullying  
D His mother finding the suicide note  
E History of post-traumatic stress disorder 
F New school adaptation
A

C Bullying

Bullying is defined as the use of force, threat, or coercion to abuse, intimidate, or aggressively dominate others. Victims of bullying are prone to internalize bad situations, therefore, these children are more likely to develop severe depression, suicidal ideations, and even death, if it’s not detected on time

294
Q

A 45-year old man comes to the office with his wife because she says that he has been snoring excessively in the past few weeks. He says that he has felt more tired than usual during the day. The patient is not on any medications. Laboratory results are within the reference range for TSH, LFT, and CBC with differential, ESR, and ferritin. HIV and PPD tests are negative. A nocturnal polysomnography test is negative and his symptoms improve with rest. Which of the following is the most likely cause of his symptoms?

A Obstructive sleep apnea
B Central sleep apnea
C Chronic inflammatory condition
D Psychological condition
E Anemia
A

D Psychological condition

Obstructive sleep apnea is not likely in this patient, since the nocturnal polysomnograph is negative. However, it can result in daytime sleepiness or fatigue.

Central sleep apnea is not likely in this patient, since the nocturnal polysomnograph is negative. Most cases of central sleep apnea are associated with a medical disorder (such as heart failure) or medications.

Fatigue may be caused by psychological or psychiatric issues such as substance abuse, a personality disorder, physical or emotional abuse, anxiety, and depression.

295
Q

A 45-year-old woman comes to the clinic because of a depressed mood ever since her college daughter moved out of the house for school last month. She states that she briefly cheers up when her daughter visits her. The patient admits to significant weight gain recently. She sleeps for 13 hours a night, but still wakes up tired. Which of the following is the most likely diagnosis

A Atypical depression
B Major depressive disorder
C Major depressive disorder with seasonal pattern (seasonal affective disorder)
D Melancholic depression
E Persistent depressive disorder (dysthymia)

A

A Atypical depression

Atypical depression is characterized by mood reactivity (improved mood when something good happens) and at least two of the following: significant weight gain or increased appetite, hypersomnia, leaden paralysis,or excessive sensitivity to rejection. This patient reports hypersomnia, weight gain, and describes mood reactivity when her daughter is around.

296
Q

A 30-year-old woman, gravida 2, para 2, comes to the office because of rapidly changing moods, irritability, anxiety and tearfulness for 3 days. Her second daughter was born by spontaneous vaginal birth a week ago. Today, she cried whilst breastfeeding, her husband was very concerned. She has not been sleeping very well, and feels very exhausted. She says that she was emotional after her first pregnancy, but that resolved two weeks post-delivery. There is no history of depression, or other medical problems. She has still been able to take care of her children, even though she has been labile in her moods. Which of the following is the most likely diagnosis?

A Childbirth-related post traumatic stress disorder
B Normal postpartum mood
C Postpartum blues
D Postpartum depression
E Postpartum psychosis
A

C Postpartum blues

Postpartum blues is common with symptom onset several days after delivery and lasting up to 2 weeks. Typical symptoms include; mood lability, irritability, appetite changes and sleep disturbance. Postpartum depression is similar to postpartum blues, however symptoms are more persistent and cause functional impairment.

297
Q

A 29-year old man comes to the office for a wellness visit. He is a veteran who returned from war six months ago, and says that he has felt depressed since then. He vaguely describes a variety of symptoms including depressed mood, apathy, insomnia, loss of appetite, and thoughts about death. His PHQ score is 3. Which of the following is the most appropriate next step in management?

A Offer reassurance, otherwise no treatment needed
B Prescribe fluoxetine
C Prescribe fluoxetine and refer for cognitive behavioral therapy
D Refer for cognitive behavioral therapy
E Schedule a follow-up appointment for one month

A

A Offer reassurance, otherwise no treatment needed

PHQ-9 = 0-27
⦁ 0-4 = minimal depression = reassurance for this patient
⦁ 5-9 = mild depression = provide support + education, RTC in 1 month
⦁ 10-14 = mod depression = antidepressant or psychotherapy MAY be used
⦁ 15-19 = moderately severe depression = antidepressant or psychotherapy SHOULD be used
⦁ 20-27 = severe depression = antidepressant AND psychotherapy

B = for mod or moderately severe depression
C = severe depression
D = for mod or moderately severe depression
E = mild depression
A = minimal depression
298
Q

A 65-year-old woman comes to the office for evaluation of her depression. She has a history of major depressive disorder that is refractory to SSRIs, buspirone, mirtazapine, and trazodone. She denies homicidal ideations, but when asked about suicide, she states that she has thought of using a rope to hang herself. However, she says she would not do that because it is against her faith. She has family that lives in another state and has been living alone since her husband passed away 7 years ago. She begins a tricyclic antidepressant (TCA) on a limited dosing regimen and comes to the office for regular monitoring of its levels. Which of the following TCAs is most appropriate for this patient?

A Amitriptyline
B Clomipramine
C Imipramine
D Maprotiline
E Nortriptyline
A

E Nortriptyline

Nortriptyline is a tricyclic antidepressant (TCA) used for major depressive disorder. It is associated with fewer and milder side effects that other TCAs.

Clomipramine (Anafranil) = TCA used to treat OCD, also given to treat newborns with congenital heart defects

Amitriptyline is highly anticholinergic, more sedating, and thus the most lethal if overdosed. Because of the pronounced anticholinergic effects, it would not be a first choice tricyclic antidepressant (TCA) in an elderly patient.

Imipramine (Tofranil) = for a child that wets his or her bed. In addition to treating enuresis, imipramine is also indicated for the treatment of panic disorder.

Nortriptyline is the TCA least likely to cause orthostatic hypotension and so is indicated in the elderly patients. This is the only TCA with a therapeutic window. In other words, if the therapeutic amount is exceeded, or if the amount is less than indicated, then the drug will have no clinical effect! This is different than the therapeutic window for lithium, which means it works within the range, but is harmful outside the range

299
Q

A 65-year-old woman comes to the office because of sleep disturbances, lack of interest in her hobbies, poor concentration, fatigue, and depressed mood. She has a long-standing history of poorly controlled diabetes mellitus. She says that she has had a burning sensation in her feet for several years. Which of the following medications is most likely to improve her conditions?

A Duloxetine
B Fluoxetine
C Mirtazapine
D Paroxetine
E Phenelzine
A

Duloxetine is an SNRI used in the treatment of major depressive disorder. It has also shown to be clinically useful for the treatment of diabetic neuropathy.

Phenelzine = MAOI = only used when other meds have failed

Mirtazapine is an atypical antidepressant. It antagonizes α2 receptors, leading to increases in norepinephrine and serotonin levels. It can cause sedation, increased appetite, weight gain, and dry mouth. This side effect profile makes it useful for a depressed patient with a low BMI (e.g.: due to anorexia or malnourishment).

300
Q

A 65-year-old woman comes to the office because of sleep disturbances, lack of interest in her hobbies, poor concentration, fatigue, and depressed mood. She has a long-standing history of poorly controlled diabetes mellitus. She says that she has had a burning sensation in her feet for several years. Which of the following medications is most likely to improve her conditions?

A Duloxetine
B Fluoxetine
C Mirtazapine
D Paroxetine
E Phenelzine
A

Duloxetine is an SNRI used in the treatment of major depressive disorder. It has also shown to be clinically useful for the treatment of diabetic neuropathy.

Phenelzine = MAOI = only used when other meds have failed

Mirtazapine is an atypical antidepressant. It antagonizes α2 receptors, leading to increases in norepinephrine and serotonin levels. It can cause sedation, increased appetite, weight gain, and dry mouth. This side effect profile makes it useful for a depressed patient with a low BMI (e.g.: due to anorexia or malnourishment).

301
Q

A 67-year-old man comes to the office because of headaches and pollen allergy. He was diagnosed with depression three months ago and was prescribed sertraline as pharmacological treatment. He is wondering if he can take another medication to decrease the headache pain. Which of the following drugs should he avoid because of the significant risk of gastrointestinal adverse events from pharmacological interactions?

A Codeine
B Dihydroergotamine
C Ergotamine
D Diphenhydramine
E Ibuprofen
A

E Ibuprofen

The use of SSRIs in major depressive disorder with concurrent use of NSAIDs increases the risk of upper gastrointestinal bleeding. This increased combined toxicity is due to pharmacodynamic synergism.

Selective serotonin reuptake inhibitors (SSRIs) such as sertraline are associated with a mildly elevated risk of upper gastrointestinal bleeding. Ibuprofen is a nonsteroidal anti-inflammatory drug (NSAID) that increases GI adverse events, including bleeding, ulceration, and perforation. In combination, ibuprofen and sertraline increase each other’s toxicity by pharmacodynamic synergism.

302
Q

A 26-year old man comes to the office because of unexplained sadness, suicidal thoughts, and agitation. He appears preoccupied and engages in face-picking and nail biting. The symptoms have been ongoing, causing severe deterioration in the student’s grades at graduate school and motivation to participate in intramural sports, previously one of his favorite past-times. Which of the following medications is the most appropriate first-line treatment for this patient?

A Bupropion
B Levominalcipran
C Mirtazapine
D Sertraline
E Venlafaxine
A

D Sertraline

In the pharmacological management of major depressive disorder, a selective serotonin reuptake inhibitors such as sertraline and fluoxetine is the drug of choice.

303
Q

A 31-year-old woman comes to the office for a re-evaluation. Two weeks ago, she came to the office because of depressed mood, poor sleep, poor concentration, and diminished appetite. She was subsequently started on fluoxetine. She now says that she does not feel her symptoms are improving and asks for another medication to treat her depression. Which of the following is the most appropriate next step in management?

A Increase the dose of fluoxetine
B Reassure the patient and follow-up in 4 weeks
C Stop the fluoxetine and start another SSRI
D Stop the fluoxetine and start bupropion
E Stop the fluoxetine and start duloxetine

A

B Reassure the patient and follow-up in 4 weeks

Antidepressants, especially SSRIs (e.g. citalopram, fluoxetine, paroxetine, sertraline), can take up to 6 weeks to take full effect. It is therefore recommended that patients stay on one drug for at least 6 weeks before a change in dose or medication is considered. This patient should be reassured that the medication can take a while longer to reach peak levels in the blood and that she should give it 4 more weeks to take effect. An adequate trial of antidepressant therapy is 4–6 weeks. Therefore, the best step in management in this case, is to reassure the patient and follow-up in 4 weeks to see if her symptoms have improved. If an individual does not respond to one specific SSRI, it is recommended to try another in the same class before moving to a medication with a different mechanism of action.

SSRI dosage or regime is generally not changed until one agent is given for at least 6-8 weeks.

304
Q

A 34-year-old woman comes to the office because she has persistent thoughts about overdosing on her medication. She has a history of depression that has not responded to pharmacological therapy. She agrees to a trial of electroconvulsive therapy. Which of the following prophylactic medications would be most likely to prevent the most common side effect of this procedure?

A Cefuroxime
B Ketorolac
C Lamotrigine  
D Ondansetron 
E Sertraline
A

B Ketorolac

When ECT is used to treat major depressive disorder, the most common medical adverse effect is headache. It is typically managed with acetaminophen given after the treatment or prophylactic IV ketorolac.

Adverse effects from ECT are divided into medical or cognitive categories. The most common medical adverse effect is headache. Patients should be told to expect a headache after each treatment. It is typically managed with acetaminophen given after the treatment. If headaches are severe, prophylactic treatment with IV ketorolac is acceptable. Ketorolac is a non-steroidal anti-inflammatory drug indicated for short-term management of moderate to severe pain.

305
Q

A 15-year-old boy comes to the office because of severe acne that has not responded to over-the-counter treatments. The patient tearfully states that he feels so embarrassed by his skin condition that he has avoided leaving the house and has started to skip school. He was formerly a top student, and he says that he has begun to fail classes, withdraw from his family and friends, and lost weight. Which of the following is the most appropriate response?

A “Do you think that your family is worried about you?”
B “It sounds like you are depressed; would you consider seeing a psychiatrist and/or counselor?”
C “Many people suffer from acne; it is nothing to be embarrassed about.”
D “We will treat your acne aggressively and that should help your other problems as well.”
E “While we will treat your skin condition, why don’t we talk a little bit more about the problems you’re having at school and in your social life.”
F “Your acne really doesn’t look that bad; it probably seems worse to you than it really is.”

A

E “While we will treat your skin condition, why don’t we talk a little bit more about the problems you’re having at school and in your social life.”

Depression is the most common mental health condition seen in primary care, although few patients discuss these symptoms directly. Signs such as social withdrawal, weight loss, and sad affect should prompt further questioning about the patient’s mental health status.

306
Q

A 72-year-old man is brought into the office by his wife because he’s been refusing to eat food for a week, because he feels severely depressed and wants to end his life. He tried to commit suicide twice in the past week, both times by running into oncoming traffic, but wasn’t physically injured either time. His symptoms have not been relieved by maximum doses of numerous medications including sertraline, citalopram, and he recently started phenelzine, and does not want to take any new medications. He’s had multiple crying spells throughout the day because he believes that his wife and children are all plotting to kill him. He has suicidal thoughts and says that he has a gun in his home. His past medical history is notable for having had two prior myocardial infarctions, and having poorly controlled hypertension. Which of the following is the most appropriate next step in management?

A Amitriptyline
B Ketamine
C Transcranial magnetic stimulation
D Electroconvulsive therapy
E Fluoxetine
A

D Electroconvulsive therapy

The primary indication for electroconvulsive therapy is severe major depression that is life threatening. ECT is more effective than antidepressants alone, and is particularly useful in patients with psychotic features and among the elderly.

307
Q

A 35-year-old man comes to the clinic because of continued depression despite a recent trial of citalopram. He has a history of major depressive disorder. He says that he does feel slightly improved, but has not returned to baseline. However, this is the first anti-depressant that has not caused any adverse effects. The physician decides to augment the therapy with another medication, which of the following is the most appropriate additional agent?

A Duloxetine     
B Isocarboxazid     
C Lithium     
D Pimozide     
E St. John wort
A

C Lithium

In patients with treatment-resistant major depressive disorder, lithium may be used as an augmenting agent in combination with a selective serotonin reuptake inhibitor.

Lithium may be necessary to support the remission of depression and may be added to the treatment regimen in a patient population that does not respond to monotherapy. Although lithium is commonly associated with the treatment of bipolar disorder, it is also used as an augmenting agent in the treatment of resistant depression. Additionally, it can be used as maintenance therapy to prevent recurrence of depression. Lithium alters cation transport in nerve and muscle cells and influences serotonin and/or norepinephrine reuptake.

Contraindications to lithium therapy include renal impairment, sodium depletion, dehydration, and severe cardiovascular disease. When using this therapy, clinicians must monitor lithium levels, renal function tests, and thyroid levels regularly.

308
Q

Which one of the following is a symptom of depression commonly seen in teens?

a. Always seeming to be on the go
b. Increased self esteem
c. Social withdrawal
d. Hallucinations

A

c. Social withdrawal

309
Q

A 10-year-old girl is brought in for evaluation of symptoms of recurrent headaches, abdominal pain and nausea for the last month. These symptoms seem to occur before school or other social activities and last minutes to hours and have no provoking or alleviating factors. Her parents report she struggles to pay attention in class and is irritable and often crying. Physical exam and vital signs are unremarkable. Which of the following is the most likely diagnosis?

a. Depression
b. Bipolar disorder
c. Schizophrenia
d. Social anxiety disorder

A

a. Depression

310
Q

Which one of the following history components carries the highest risk of suicide in a patient with depression?

a. 1st degree relative who died by suicide
b. Substance abuse disorder
c. Intrusive thoughts of dying
d. Lack of a significant other

A

a. 1st degree relative who died by suicide

311
Q

A 45-year-old woman was recently diagnosed with migraine headaches and prescribed a triptan. Medical history is significant for depression treated presently with an SSRI. What drug interaction should she be educated about with this combination?

a. Tardive dyskinesia
b. Seizure
c. Serotonin syndrome
d. SIADH
e. Thyrotoxicosis

A

c. Serotonin syndrome

312
Q

A 9-year-old male is diagnosed with depression. Which of the following is the treatment of choice?

a. Abilify (Aripiprazole)
b. Escitalopram (Lexapro)
c. Imipramine (Tofranil)
d. Fluoxetine (Prozac)

A

d. Fluoxetine (Prozac)

313
Q

A 17-year-old girl is diagnosed with depression. Her predominate symptoms include hypersomnia, fatigue and depressed mood. Physical examination is unremarkable. Which of the following diagnostic tests is most indicated in the search for a medical etiology of her symptoms?

a. RPR and chlamydia testing
b. Chest x-ray
c. CT scan of the head
d. TSH, CBC

A

d. TSH, CBC

314
Q

A 28-year-old woman is brought in to the ED by her roommate. The patient has refused to go to her job for the last week and has not showered or eaten anything for several days. She admits to hearing a voice telling her that someone is trying to poison her with food and the shower water also contains poison.. Past medical history reveals an episode of major depression 2 years ago which has resolved. Which of the following is the most likely diagnosis?

a. Schizoaffective disorder
b. Manic episode
c. Brief psychotic disorder
d. Depression with psychosis

A

a. Schizoaffective disorder

315
Q

The criteria for major depressive disorder requires the symptoms of depression to have been present for which of the following time periods?

a. Two weeks
b. Two months
c. More than one year
d. One year

A

a. Two weeks

316
Q

When is the risk of suicide the greatest?

a. While the patient is in treatment
b. During the first 90 days after depression begins to lift
c. After 4 months of treatment
d. Depths of depression

A

b. During the first 90 days after depression begins to lift

317
Q

A 70-year-old woman with colon cancer presently undergoing chemotherapy comes in with symptoms consistent with depression and insomnia. She has trouble keeping weight on due to the nausea associated with her chemotherapy. Which of the following is the treatment of choice for her symptoms?

a. Mirtazapine (Remeron)
b. Nortriptyline (Pamelor)
c. Citalopram (Celexa)
d. Bupropion (Wellbutrin)

A

a. Mirtazapine (Remeron)

318
Q

A 45 year-old man comes in to the clinic with symptoms of depression for the last several months. He admits to having suicidal thoughts but does not have a plan or access to lethal means. He denies homicidal ideations and says “I would never kill myself”. In addition to pharmacologic therapy what else should be done for/with this patient at this time?

a. Inpatient psychiatric hospital admission
b. Have him live with his sister until his depression clears
c. Complete a suicide safety plan
d. Discourage him from driving

A

c. Complete a suicide safety plan

319
Q

A patient comes in to the office and is diagnosed with depression. She is also a smoker who wants to quit. Which one of the following antidepressant medications would be the best choice for this patient?

a. Nortriptyline (Pamelor)
b. Trazodone (Desyrel)
c. Fluoxetine (Prozac)
d. Bupropion (Wellbutrin)

A

d. Bupropion (Wellbutrin)

320
Q

A 60-year-old man suffers from depression and severe diabetic neuropathy. Which of the following is the treatment of choice for this patient?

a. Duloxetine (Cymbalta)
b. Sertraline (Zoloft)
c. Fluoxetine (Prozac)
d. Citalopram (Celexa)
e. Paroxetine (Paxil)

A

a. Duloxetine (Cymbalta)

321
Q

A 25-year-old woman comes in with persistent symptoms of recurrent generalized abdominal pain, urinary difficulties, and intermittent fever for the last 8 months. Review of her records reveals 20 clinic appointments for this complaint over the last 6 months. Despite an extensive workup including consultations with gastrointestinal and genitourinary specialists, no etiology for her symptoms has been identified. Her past medical history includes generalized anxiety disorder, depression, and a suicide attempt one year ago. Which of the following is the most likely diagnosis?

a. Munchausen syndrome by proxy
b. Phobic disorder
c. Panic disorder
d. Somatic symptom disorder
e. Conversion disorder

A

d. Somatic symptom disorder

322
Q

A 25-year-old female comes to the office because of recurrent sudden episodes of palpitations, dyspnea, accompanied by a sense of terror and impending doom, and a feeling of loss of control. These episodes have occurred about twice a month for the past 9 months and now she is almost constantly afraid that they will happen again. Examination and laboratory investigations show no abnormalities. Which of the following is the most likely diagnosis?

A Hypochondriasis
B Panic disorder
C Pheochromocytoma
D Hypothyroidism
E Hyperthyroidism
A

B Panic disorder

Panic disorder is characterized by the spontaneous and unexpected occurrence of panic attacks accompanied by at least one month of fear of another panic attack. Panic attacks are defined as a period of intense fear in which anxiety symptoms develop suddenly and usually peak in less than 10 minutes.

323
Q

A 28-year-old man comes to the office because of a new symptom he has been experiencing during his panic attacks. He was diagnosed with panic disorder two months ago and is following treatment. However, his panic attacks did not subside and now he feels like the world is drifting away and he is somehow separate from himself and his emotions. Which of the following is the patient experiencing during his panic attacks?

A Delusions
B Depersonalization
C Derealization
D Hallucinations
E Illusions
A

B Depersonalization

Depersonalization is an alteration in self-awareness. It can consist of a reality or detachment within the self, regarding one’s mind or body, or being a detached observer of oneself. It can occur during panic attacks, sleep deprivation, migraines, and temporal lobe epilepsy.

Derealization is an alteration in the perception or experience of the external world, making it seem unreal. The patient may feel that their environment is lacking in spontaneity, emotional coloring, and depth. It is a dissociative symptom that may occur for example in temporal lobe epilepsy, migraines, and head injuries.

The common symptoms of a panic attack include palpitations, tachycardia, diaphoresis, tremor, shortness of breath, chest pain or discomfort, nausea, abdominal discomfort, dizziness, lightheadedness, paresthesias, derealization or depersonalization, fear of losing control or “going crazy,” and fear of dying.

324
Q

A 25-year-old man comes to the office because of recurrent attacks of chest pain for six months. He says that the attacks are random and are associated with sweating, dizziness, and a general sense of doom. He is a college student and states that his symptoms started while he was sitting in a classroom. He has stopped going to class for the past month because he is afraid an attack will happen again and escape will be difficult. He denies the use of drugs or alcohol. Examination and laboratory tests are all within normal limits. Which of the following is the most likely diagnosis?

A Malingering
B Hypochondriasis
C Brief psychotic disorder
D Panic disorder
E Factitious disorder
A

panic disorder

  • chest pain
  • sweating
  • dizziness
  • fear of dying / death
afraid an attack will happen again
recurrent attacks (need 2+)

D Panic disorder

Panic disorder is characterized by the spontaneous and unexpected occurrence of panic attacks accompanied by at least one month of fear of another panic attack. Panic attacks are defined as a period of intense fear in which anxiety symptoms develop suddenly and usually peak in less than 10 minutes.

325
Q

A 52-year old man comes to the office because of bouts of tachycardia accompanied by a sense of impending doom for seven months. He says he has been concerned about these unpredictable attacks for the last two months and that he cannot think of anything that could have started them. These episodes begin as palpitations and progress to pounding tachycardia. They are accompanied by cold sweats, tremor, dizziness, and the feeling that he is going to die. Serum catecholamines and cardiac enzymes are within normal limits, as are his heart rate and blood pressure, and a week-long trial on a Holter monitor shows no evidence of arrhythmias. The patient works as a full-time yoga instructor and lives with his wife and young child. Which of the following is the most likely diagnosis?

A generalized anxiety disorder
B hypochondriasis
C acute stress disorder
D panic disorder
E specific phobia
A

D panic disorder

  • tachycardia
  • fear of impending doom
  • 7 months (> 1 month)

palpitations, tachycardia, sweating, trembling, dizziness, feelings of death/dying

Panic disorder is characterized by the spontaneous and unexpected occurrence of panic attacks, the frequency of which can vary from several attacks per day to only a few attacks per year. Panic attacks are defined as a period of intense fear in which symptoms develop abruptly and peak rapidly <10 minutes from symptom onset.

Although panic attacks can occur in other anxiety disorders, they occur without a discernible cause in panic disorder, as opposed to other disorders such as specific phobias. The DSM-5 criteria for panic disorder are: panic attacks associated with longer than one month of subsequent persistent worry about having another attack or consequences of the attack, or significant maladaptive behavioral changes related to the attack.

326
Q

A 35-year-old woman comes to the emergency department because of acute onset of difficulty breathing that started 30 minutes before when she was walking in the park. It first started as tingling in her arms which quickly progressed to include difficulty breathing, nausea, dizziness and stabbing right-sided chest pain that does not change with inspiration. She reports having had three episodes like this in the past 6 months, all while walking around the city. She consequently avoids spending a lot of time walking outside in fear of future recurrences. Most of her symptoms have subsided by the time of the evaluation. Vital signs are notable for tachycardia to 105/min. Physical examination is normal. She has no personal or family history of cardiovascular or pulmonary disease. Her echocardiogram is normal, D-dimer is negative, and troponins remain normal after 6 hours. Which of the following is the most likely diagnosis?

A Generalized anxiety disorder
B Myocardial infarction
C Panic disorder
D Pericarditis
E Pulmonary embolism
A

C Panic disorder

Panic disorder is characterized by recurrent, unexpected episodes of autonomic arousal and somatic symptoms, combined with a fear of future episodes and a feeling of “going crazy.” Medical causes should be ruled out prior to giving a diagnosis of panic disorder.

paresthesias, SOB, nausea, dizziness, chest pain
30 minutes
fear of future recurrences

327
Q

A 17-year-old girl comes to the office because of recurrent episodes of palpitations, sweating, chills, nausea, dizziness, numbness in her extremities, and shortness of breath, accompanied by intense fear over the past 4 months. The episodes have been triggered by various things like seeing an old middle school friend in one instance, and crossing the street in another instance. She has also experienced episodes that don’t seem to be triggered by anything at all. She is a model and her symptoms started after she was voted off from a very popular television show. Since then, she’s started avoiding the television, because it reminds her of the show, and it makes her feel rejected. Which of the following is the most likely diagnosis?

A Bipolar disorder type II
B Cyclothymia
C Generalized anxiety disorder
D Panic disorder
E Specific phobia
A

D Panic disorder

Panic disorder is characterized by recurrent spontaneous panic attacks without an obvious cause with at least one month of persistent fear of experiencing another attack or significant behavioral changes related to the attack.

328
Q

A 50-year-old woman describes a recent incident in which she was in a bus and began experiencing tachycardia and feelings of impending doom. They began as palpitations and progressed to pounding tachycardia. She has had multiple similar episodes over the past year. They are accompanied by cold sweats, trembling, dizziness, and the feeling that she will die on the bus either by heart attack or motor vehicle accident. In the past she has been prescribed fluoxetine, escitalopram, and most recently duloxetine, without relief of her symptoms. Her examination and laboratory investigations are within normal limits. The woman works as a full-time middle school teacher and lives with her husband and two boys. Which of the following is the most potent medication that can be used to treat this patient’s condition?

A Chlordiazepoxide
B Clonazepam
C Alprazolam
D Flurazepam
E Lorazepam
A

tachycardia, palpitations, impending doom, sweats, trembling, dizziness, feelings of death = panic attack

acute treatment = Benzo’s

clonazepam = klonopin
alprazolam = Xanax
lorazepam = Ativan

Clonazepam and triazolam are the most potent benzodiazepines. Knowledge of potency and equivalence between dosages of drugs in this class is important when prescribing benzodiazepines, so that under- or overdosages do not occur.

Clonazepam and triazolam are the most potent benzodiazepines, followed by alprazolam and lorazepam

Lorazepam is a benzodiazepine of intermediate potency

329
Q

A 26-year-old woman comes to the emergency department because of heart palpitations, profuse sweating, difficulty breathing, and chest pain. She has had these episodes infrequently over the past 3 months and is fearful of future episodes. She has no history of drinking alcohol, smoking tobacco, or using illicit drugs. She also reports having no allergies and currently takes no medications. Vital signs show her temperature is 37.2°C (99°F) , pulse is 120/min, respirations are 25/min, and blood pressure is 130/70 mm Hg. Oxygen saturation is 98% on room air. Physical examination shows the patient is sweating profusely but has a normal lung, heart, and neurological exam. An ECG shows no abnormalities. Which of the following is the most appropriate initial treatment option for this patient?

A Sertraline
B Lorazepam
C Nitroglycerin 
D Cognitive behavioral therapy
E Education and reassurance
A

panic disorder

E Education and reassurance….not a benzo…

Benzodiazepines are a useful option in patients with acute severe panic attack but should be used after a trial of explanation and reassurance about the condition.

Panic attacks are best treated with education and reassurance. Benzodiazepines can be useful in acute management but are not indicated in all patients. Longer-term management of panic disorder is best achieved with cognitive behavioral therapy with or without pharmacotherapy where SSRIs are first-line.

330
Q

A 24-year-old man enters a psychiatric rehab facility after being arrested on a felony assault charge. His aggression and erratic behavior are ascribed to bipolar disorder and post-traumatic stress disorder (PTSD) from childhood trauma. He reports to having terrible dreams and trouble sleeping. He is prescribed prazosin to help treat his nightmares. One of the adverse effects associated with this drug is orthostatic hypotension. What is the mechanism of this drug?

A Adrenergic antagonist with α1 receptor selectivity
B Adrenergic agonist with α2 receptor selectivity
C Adrenergicantagonist with α2 receptor selectivity
D Non-selective α-adrenergic antagonist
E Serotonin Selective Reuptake Inhibitor

A

prazosin = alpha-1 blocker

A Adrenergic antagonist with α1 receptor selectivity

Post-traumatic stress disorder (PTSD) is an anxiety disorder that may occur after traumatic events, and is characterized by intrusive thoughts, nightmares, and flashbacks of these events. Selective serotonin reuptake inhibitors (SSRIs) are first-line choices to treat PTSD. However, prazosin, a specific α1-adrenergic antagonist, can be used to treat persistent nightmares.

331
Q

A 24-year old man comes to the clinic because of mood changes that his mother was concerned about. He was on a date at a restaurant when a drive-by shooting occurred outside the front door. For the next three weeks, he could not get the scene out of his mind, with random flashbacks to the noises of the gunshots ringing in his head. He often rushed to the window whenever he heard noise outside. However, his mother would deny hearing anything. He has nightmares often, and began to avoid the restaurant and the young woman he had taken out on that date. Which of the following is the most likely diagnosis?

A Acute stress disorder
B Brief psychotic disorder
C Delusional disorder
D Panic Disorder
E Post-traumatic stress disorder
A

A Acute stress disorder

Acute stress disorder and post-traumatic stress disorder (PTSD) is characterized by the persistent intrusive thoughts relating to a past traumatic event. However, PTSD is diagnosed if the symptoms have lasted for at least one month.

The differentiating factor between acute stress disorder and PTSD is solely time. Acute stress disorder is diagnosed if the duration of symptoms has been less than a month. If the symptoms last longer than a month, it is known as PTSD.

332
Q

A 45-year-old woman comes to the clinic for a follow-up appointment three weeks after she was raped in a park near their home. She reports that ever since the event, she has stopped walking their dog in the park, will not leave the home after dark, and has vivid nightmares recounting the event. Her husband adds that she pulls away from him anytime he attempts to be intimate with her. Which of the following is the most likely diagnosis?

A Post-traumatic stress disorder     
B Acute stress disorder      
C Panic disorder     
D Brief psychotic disorder     
E Specific phobia
A

B Acute stress disorder

In the aftermath of a traumatic event (e.g. rape, motor vehicle accident), patients will often develop psychiatric symptoms that interfere with their daily lives. They have flashbacks, intrusive dreams and many fears that prevent them from living their lives as it was before. In this case, these symptoms have occurred for less than one month, so it is acute stress disorder.

333
Q

A 28-year-old woman comes to the clinic because of trouble sleeping due to intense nightmares for the past 2 months. She has no past significant medical history and currently takes no medications. She says that she has witnessed multiple deaths of close friends and that her own life has been threatened numerous times. She feels alone and has been distancing herself from her family and friends. She continues to experience fear and negative thoughts that something bad will happen. Which of the following is the best choice of medication to prescribe?

A Amitriptyline
B Lorazepam
C Modafinil
D Sertraline
E Zolpidem
A

past 2 months = PTSD
tx = SSRIs

D Sertraline

Selective serotonin reuptake inhibitors (SSRIs) are the first-line treatment for post-traumatic stress disorder. Only sertraline and paroxetine are FDA-approved to treat PTSD. **

334
Q

A 55-year-old retired European solider comes to the clinic because of an unusual sense of smelling burning gas for over a month. He would wake up and check the stove but finds it off. He had his home checked multiple times and no abnormalities were found. His wife also denies ever smelling burning gas at home. He has trouble sleeping because he often has nightmares about war. Other than these symptoms, he has no other complaints and has a healthy relationship with his family. From this limited history, which of the following is the most likely diagnosis?

A Dementia
B Psychosis
C Substance Abuse
D Post-Traumatic Stress Disorder
E Panic Disorder
A

D Post-Traumatic Stress Disorder

Post-traumatic Stress Disorder is the presence of recurrent, involuntary and distressing thoughts for longer than a month associated with past traumatic events. These intruding thoughts can come in the form of dreams, flashbacks, and avoidance of possible cues that resemble an aspect of the past trauma.

335
Q

A 25-year-old woman comes to the office with a six-week history of recurrent nightmares of when she was raped at knifepoint earlier this year. She now avoids situations where unknown men will be present, to the extent that she had to quit her job at the bank. She reports being “jumpy” anytime she hears footsteps behind her. Which of the following is the most likely diagnosis?

A Acute stress disorder
B Adjustment disorder with anxiety
C Panic Disorder
D Post-traumatic stress disorder
E Specific phobia
A

D Post-traumatic stress disorder

This woman has post-traumatic stress disorder (PTSD). She has been exposed to a situation involving serious injury or threat - being raped at knifepoint. She is persistently re-experiencing the event in the form of nightmares and has strong physiologic responses (feeling “jumpy”). Furthermore, she repeatedly avoids reminders of the traumatic event including places where unfamiliar men might be. These are all classical symptoms of someone that may be experiencing PTSD. It is very likely that these symptoms cause significant distress, especially because she is seeking medical help. The symptoms have been going on longer than a month, which is what differentiates acute stress disorder and PTSD apart.

336
Q

A 30-year-old woman was attacked, held at gunpoint, robbed, and beaten after leaving a restaurant in the late evening. Despite this, she managed to report the incident to police and continue with her daily activities. Two months later, she comes to the psychiatric office because she has been having difficulty going to work and participating in other activities. Which of the following constellation of symptoms would she most likely report to her psychiatrist?

A Depression and suicidal thoughts    
B Flashbacks and increased arousal    
C Confusion and disorientation    
D Hyperphagia and hypersomnia    
E Euphoria and racing thoughts
A

PTSD

B Flashbacks and increased arousal

A = depression
C = delirium / dementia / psychosis
D = atypical depression
E = mania
337
Q

A 23-year-old woman comes to the clinic because of insomnia for three months. Her medical history is noncontributory and she currently takes no medications. She says that she has nightmares and flashbacks to the death of one of her close friends. She often feels alone and has been distancing herself from close friends who also share some of these experiences. She continues to experience fear and negative thoughts that something bad will happen. She appears anxious and startles easily. Which of the following is the most likely diagnosis?

A Acute stress disorder
B Adjustment disorder with anxiety
C Brief psychotic disorder
D Major depressive disorder
E Post-traumatic stress disorder
A

E Post-traumatic stress disorder

Post-traumatic stress disorder may be seen in patients after exposure to severe traumatic events and manifests as flashbacks, nightmares, hyperarousal, and patterns of avoidance. The duration of symptoms must be at least a month to meet diagnostic criteria.

338
Q

A 25-year-old soldier presents to your office stating that he has been having anxiety since returning from the war many months ago. The patient reports recurring nightmares of an improvised explosive device that injured his squad while on patrol. The patient also complains of restlessness and irritability around family members, and he has been avoiding his fellow comrades outside of the office. He would like to try something to get over this. Which is the most appropriate pharmacologic therapy in this patient?

A amitriptyline     
B Paroxetine 
C phenelzine     
D lorazepam     
E risperidone
A

B Paroxetine

Paroxetine and Sertraline are the only SSRI’s used to treat PTSD and they are 1st line!

339
Q

A 50-year-old woman is brought in by her spouse with concerns that she rarely goes out of the house, especially alone. She reports that even when contemplating going to the grocery store, she becomes terrified and at small gatherings may experience dizziness, SOB, and trembling and feels she will have difficulty “escaping” should the need arise. Which of the following is the most likely diagnosis?

a. Agoraphobia
b. PTSD
c. Generalized anxiety disorder
d. Social phobia

A

a. Agoraphobia

340
Q

Which of the following disorders is characterized by a triad of painful re-experiencing, patterns of avoidance, and hyperarousal?

a. Obsessive compulsive disorder
b. Generalized anxiety disorder
c. Panic attacks
d. Post traumatic stress disorder

A

d. Post traumatic stress disorder

341
Q

A 50-year-old woman admitted to the surgical floor post partial colon resection has a fever of 39°C (102.2°F). Home medications include: simvastatin (Zocor), metoprolol (Toprol XL), venlafaxine (Effexor) and trazodone (Desyrel) daily. Blood pressure is 180/100 mmHg, heart rate is 140/min and regular. Physical exam reveals a tremor of both her hands, stiff muscles and ankle clonus. She appears restless, anxious and angry. The nurse reports administration of acetaminophen (Tylenol) for her fever an hour ago and it hasn’t touched it. In addition the patient has been receiving IV fentanyl for pain management and ondansetron (Zofran) for post surgical nausea. Which of the following is the most appropriate initial intervention in this patient?

a. Administer cyproheptadine
b. Administer acetaminophen (Tylenol)
c. Administer IV lorazepam
d. Discontinue the offending medications
e. Increase metoprolol (Toprol XL)

A

d. Discontinue the offending medications

serotonin syndrome

. If no improvement after discontinuation of the offending medications the next step is to administer lorazepam. If no improvement or worsening symptoms then the patient should receive cyproheptadine.

342
Q

A 17-year old Japanese-American boy comes to the office because of “taijin kyofusho” for the past 10 months. He explains that taijin kyofusho is a condition where “his gaze upsets people so they look away and avoid him.” His mother expresses confusion, but support, saying that Japanese culture is very important to her son and that recently he has found himself withdrawn from his American high school peers and immersed in texts about Japanese culture. The boy avoids contact with people outside his family and seems very downcast and concerned that no one will understand or like him. He doesn’t have any hallucinations or any precipitating trauma. Which of the following is the most appropriate diagnosis?

A Factitious disorder
B Separation anxiety disorder
C Social anxiety disorder
D Somatic symptom disorder
E Specific phobia
A

C Social anxiety disorder

Taijin kyofusho is described in the DSM-V as a culture-related diagnostic issue associated with social anxiety disorder. It is characterized by a fear of social judgment or judgment in social situations.

343
Q

A 15-year old girl with type 1 diabetes mellitus and hypothyroidism comes to the emergency department because of sweating, shaking, and palpitations. One hour earlier, she had been standing in front of her peers preparing to deliver a speech for a class assignment. She states that she suddenly started to fear that she would make a mistake and her classmates would start laughing at her. Onset of symptoms began shortly after. A thyroid-stimulating hormone (TSH) level is measured at 4.0 µU/mL, and a blood glucose level is measured at 90 mg/dL. Which of the following is the most likely diagnosis?

A Diabetic ketoacidosis     
B Factitious hyperthyroidism     
C Hypoglycemic event     
D Panic disorder      
E Social phobia
A

E Social phobia

Social phobia, also known as social anxiety disorder, is defined as a marked fear of one or more social or performance situations involving exposure to unfamiliar people or possible scrutiny by others. Persons fear being humiliated or embarrassed. Exposure to certain feared situations, like public speaking, often invokes an autonomic response leading to tachycardia, diaphoresis, and tremors. Triggers are usually avoided in the future or performed with distress, and it interferes with normal routines. The person recognizes the fear is excessive or unreasonable, but this may be absent in children. In individuals under the age of 18, the behavior pattern must be present for a minimum of 6 months and not secondary to drug abuse or other medical conditions. However, in light of the choices presented, social phobia is the most appropriate diagnosis.

According to the American Diabetic Association, a self-monitored blood glucose ≤70 mg/dL in diabetics should raise concern for hypoglycemia.

344
Q

A 14-year old boy comes to the office because of anxiety when using public restrooms for the past six months. He has to go home from school everyday at lunch to use the toilet. If he tries to enter a public restroom he experiences anxiety and is not able to perform his bodily functions. He fears someone may walk in, and his fear of embarrassment puts too much pressure on him. He does not consider his fear to be excessive. Which of the following is the most likely diagnosis?

A Adjustment disorder
B Agoraphobia
C Delusional disorder
D Social anxiety disorder
E Specific phobia
A

D Social anxiety disorder

Social anxiety disorder (SAD), also known as social phobia, results from disproportionate fears of scrutiny, embarrassment, and humiliation in social situations. The illness usually begins in childhood or adolescence. In situations that cause symptoms, patients may exhibit blushing, sweating, trembling, and palpitations. They may experience panic attacks in these situations as well. Individuals with SAD often recognize their irrational level of anxiety and fear that others will notice. However, this awareness is not necessary for proper diagnosis. Individuals will often avoid situations that cause such anxiety and may become socially withdrawn as a result. SAD is diagnosed when symptoms persist for six months or more.

Specific phobia is characterized by a marked fear of an object or situation that leads to avoidance of the cause of fear. The principal difference between specific phobia and social anxiety disorder centers on the focus of the fear

345
Q

A 12-year-old girl comes to the pediatrician’s office with her mother because of an intense fear of leaving the house for the past year. The patient reports that she has fears over leaving the house alone and refuses to take the public school bus because “the other kids think that I’m weird.” Her mother says that her daughter never goes out to play with the other kids or even goes out to the park when it’s relatively empty. The patient says that she “doesn’t like wide spaces,” and denies any bullying at school. Her grades are suffering as a result of this problem. Which of the following is the most likely diagnosis?

A Agoraphobia
B Post traumatic stress disorder 
C Separation anxiety disorder
D Social anxiety disorder
E Specific phobia
A

A Agoraphobia

Agoraphobia is an anxiety disorder characterized by an irrational and persistent fear in response to situations where escape or help may become difficult. Hence, these patients experience these symptoms when using public transportation, being in open or enclosed spaces, standing in line a crowd or leaving the home alone. This fear is out of proportion and these patients will often try to avoid these situations. As a result, they experience severe functional, occupational, and social impairment. These symptoms must last for at least 6 months. Treatment involves cognitive behavioral therapy. Medications for agoraphobia include SSRIs and benzodiazepines

Social anxiety disorder, also known as social phobia, is defined as an individual who has intense fear and anxiety in social situations. There is a subset of social anxiety disorder known as a specific social phobia. These patients will experience palpitations and anxiety during events such as presentations or examinations.

346
Q

A 30-year-old woman comes to the office because of unbearable anxiety when thinking about leaving her home alone. This anxiety has persisted over the past year. She says that she has been trying to avoid leaving the home because she worries that she might have a panic attack in public. She states that her anxiety has severely impacted her social life and her ability to find work. Which of the following is the most appropriate treatment?

A Benzodiazepine administration
B Combination of a benzodiazepine and cognitive behavioral therapy
C Combination of a selective serotonin reuptake inhibitor, exposure therapy, relaxation, and breathing retraining.
D Inpatient treatment with a selective serotonin reuptake inhibitor and cognitive behavioral therapy
E Social skills training

A

6 months
anxiety about leaving the house
worried about having a panic attack in public
= AGORAPHOBIA

C Combination of a selective serotonin reuptake inhibitor, exposure therapy, relaxation, and breathing retraining.

The best treatment for agoraphobia is a combination of a selective serotonin reuptake inhibitor (SSRI), exposure therapy, relaxation, and breathing training

347
Q

A 20-year-old man comes to the office because of anxiety. He says the anxiety causes him to avoid public spaces, most specifically his university classes. He feels intimidated because his classes are held in large lecture halls with hundreds of students present. He has completely stopped attending his classes for the past 2 months. He previously attended classes somewhat regularly in his first year of college, when he had a friend who would walk with him to class. However, he no longer shares any of the same classes with that friend this semester and is afraid of going to class alone. He denies alcohol, tobacco, or recreational drug use. Physical examination shows no abnormalities. Which of the following is the most likely diagnosis?

A Agoraphobia
B Generalized anxiety disorder
C Panic disorder
D Posttraumatic stress disorder
E Specific phobia
A

AGORAPHOBIA

Agoraphobia is a psychiatric disorder that results from anxiety regarding and avoidance of situations such as crowds, shopping malls, public transportation, and being away from home alone. Occurrence of actual panic attacks may be low. The primary source of anxiety tends to remain the same for all situations. Anxiety usually stems from a fear of developing incapacitating or embarrassing symptoms, without an easy means to escape the situation or receive help. Individuals may establish a “safe zone” within which they feel comfortable or engage in “safety behaviors” to reduce anxiety in situations in which they would otherwise feel uncomfortable. Safety behaviors include being accompanied by a familiar person, carrying a comforting item, or staying near the exit. Risk factors for agoraphobia include younger age, female gender, and the presence of panic disorder or other phobias.

348
Q

A 28-year-old man comes to the office because of fear of leaving his home or walking on the street for the past five years. He avoids going outside, feels panic when he does, and asks for transportation from friends and family wherever he goes because he feels really anxious when using public transportation. Which of the following disorders is the most appropriate diagnosis?

A Agoraphobia
B Depression
C Panic disorder
D Separation anxiety disorder
E Specific phobia, situational type
A

A Agoraphobia

349
Q

A 24-year-old man comes to the clinic because of his fear of spiders. He states that he gets severe anxiety when he sees or thinks about spiders. As a result, he has not ventured into his garage for years. He feels foolish and that his fear is irrational, but he cannot overcome it. It has meant that he also has said no to camping trips and other social opportunities. He is otherwise in good health. He uses no medications or recreational drugs, Physical examination is unremarkable. Which of the following is the most likely diagnosis?

A Agoraphobia
B Panic disorder
C Specific phobia
D Generalized anxiety disorder
E Social anxiety disorder
A

C Specific phobia

A specific phobia is an anxiety disorder characterized by an intense fear and related avoidance of an object, situation, or place. The fear is out of proportion to the actual danger posed by the trigger and the patient recognizes that their fear is irrational. Exposure to the trigger leads to a severe emotional response and distress.

For the diagnosis of a phobia, the avoidance and fear must have a significant impact on usual routine, activities, or relationships.

Specific phobias are classified into 5 groups: animals, natural environment, blood-injection-injury, situational, and other. This patient has a specific phobia for spiders, also known as arachnophobia.

Treatment for specific phobias involves cognitive behavioral therapy, and exposure therapy (virtual or real) to desensitize the individual.

350
Q

Which of the following is the most effective treatment for specific phobias?

a. Electroconvulsive therapy
b. Behavioral therapy
c. Benzodiazepines
d. SSRIs

A

b. Behavioral therapy

351
Q

Which of the following disorders is best described as a lasting and groundless fear of a specific object, activity, or situation?

a. Phobia disorder
b. Obsessive compulsive
c. Panic disorder
d. Generalized anxiety

A

a. Phobia disorder

352
Q

A 20-year-old female college student comes to the office because of altered behavior and mood. Following a positive pregnancy test two weeks ago, she began to experience symptoms of insomnia, problems with memory, loss of energy, inability to make decisions, and disorganized thinking. She denies any prescription or illicit drug use, but she does state that she has seen small trolls with green hair in her dormitory. She is prescribed medication and her symptoms begin to subside one week later. Which of the following is the most likely diagnosis?

A Brief psychotic disorder      
B Depersonalization disorder      
C Paranoid schizophrenia      
D Schizoaffective disorder      
E Schizophreniform disorder
A

hallucinations - psychosis
2 weeks
= either brief episode or schizoaffective, however mood + psychosis occurring at same time = not schizoaffective

also, patient just likely stressed / worried about pregnancy

A Brief psychotic disorder

Brief psychotic disorder can be distinguished from schizophreniform disorder and schizophrenia based on duration of symptoms. Brief psychotic disorder lasts for less than one month, schizophreniform disorder lasts one to less than six months, and schizophrenia lasts for longer than six months.

Brief psychotic disorder is a rare psychiatric disorder that is characterized by psychotic symptoms similar to schizophrenia that persist for less than one month. They are commonly related to stress, and these patients usually recover completely, particularly when the stressor is removed.

353
Q

A 36-year-old man comes to the office for a follow-up visit. Six months ago, he was brought to the emergency department by his wife because she found him covering the house in foil in attempts to protect his family from a foreign invasion. His visit to the emergency room occurred one week after the death of his son in a traffic accident. His speech at the time was disorganized and tangential. Toxicology screens were negative. He has no previously diagnosed psychiatric conditions. He received risperidone, which was tapered and discontinued after three weeks. Since then, the patient and his wife report no other similar incidents, and he has since returned to his normal work hours. Which of the following is the most likely diagnosis?

A Mania
B Schizophrenia
C Schizophreniform disorder
D Major depression with psychotic features
E Brief psychotic disorder
A

delusions - 6 months ago
disorganized / tangential speech
= PSYCHOSIS
occurred 1 week aft death of son = stressful event

3 weeks later = resolved with Risperidone

= E Brief psychotic disorder

Brief psychotic disorder is defined by the presence of psychotic symptom(s) with sudden onset and full remission within one month. Because duration of symptoms is a criterion for classification, diagnosis of brief psychotic disorder can only be confirmed in retrospect after time has elapsed and the patient has stabilized. Some patients may go on to meet criteria for other conditions. Stressful life events have been found to precipitate brief psychosis in some patients, exemplified by the death of the patient’s son in this vignette.

354
Q

A 20-year-old male college student comes to the emergency department because of behavioral changes for the past three weeks. His girlfriend states that he rarely leaves his bedroom, shows no emotion, and claims to be working on a project for the government, the details of which he refuses to share. She adds that he has placed black curtains over all the windows in the apartment so “the spies next door can not monitor his progress.” The patient denies any alcohol or drug use, mentions that the government has been communicating with him through a chip placed in his brain, and accuses the ED staff of trying to hinder his work. Which of the following is the most appropriate diagnosis?

A Brief psychotic disorder     
B Delusional disorder     
C Schizoaffective disorder     
D Schizophrenia     
E Schizophreniform disorder
A
3 weeks
delusions = PSYCHOSIS
< 1 month = brief psychotic disorder
1 month - 6 months = schizophreniform
6+ months = schizophrenia

A Brief psychotic disorder

Brief psychotic disorder refers to a psychiatric illness that is similar to schizophrenia and schizopheniform disorder with regard to clinical features. One feature of the psychosis present in all three disorders is hallucinations, or sensory perceptions that are not externally present. Delusions, or fixed, false beliefs, may be present and may be bizarre or non-bizarre. Disorganized speech patterns (i.e. tangentiality, circumstantiality) and behavior (i.e. inability to complete daily tasks) are also common. The distinction between the three disorders is made based on the duration of symptoms.

Brief psychotic disorder lasts for less than one month, schizophreniform disorder lasts for one to less than six months, and schizophrenia lasts for over six months.

355
Q

A 23-year-old man comes to the emergency department because of strange behavior for the past week. His girlfriend states that he has been speaking to people who are not present. He also believes that is being followed all the time and suspects his girlfriend is cheating on him. The girlfriend notes that the patient has missed several days of work lately, but denies that the patient has any significant medical problems or medication use. Imaging and a urine drug screen show no abnormalities. What is the next best step in pharmacologic treatment for this patient’s condition?

A Alprazolam
B Haloperidol
C Imitriptyline
D Naloxone
E Quetiapine
A

1 week of delusions = brief psychotic episode

TX = antipsychotics (usually Risperdal or Seroquel)

E Quetiapine

. The first-line treatment is with antipsychotics such as risperidone or quetiapine

356
Q

A 20-year-old man is brought to the office by his parents because of changes in his behavior during the past 5 weeks. He is a college sophomore and has had poor grades this semester due to frequent marijuana use. He says that he hears voices even when he is not intoxicated. He has lost most of his friends due to his strange behavior and rarely socializes. On mental status examination, his speech is clear, but his thought process is disorganized. Which of the following is the most likely diagnosis?

A Schizophrenia
B Brief psychotic disorder
C Schizophreniform disorder
D Schizoaffective disorder
E Marijuana use disorder
A

5 weeks = > 1 month
auditory hallucinations = psychosis
disorganized thought process = PSYCHOSIS X 5 WEEKS
= not schizophrenia yet, and 1 month or more = not brief psychotic disorder

No depression or mania = not schizoaffective

The diagnosis of a schizophrenia spectrum disorder requires the presence of two or more of the following characteristic symptoms: (1) delusions, (2) hallucinations, (3) disorganized speech, (4) grossly disorganized or catatonic behavior, and (5) negative symptoms like affective flattening or avolition. This patient reports auditory hallucinations and has disorganized speech; in addition, his social withdrawal is suggestive of negative symptoms. These changes in behavior have also created dysfunction in his education and personal life. In this case, the diagnosis is established based on the duration of symptoms. Brief psychotic disorder presents with symptoms for less than 1 month, schizophreniform disorder with symptoms for more than 1 month but less than 6 months, and schizophrenia with symptoms for more than 6 months.

357
Q

A 20-year-old man is brought to student health services by his girlfriend due to a 4-month history of bizarre behavior. She reports that he has become increasingly irritable and no longer attends classes or spends time with friends. He stays at home and frequently appears to have conversations with internal voices. He reports that aliens control his mind unless he wears a special outfit. Physical examination shows a disheveled young man wearing clothing adorned with strips of aluminum foil. Mental status examination shows disorganized speech and flattened affect. Which of the following is the most likely diagnosis?

A Schizophrenia
B Brief psychotic disorder
C Schizophreniform disorder
D Schizotypal personality disorder
E Schizoid personality disorder
A

4 months
voices = hallucinations = PSYCHOSIS
no depression or mania

C Schizophreniform disorder

358
Q

A 25-year-old man is brought to the office by his wife because of a change in his behavior during the past 3 months. He had graduated from college with honors but has been skipping classes in graduate school for the past term. He reports that he had a “revelation” that he was chosen for a “special mission” and has instead been engaged in “honoring the machines.” He states that he hears messages from “the deity.” Mental status examination shows disorganized thought process. Urine toxicology screening is negative. Which of the following is the most likely diagnosis?

A Brief psychotic disorder
B Schizophreniform disorder
C Schizoaffective disorder
D Delusional disorder
E Schizophrenia
A

3 months
delusions
disorganized thought process
2+ psychotic symptoms + < 6 months but > 1 month

= B Schizophreniform disorder

359
Q

A 30-year-old homeless individual is admitted to the hospital after he was found trying to cut his wrist. He has a history of major depressive disorder and has been admitted at the same hospital several times for attempted suicide. He reports that his depression had been worsening over the past two months and that he has been experiencing auditory hallucinations for the past year. Two months ago, he stated that he no longer felt depressed and that his mood was back to baseline. However, his auditory hallucinations began to preach religious instructions, and one week ago, they began instructing him to kill himself. On physical examination, he is alert and oriented but does not make eye contact. Which of the following is the most likely diagnosis?

A Bipolar disorder
B Brief psychotic disorder
C Depression
D Schizoaffective disorder
E Schizophrenia
A

D Schizoaffective disorder

Mood symptoms, such as depression, must be absent for at least two weeks while the psychotic symptoms continue (thus differentiating this condition from a major mood disorder with psychotic features). Psychotic symptoms are widely variable, and can be the presence of hallucinations, delusions, and disorganized thoughts and behaviors.

360
Q

A 44-year-old man is brought to the hospital by police because his landlord has observed him acting erratically for the past few days. According to neighbors, he has been known for 10 years to have ecstatic wanderings around his yard naked and defecating in public, or simulating sexual intercourse with lawn ornaments and street lamps while crying. Medical history is significant for auditory and visual hallucinations. His sister states that he also struggled with elevated mood periodically over the past year. She states that during these episodes he does not sleep for many days in a row, believes he is the president, and makes large purchases with his credit card. His temperature is 37.0°C (98.6°F), pulse is 68/min, respirations are 10/min, and blood pressure is 132/86 mm Hg. Which of the following is the most likely diagnosis?

A Mood disorder
B Normal behavior
C Schizoaffective disorder
D Schizophrenia
E Schizophreniform disorder
A

C Schizoaffective disorder
- bipolar type = mania + psychosis

The patient in this scenario displays positive (hallucinations and delusions) symptoms of schizophrenia and prominent affective symptoms. Some with schizoaffective disorder will also display negative (poverty of speech, flat affect, poor motivation, social withdrawal and lack of concern for social conventions) symptoms of schizophrenia. Schizophrenic and affective symptoms points to a diagnosis of schizoaffective disorder. The DSM-V further classifies schizoaffective disorder into manic-type or depressive type depending on the affective symptoms. This patient would fit under the manic category.

361
Q

A 22-year-old man is brought to the hospital because his roommate says that he has been acting very strange for the past few hours. His roommate describes the patient as previously socially active and outgoing guy whose behavior has grown to be paranoid and isolated. He has stopped attending classes, exercising, and does not seem to be eating very often. His roommate acknowledged past episodes involving the patient assumingly combating demons. Family history is significant for a sister with major depressive disorder. His temperature is 37.0°C (98.6°F), pulse is 82/min, respirations are 16/min, and blood pressure is 126/82 mm Hg. Physical examination shows the patient has a flat affect with disorganized speech. Which of the following features would lead to a diagnosis of schizoaffective disorder?

A Catatonic episodes
B Evidence of a manic or mixed episode
C Panic attacks
D Presence of symptoms for more than 1 month
E Presence of symptoms for more than 6 months

A

B Evidence of a manic or mixed episode

patient having symptoms of psychosis
also needs mood disorder

362
Q

A 25-year-old woman comes to the clinic because she has been hearing voices for the past 2 weeks. She says that the voices are telling her that her family wants her poisoned. Her husband states that he has noticed her talking to herself for the past few days. The patient reports trouble sleeping, loss of appetite, decreased energy, and feelings of worthlessness. Which of the following is the most likely diagnosis?

A Brief psychotic disorder
B Delusional disorder
C Major depressive disorder with psychotic features
D Schizoaffective disorder
E Schizophreniform disorder
A

2 weeks
delusions & hallucinations = psychosis

A Brief psychotic disorder

Brief psychotic disorder is the presence of at least one psychotic symptom that lasts for less than 1 month. It is more common in women and immigrants.

not schizophreniform (1+ month, but < 6 months)

not delusional disorder = bizarre / unlikely, but could happen, only one psychotic symptom

Delusional disorder is characterized by the presence of one or more delusions for at least a month in an otherwise normally functioning individual

Not MDD w/ psychotic features - The diagnosis of major depression requires a period lasting longer than two weeks with at least five symptoms of depressed mood, loss of interest, insomnia, change in appetite, psychomotor agitation, low energy, poor concentration, thoughts of hopelessness or guilt, and recurrent thoughts of suicide.

363
Q

A 24-year old male comes to the emergency department because of hallucinations after a weekend “bender.” The hallucinations are complex, including voices and music, and do not extinguish within 48 hours of stopping the alcohol use. The patient denies use of hallucinogens and toxicology screen returns negative. Mental status exam is normal. What is the likely diagnosis?

A Alcohol withdrawal
B Hallucinogen abuse
C Middle ear lesion
D Psychotic disorder
E Salicylate toxicity
A

D Psychotic disorder

Alcohol withdrawal most commonly starts 6 to 24 hours after last alcohol intake. The early symptoms involve anxiety, paresthesia, fever and headache. Later symptoms can include seizures, visual hallucinations, and delirium.

364
Q

A 42-year-old man comes to the clinic because his wife believes that he has been “losing his mind” for the past two months. He works as a lawyer and recently was named employee of the month. He believes that the Earth will be hit by a comet within the next year and has begun to stockpile water and cans of food in his basement. He admits that he has no evidence that this will occur, but simply “knows” that it is going to happen. He exercises regularly and gets 8 hours of sleep each night. Family history is significant for a brother with schizophrenia. He takes no medications. His temperature is 37.0°C (98.6°F), pulse is 68/min, respirations are 11/min, and blood pressure is 132/78 mm Hg. Physical examination shows a well-appearing male in no apparent distress. Laboratory studies are unremarkable. Which of the following is the most likely diagnosis?

A Delusional disorder
B Dementia
C Hypomanic episode
D Mood disorder
E Specific phobia
A

2 months

not dementia
not hypomanic
not mood disorder
not phobia

Delusional disorder is defined as a fixed, persistent, false belief system that lasts longer than one month. These patients do not experience a decrease in functional status and do not qualify for the diagnosis of another psychiatric disorder.

365
Q

A 37-year-old woman comes to the office for an annual visit. She begins recounting her “crazy week.” She says that every day that she wakes up, there are different people acting as her neighbors. They each have a different personality than before. She is not sure if they are acting or wearing different costumes, or if they are shape-shifting into different people. One time, she says that one of her neighbors turned into an animal and was barking all night in the house. She is afraid that one day, they will try to get into her home and take advantage of the fact that she cannot recognize their different forms. Which of the following most accurately describes this patient’s delusion?

A Cotard delusion
B Cacodemonomania
C Dissociative identity disorder
D Capgras syndrome
E Fregoli syndrome
A

E Fregoli syndrome

Fregoli delusion, also known as the delusion of doubles, is a rare disorder in which a person holds the delusional belief that different people in his life are in fact a single person who changes appearance or is in disguise. This disguised person is believed to be taking THE FORM of other people or creatures. The delusion often has qualities of paranoia, since the patient thinks that this other person is persecuting him/her.

Cotard delusion is a condition in which a person believes that he is already dead and no longer exists as a living being, either in part or in whole

Cacodemonomania is the pathological belief that someone is poisoned or unduly influenced by an evil spirit, e.g. the devil. It may lead to the patient seeking exorcism

Capgras Syndrome - This is the pathological belief that family and friends have been replaced by imposters. It may occur in paranoid schizophrenia, brain trauma, or dementia.

366
Q

Which of the following psychiatric conditions is LEAST likely to be associated with an eating disorder?

a. Obsessive compulsive disorder
b. Personality disorder
c. Schizophrenia
d. Substance abuse
e. Anxiety disorder

A

c. Schizophrenia

367
Q

Which of the following are categorized as a positive symptom of schizophrenia?

a. Delirium
b. Isolation
c. Drug abuse
d. Delusions
e. Apathy

A

d. Delusions

368
Q

A 48-year-old man with a history of schizophrenia comes in for a routine visit. He has been doing well on risperdone (Risperdal) for the last 22 years. Physical examination reveals that he is puckering his lips continuously and moving his jaw from side to side. Which of the following side effects does this describe?

a. Dystonic reaction
b. Facial clonus
c. Tardive dyskinesia
d. Akathesia

A

c. Tardive dyskinesia

369
Q

A 35-year-old man comes to the inpatient psychiatric ward because of psychosis. He was admitted because he thinks his neighbors are spying on him and devising ways to kill him. He says they have inserted cameras in several rooms of his house to monitor his activities. He claims to hear them through the walls saying they are “going to get him”. The patient’s wife called the police when he bought a gun stating that he was going to wait for them to come. Which of the following is the most appropriate pharmacologic treatment in this patient?

A Benztropine    
B Diazepam    
C Fluoxetine    
D Lithium    
E Risperidone
A

delusions

E Risperidone

Risperidone is an atypical antipsychotic commonly used to treat schizophrenia, a psychotic disorder characterized by hallucinations, delusions, and abnormal social functioning.

Risperidone is an atypical antipsychotic commonly used to treat schizophrenia. It is notably useful in treating the patient’s present symptoms of paranoid delusions and auditory hallucinations. Common side effects are weight gain, postural hypotension, drowsiness and extra pyramidal side effects.

370
Q

A 42-year-old man comes to the emergency department because of hallucinations. He is subsequently admitted to the inpatient psychiatric ward. He has a long history of schizophrenia. Ruminations about different galaxies and their appropriate aliens have left the patient non-functional in terms of day-to-day life. He describes a small, yellow alien that has been following him around for the past 3 weeks. He says this alien is peaceful, does not speak, and only stands in corners and looks at him. The patient denies distress, suicidal thoughts, homicidal thoughts, thoughts of harming himself or others, or recent drug ingestion. Which of the following is associated with a worse prognosis for this diagnosis?

A Family history of mood disorders other than schizophrenia
B Female sex
C Gradual onset
D Late onset
E Positive symptoms
A
not B
not E (positive symptoms easier to treat than negative)

not A - A family history of schizophrenia is associated with a worse prognosis for current patients with schizophrenia. However, a family history of other mood disorders without schizophrenia warrant a better prognosis

C = GRADUAL ONSET

Schizophrenia risk and prognosis worsen with a family history of schizophrenia, early onset, gradual onset, poor social support, negative symptoms, male sex, many relapses, poor functioning in society, and substance abuse.

371
Q

A 75-year-old woman comes to the clinic for a routine check-up. She has a long history of schizophrenia, requiring antipsychotic treatment on many various agents including risperidone and haldol. She denies a history of depression, suicidal ideation, homicidal ideation, or illicit drug use. Physical examination shows involuntary chewing movements, lip smacking, and grimacing. She says these symptoms may have started about 2 months ago, but she thought it was just a sign of old age. Which of the following is the most likely diagnosis?

A Acute dystonia
B Akathisia
C Athetosis
D Parkinsonism
E Tardive dyskinesia
A

E Tardive dyskinesia

Acute dystonia involves twisting of the neck, trunk, and limbs into uncomfortable positions and usually occurs earlier in the course of treatment.

Akathisia is a common side-effect of antipsychotics. However, it is a movement disorder characterized by a sense of restlessness and inability to sit still.

Athetosis is a form of motor dysfunction characterized by slow, involuntary movements, fluctuations in muscle tone, and imbalance. It is often associated with cerebral palsy

Drug-induced parkinsonism is characterized by a resting tremor, increased rigidity, masked facies and other characteristics often seen in patients with Parkinson syndrome.

372
Q

A 28-year-old man comes to the office because of chest swelling. He was recently diagnosed with schizophrenia and began treatment with risperidone six weeks ago. Starting two weeks ago, he noticed his chest was getting larger and was painful to touch. Physical examination shows mild tenderness to palpation of the chest and an enlargement of breast tissue. Which of the following most accurately describes the pathophysiology of this patient’s gynecomastia?

A Dopamine antagonism in the brainstem produces this on-target side effect
B Dopamine antagonism in the breast tissue produces this on-target side effect
C Dopamine antagonism in the pituitary produces this on-target side effect
D Dopamine antagonism in the striatum produces gynecomastia via its thalamocortical feedback loop
E Some dopamine antagonists produce gynecomastia as an off-target side effect

A

C - Dopamine antagonism in the pituitary produces this on-target side effect

D2 receptor antagonists are often used as antipsychotics and can cause gynecomastia by blocking the effects of dopamine in the tuberoinfundibular pathway. As this is the site of both the adverse effect and part of the therapeutic effect, this is called an “on-target” side effect.

Most patients with schizophrenia will begin pharmacologic therapy with a drug that blocks dopamine D2 receptors. A common side effect of D2 antagonism is an elevation in prolactin secretion. The normal physiology is as follows: dopamine released by the hypothalamus inhibits prolactin secretion when that dopamine binds D2 receptors on the prolactin-secreting cells in the anterior pituitary gland. Inhibiting the physiologic inhibitor, dopamine, causes hyperprolactinemia.

373
Q

A 55-year-old man comes to the emergency department because of delusions and hallucinations. He is subsequently admitted to the inpatient psychiatric ward. He has a long history of schizophrenia. His family says he only talks about extraterrestrial creatures. They say he is now unable to function properly in daily life. While in the psychiatric ward the patient occasionally rambles about a small alien that follows him, does not speak, and stares at him. The patient denies distress and finds the alien interesting and justifies its action by stating they are peaceful and observatory creatures. He denies suicidal thoughts, homicidal thoughts, or any other hallucinations or delusions. Which of the following pathways in the brain is responsible for this patient’s hallucinations?

A Mesocortical
B Mesolimbic
C Nigrostriatal
D Prefrontal cortical
E Tuberoinfundibular
A

B Mesolimbic

Many standard antipsychotics used to treat schizophrenia target dopaminergic pathways such as the mesolimbic, mesocortical, and tuberoinfundibular pathways. The tuberoinfundibular pathway regulates hormone secretion in the pituitary gland; as a result, many antipsychotics can cause and an unintentional increase in prolactin secretion. The mesocortical pathway, which functions in modulating the prefrontal cortex, is associated with negative symptoms including decreased pleasure, emotion, relationships, and motivation. Positive symptoms are believed to originate in the mesolimbic pathway.

374
Q

A 34-year-old man comes to the office because of chest swelling. Six weeks ago, he was diagnosed with schizophrenia which is well-managed with risperidone. Two weeks ago, he noticed his chest was getting larger and was painful to the touch. Physical examination shows bilateral enlargement of breast tissue with mild tenderness to palpation. Which of the following hormones, if abnormally elevated, would most likely cause this condition?

A Adrenocorticotropic hormone
B Insulin
C Parathyroid hormone
D Prolactin
E Thyroid stimulating hormone
A

D Prolactin

Patient has gynecomastia due to hyperprolactinemia

Most patients with schizophrenia will begin pharmacologic therapy with a drug that blocks dopamine D2 receptors. A common side effect of D2 antagonism is an elevation in prolactin secretion. The normal physiology is as follows: Dopamine released by the hypothalamus inhibits prolactin secretion when that dopamine binds D2 receptors on the prolactin-secreting cells in the anterior pituitary gland. Inhibiting the physiologic inhibitor, dopamine, causes hyperprolactinemia.

Risperidone exerts its therapeutic effect centrally by targeting the mesolimbic, prefrontal cortex limbic, and tuberoinfundibular pathways. Gynecomastia is a common side effect of risperidone due to its dopamine D2 receptor antagonism on the tuberoinfundibular pathway, explaining this patient’s “chest swelling.”

375
Q

A 21-year-old man is brought to the office by his parents. He took the semester off from college to work full-time on decoding the hidden text in the telephone book, which he believes will save the country from impending disaster. He stays in his room most of the day and rarely interacts with anyone unless spoken to directly. When asked for more information about the code he is deciphering, he gives one or two-word replies at a barely audible volume. He frequently slurs consonants and trails off when speaking. Which of the following best characterizes this patient’s speech deficit?

A Catatonia
B Disorganized speech
C Alogia
D Flat affect
E Alexia
A

C Alogia

Alogia, or poverty of speech, is characterized by reduced speech output and verbal fluency, as well as slurring and low volume such that the content and manner of speech are affected.

Alexia is an acquired reading disorder, not a speech deficit. It is usually caused by an infarct to the left posterior cerebral artery and is characterized by an inability to read. The ability to write may remain intact.

Catatonia is a state of neurogenic immobility and behavioral abnormalities. It is associated with psychiatric conditions such as schizophrenia, bipolar disorder, and depression. Mutism is one of the diagnostic criteria for catatonia, but this patient does reply to questions.

Disorganized speech reflects disorganized thinking in an underlying formal thought disorder. Patients with psychosis who display disorganized thinking cannot express their thoughts clearly, frequently going off on tangents and making statements whose content cannot be followed by the listener.

376
Q

A 30-year-old man comes to the emergency department because he is hearing voices telling him to kill himself. He recently stopped taking his prescribed ziprasidone because he believes the government is using it to poison him. He denies using any illicit drugs, depressed mood, grandiose mood, excessive energy, or sleep disturbance. His pulse is 80/min, respirations are 14/min, and blood pressure is 120/85 mm Hg. Examination shows that the patient has a flat affect and disorganized speech. Which of the following neuron receptors is most likely hypofunctioning in this patient’s disorder and also antagonized by phencyclidine?

A β-adrenergic receptor     
B Dopamine D2 receptor     
C Metabotropic glutamate receptor-2     
D Muscarinic acetylcholine receptor     
E N-methyl-D-aspartate receptor
A

E N-methyl-D-aspartate receptor

While there is limited evidence for dopamine receptor dysfunction in patients with schizophrenia, phencyclidine does not act on dopamine receptor function.

Two prominent theories of the underlying mechanisms of schizophrenia are the dopamine and glutamate hypotheses.

The dopamine hypothesis proposes excessive D2 receptor activation is a possible cause of schizophrenic symptoms.

For the glutamate hypothesis, NMDA receptor hypofunctioning is believed to be an underlying pathophysiological change present in the disorder.

Research suggests those diagnosed with schizophrenia have fewer N-methyl D-aspartate (NMMA) glutamate receptors in their brains. Ketamine and phencyclidine (PCP) are NMDA receptor antagonists that can produce symptoms very similar to those of schizophrenia.

377
Q

A 26-year-old Caucasian woman is brought to the office for a follow-up evaluation of her mental status and cognitive functioning. Her medical history is relevant to schizophrenia. Upon medical interrogation, the patient frequently loses her train of thought, makes long pauses, and begins talking in the middle of sentences about completely unrelated topics. On physical examination, the patient knows her name and responds in a meaningful manner to verbal instructions or gestures. Before finishing the exam she comments about how beautiful the office is. Which of the following best describes the phenomenon being observed in this patient?

A Dementia
B Disorientation
C Dissociative fugue
D Log rolling
E Thought blocking
A

E Thought blocking

Thought blocking, classically seen in schizophrenia, is a phenomenon where a person frequently pauses, loses train of thought, and begins talking about completely unrelated topics at unexpected times.

Disorientation is a cognitive disability characterized by losing one’s sense of person, place, and time.

Dissociative fugue is a psychiatric disorder characterized by an episodic, reversible amnesia of one’s personal identity, memories, and personality. These episodes last anywhere from hours to months, but memories eventually recover intact. Here, however, the patient knows her name, which makes the presence of a dissociative fugue less likely.

Log rolling is a maneuver used to move a patient without flexing the spinal column. It is used to move a patient without bending or twisting the body, or sometimes to facilitate vomiting by placing a patient on his/her side.

378
Q

A 30-year-old woman comes to the clinic because of a progressive decline in social and occupational functioning over the past year, along with a withdrawal from her regular activities. In addition, her family notes that over the past 4 months she has had paranoid delusions, exhibited disorganized speech, and has been hearing voices. She has not had any major depressive or manic episodes. A physical examination reveals a disheveled female with a flat affect, poor eye contact, and loosely-associated speech. A toxicology screen and basic laboratory analysis are unremarkable. She is not on any medications. Which one of the following is the most likely diagnosis?

A Brief psychotic disorder
B Delirium
C Delusional disorder
D Schizoaffective disorder
E Schizophrenia
A
year
social / occupational decline
withdrawal
delusions
disorganized speech
hallucinations
= E schizophrenia = 6 months, with 1+ month of 

negative symptoms = flat affect / poor eye contact / loosely associated speech

not brief psychotic disorder = 1 day +, < 1 month
not delusional disorder = 1+ delusion lasting 1+ month, no other psychotic symptoms

not schizoaffective = psychosis + mood disorder, but psychosis has to occur without mood (depression or mania) = patient has NOT had any major depressive or manic episodes

379
Q

A 45-year-old man comes to the emergency department because of delusions and hallucinations. He is admitted to the inpatient psychiatric ward. He has a long history of schizophrenia and alarmed his family after talking about extraterrestrial beings. Frequent discussions of aliens and alternate galaxies has resulted in the loss of his job and an inability to function in day-to-day interactions. During his time in the emergency department, he describes a blue alien following him around. This alien does not speak but simply stands in the corner and observes the patient. The patient denies distress and calls the alien his friend. The patient denies suicidal thoughts, homicidal thoughts, or any other hallucinations or delusions. Prior documentation shows a questionable history of substance abuse. Which of the following substances is most likely to be abused by patients with this type of presentation?

A Alcohol
B Caffeine
C Cocaine
D Marijuana
E Tobacco
A

E Tobacco

Tobacco abuse is the most common form of substance abuse in people with schizophrenia. 75-90% of people with schizophrenia are addicted to nicotine, three times greater compared to the general population.

380
Q

A 49-year-old man comes to the emergency department because of hallucinations. He is admitted to the inpatient psychiatric ward and is awaiting transport from the emergency department. He has a long history of schizophrenia and alarmed his family with talks about aliens from outer space. He often has ruminations about different galaxies and their particular alien species, but he has remained functional in terms of day-to-day life as a custodian. The patient describes with vivid detail a green alien in the room that does not speak but simply stands in the corner and observes the patient. The patient denies distress and finds the alien interesting and justifies its action by stating they are peaceful and observatory creatures. He is in no acute distress and denies suicidal thoughts, homicidal thoughts, or any other hallucinations or delusions. Which of the following is the most appropriate physician response to the patient’s current condition?

A Conservative management, monitor the patient
B Treat with atypical antipsychotics
C Treat with bupropion
D Treat with fluoxetine
E Treat with haloperidol immediately for agitation

A

A Conservative management, monitor the patient

Hallucinations are a hallmark of schizophrenia. They do not necessarily require pharmacological treatment when they are not distressing, harmful, or directive toward the patient.

Antipsychotics are the standard first-line of treatment for schizophrenia; however, the patient is calm and his hallucinations are peaceful. There is no urgent need to medicate him.

This patient’s hallucinations are not commanding or derogatory. They are not instructing him to do anything like hurt himself or hurt others. Furthermore, he is not in any distress. In fact, he is amused, and has been able to keep a job. The most appropriate next step is conservative management and just monitor the patient to see if there are any changes from baseline. It is alarming if the patient is doing anything under the instruction of his/her hallucination(s). However, this patient describes an exclusively observational relationship between himself and his hallucinations.

There is a fine line to be drawn between hallucinations that are harmless and harmful. As long as the patient is not listening to the hallucinations and they are not distressing, no treatment is necessary. However, if the patient is alarmed and distressed or following commands instructed by the hallucinations, medication can help. There is no overall cure for schizophrenia, so medicating this patient even when he is in a calm, harmless state only puts him at an unnecessary risk of adverse side effects.

381
Q

A 24-year-old man comes to the clinic because of symmetric bradykinesia and postural instability. Records show he came to the clinic 2 months ago because of paranoia. He said his neighbors were running around his house, whispering about him at night, and plotting to take away his dog. However, he now says he does not have a dog. He began taking a prescribed medication after his prior visit and says it has helped him calm down and has alleviated his delusions. He says instead of his mind, he now feels he cannot control his body. Which of the following medications was this patient most likely prescribed?

A Apomorphine
B Benzodiazepine 
C Levodopa
D Metoclopramide 
E Phenothiazine
A

E Phenothiazine

Phenothiazine is a tranquilizer that can be used to treat schizophrenia, a psychotic disorder characterized by paranoid delusions and hallucinations. However, this medication can cause Parkinson-like symptoms because it decreases dopamine levels in the nigrostriatal pathway

382
Q

A 30-year-old woman comes to the office because of auditory hallucinations. She says she has been hearing voices telling her to hurt herself for the past 6 months. She lives with her parents and believes they have been plotting against her for the past two weeks. Mental status examination shows a disorganized speech, flat affect, and positive persecutory delusions with auditory hallucinations. Which of the following chemical changes is associated with the most likely diagnosis?

A Decreased acetylcholine      
B Decreased γ-aminobutyric acid 
C Decreased norepinephrine, serotonin, and dopamine      
D Decreased serotonin      
E Increased dopamine
A

6 months - auditory hallucinations
delusions
disorganized speech / flat affect
= schizophrenia

E Increased dopamine

schizophrenia = too much dopamine
antipsychotics = dopamine antagonists
parkinsons = too little dopamine

Schizophrenia is associated with changes in glutamate and dopamine levels. Many antipsychotics block D2 receptors in order to improve the positive symptoms of schizophrenia.

This presentation is suggestive of schizophrenia, which is characterized by an elevation of the neurotransmitter dopamine. Schizophrenia is diagnosed by having signs of the illness for six months with at least one month of active symptoms.

Patient must have at least two of the following symptoms:
Delusions
Hallucinations
Disorganized speech
Catatonic behavior
Negative symptoms (e.g., flat affect or mutism)

At least one of the two symptoms must be delusions, hallucinations, or disorganized speech.

Schizophrenia is treated with antipsychotic medications, which primarily have effects on dopamine receptors in the brain, specifically D2 receptors. Here they have an antagonist effect, meaning that they block the receptors. This classification of drugs is needed in order to reduce the excess amounts of dopamine in the brains of patients with schizophrenia and other psychotic disorders. Chemically, schizophrenia is also associated with decreased glutamate activity. This is supported by the fact that many NMDA glutamate receptor antagonists such as ketamine and phencyclidine have side effects similar to symptoms of schizophrenia.

383
Q

A 22-year-old man comes to the emergency department because of “unusual behavior” for the past 6 months. He is a college student living with his parents who brought him in because he has been having more frequent and worsening outbursts of anger. The patient is also spending much more time alone in his room, rarely goes out with friends, and constantly skips classes (which is not normal for him). The patient says he has been staying in his room because the government is spying on him and he is not sure who he can trust. He has seen the agents outside his window, and hears the bugs in his room. Assuming the standard treatment for this type of presentation, which of the following chemical receptors is most likely to be targeted?

A Acetylcholine
B Dopamine
C GABA
D N-methyl-D-aspartate
E Norepinephrine
A

hallucinations (AVH)
delusions
6 months = schizophrenia

B Dopamine

Antipsychotics are the first-line treatment for schizophrenia. Many of them primarily interact with dopaminergic pathways. A number of antipsychotics also interact with glutaminergic pathways.

The first-line treatment for schizophrenia is antipsychotic medication. Most classic antipsychotics (neuroleptics) act mainly on dopamine receptors. Atypical antipsychotics act on both 5-HT and dopamine receptors. Many antipsychotics also interact with glutaminergic pathways, although the majority of antipsychotics used to treat schizophrenia primarily target dopaminergic pathways.

384
Q

A psychologist sees a patient with atypical schizophrenia on a weekly basis in an effort to minimize medication use and utilize behavioral modifications and psychological interventions. The psychologist wants to assess the patient’s personality with a projective test that involves noting the response, perception and understanding of 10 black and white stimuli in a sequential manner. Which test did the psychologist employ?

A Rorschach Test
B Beck Depression Inventory
C Luria-Nebraska Neuropsychological Battery
D Thematic Apperception Test

A

A Rorschach Test

Also known as the ‘inkblot test,’ the Rorschach test is used to gauge a patient’s psychological state by asking the subjects to respond to a series of inkblot images. The perceptions of the inkblots are recorded and then analyzed using psychological interpretation. It is often used to detect a thought disorder for example in patients that are hesitant to describing their feelings and thought processes with others.

Thematic apperception test = This is a projective psychological test that reveals underlying motives, concerns, and perspectives by asking the patient to make up a story of a provided scene.

385
Q

A 40-year-old man comes to the emergency department because of schizophrenia, which he was diagnosed for 3 months ago. His family says for the past 3 weeks he has been constantly talking about extraterrestrial beings. He lost his job a week ago due to his ruminations about different galaxies and the aliens who live there. He describes with vivid detail a green alien in the room that does not speak but simply stands in the corner and observes him. The patient denies distress and finds the alien interesting and justifies its action by stating they are peaceful and observatory creatures. He denies suicidal thoughts, homicidal thoughts, or any other hallucinations or delusions. Which of the following is considered the most appropriate type of behavioral therapy for this patient’s condition?

A Flooding
B Group therapy 
C Milieu therapy
D Self-control therapy
E Systematic desensitization
A

Milieu therapy is a form of psychotherapy thought to be of value in treating personality disorders, behavioral problems, and schizophrenic patients.

386
Q

A 45-year-old man comes to the clinic for his monthly depot haloperidol shot for management of his schizophrenia. After leaving the clinic with his family, he has trouble looking at the street and feels like he can no longer look down. He begins cursing loudly and exhibiting frustration over the fact that perhaps he was given too much haloperidol during his monthly appointment. His family drive him back to the outpatient mental health clinic and explain the incident. Which of the following medications will most likely relieve this patient’s problem?

A Phenytoin
B Risperidone
C Alprazolam
D Acetaminophen
E Diphenhydramine
A

Oculogyric crisis occurs when the eyes deviate upwards or downwards and essentially gets stuck in position; the treatment is diphenhydramine.

This patient is experiencing an oculogyric crisis. This occurs when the eyes deviate upwards or downwards and essentially gets stuck in position as a result to certain drugs or medical conditions. This can particularly be frustrating and painful when you are focusing on something like reading or driving.

Drugs that can trigger an oculogyric crisis include haloperidol, carbamazepine, chloroquine, metoclopramide, levodopa, and reserpine. High-potency neuroleptics are the most common cause in the clinical setting

The treatment for this as for any dystonic reaction (extrapyramidal side effect) to haloperidol is anticholinergic diphenhydramine. Epilepsy can manifest as oculogyric seizures, also called versive seizures.

387
Q

A 32-year-old man comes to the office for a follow-up after being diagnosed with atypical schizophrenia. He has no complaints at this time but wants to discuss his course of treatment. He sees a psychologist on a weekly basis in an effort to minimize medication use and utilize behavioral modifications and psychological interventions. The psychologist wants to assess the patient’s personality with a subjective test that involves the patient making up a story about what is going on in a cartoon picture of people. Which of the following tests best fits this criteria?

A Beck Depression Inventory
B Folstein test
C Luria-Nebraska Neuropsychological Battery
D Rorschach test
E Thematic Apperception Test
A

E Thematic Apperception Test

The Thematic Apperception Test (TAT) is a projective test that reveals underlying motives, concerns, and the way a person see the social world through the stories they make up about ambiguous pictures of people.

388
Q

A 10-year-old boy is brought into the clinic with a chief complaint of “poor grades”. He is failing in reading, but writes well. He enjoys math and is passing all his other courses. He tells you he does not like to read because he is the slowest reader in the class and has a hard time making out the right words. Upon physical examination you find he is of average intelligence with a normal mental status exam. What is the most likely diagnosis?

a. Dyscalculia
b. ADHD
c. Dyslexia
d. Dysgraphia

A

c. Dyslexia

389
Q

Which of the following terms is best described as a writing disability that consists of poor handwriting, difficulty with spelling, inappropriate word sequencing and putting thoughts into writing?

a. Dyslexia
b. Dyscalculia
c. Dysgraphia
d. Intellectual disability

A

c. Dysgraphia

390
Q

A 40-year-old woman is brought to emergency room following a suicide attempt wherein she slit her wrists. She has a long history of depressive episodes. However, in between she reports periods of high energy and insomnia, during which she will spend thousands of dollars on useless items. She also believes that she will be the next “American Idol” winning contestant if she were to enter the contest. Which of the following would be an expected mental status observation in this patient if she presented during one of these episodes?

A Word salad     
B Flight of ideas     
C Magical thinking
D Thought blocking      
E Ideas of reference
A

depressive episodes + high energy/insomnia = bipolar

B Flight of ideas

Mania is a high-energy state seen in certain psychiatric disorders. Manic patients may demonstrate flight of ideas, in which they switch rapidly between tenuously related ideas and concepts.

To be a considered a manic episode, the episode must last at least one week, and must cause obvious impairment in the patient’s life.

Word salad is the term for an incoherent collection of words. This can be seen in patients with Wernicke’s aphasia from a stroke.

Ideas of reference are seen in delusional disorders, including schizophrenia. They involve the belief that casual events, people’s remarks, etc. are referring to oneself when, in fact, they are not.

Magical thinking refers to the idea that thinking about an event or object will then somehow affect it. This symptom may be seen in extremes in patients with certain personality disorders or schizophrenia.

391
Q

A 30-year-old woman comes to the emergency department after exhibiting unusual behavior. According to her mother, she has not slept for more than a week, appears to be extremely agitated, and is unable to concentrate. Her mother says she has seen her like this in the past and it is usually followed by an episode of severe depression. She is admitted for psychiatric evaluation and treatment is initiated. Three weeks after she is discharged from the hospital, she begins complaining of polyuria and intense thirst. Urinalysis reveals a specific gravity of 1.001 and an osmolality of 190 mOsm/kg. After one hour of water restriction, urine osmolality is 210 mOsm/kg. Which of the following is the most likely medication that could cause these symptoms?

a) Carbamazepine
b) Demeclocycline
c) Lamotrigine
d) Lithium
e) Valproic Acid

A

d) Lithium

Adverse effects of lithium include tremor, edema, heart block, hypothyroidism, teratogenesis, and nephrogenic diabetes insipidus. It is diagnosed by a lack of urine osmolality change with water deprivation. The normal response to dehydration is to concentrate urine, causing urine osmolality to increase and stabilize above 280 mOsm/kg. Patients with diabetes insipidus will have an increase in osmolality of less than 30 mOsm/kg per hour for at least 3 hours.

In patients undergoing pharmacotherapy for bipolar disorder and later present with thirst and polydipsia, primary polydipsia and nephrogenic diabetes insipidus secondary to lithium may be the cause. If urine osmolality does not improve with water restriction, the most likely diagnosis is diabetes insipidus.

392
Q

A 28-year-old woman is brought to the emergency department by her sister because of erratic behavior. The sister claims that while driving the patient said “let’s close our eyes” while crossing red-traffic lights, and then laughed frenetically. The patient works as a stockbroker and over the past 2 months she has joined multiple organizations as a consultant. She claims to sleep with different men after closing business deals because she feels empowered. When asked about her personal information, she talks very rapidly, claims to feel better than ever, while assuring that there is no need to keep her in the hospital. Her medical history is relevant for being diagnosed with major depressive disorder a year ago, and anorexia nervosa when she was 15-years-old. Physical examination is noncontributory. Her temperature is 36.1°C (96.9°F), pulse is 94/min, respirations are 29/min, and blood pressure is 125/60 mm Hg. Which of the following is the most likely diagnosis?

A Bipolar disorder type 1
B Bipolar disorder type 2
C Borderline personality disorder 
D Brief psychotic disorder
E Histrionic personality disorder
A

A Bipolar disorder type 1

Bipolar disorder type 1 is a mood disorder characterized by at least one episode of mania with or without a hypomanic or depressive episode. The patient must experience inflated self-esteem, decreased need for sleep, increased rate of speech, flight of ideas, increased libido, goal-directed activity/psychomotor agitation, and erratic behavior.

Bipolar 2 disorder is a mood disorder characterized by at least one episode of hypomania and at least one episode of major depression. A hypomanic episode, unlike a manic episode, is not severe enough to cause social and/or occupational impairment or to necessitate hospitalization

The hallmark sign in these patients is the presence of a manic episode, which is characterized by a distinct period of abnormally and persistently elevated, expansive, or irritable mood, lasting at ≥7 days.

393
Q

A 24-year-old man is brought to the emergency department by his girlfriend because he has not slept for 72 hours. She says he is usually a calm and relaxed person. The patient says “it is no big deal because I have finally figured out how to beat online blackjack and made a lot of cash!” While pacing around the room during the interview, he continues to speak rapidly about his plans for “building the better race car,” and “picking stock winners.” His pulse is 110/min. His nasal mucosa is normal and a urine toxin screen is negative. Which of the following is the most likely diagnosis?

A Acute mania
B Alcohol intoxication
C Cocaine intoxication 
D Narcissistic personality disorder 
E Psychosis
A

A Acute mania

Symptoms of an acute manic episode can be remembered with the mnemonic DIG FAST (distractability, irresponsibility, grandiosity, flight of ideas, activity increase, sleep deficit, and talkativeness). At least three of the symptoms within DIG FAST must be present and persist for at least a week, causing the inability for one to take care of oneself for diagnosis.

394
Q

A 24-year-old woman presents to the psychiatry department with concerns over her mood. Her boyfriend convinced her to speak to someone about her unstable emotional states. However, she does not report any significant impairment on her daily functioning. She was a victim of sexual assault years ago, and has been diagnosed with cyclothymic disorder. Which of the following symptoms is most commonly characterized by patients with this disorder?

A Episodes of hypomania occurring for at least two years
B Insomnia for a week
C Psychosis
D Severe mood swings
E Suicidal thoughts
A

A Episodes of hypomania occurring for at least two years

Cyclothymic disorder is a mild form of bipolar disorder in which a person has mood swings over a period of at least 2 years that go from mild depression to emotional highs.

Mood swings are relatively less severe than in bipolar disorder or major depression. According to DSM-V, cyclothymic disorder is characterized by episodes of hypomania and mild depression for at least 2 years. An episode of hypomania is like a manic episode, except not severe enough to cause significant impairment on daily functioning. Hypomanic episodes last at least 4 consecutive days. Conversely, a manic episode is a distinct period of elevated or irritable mood and high energy lasting at least a week.

395
Q

A 42-year-old woman comes to the emergency department because she has been very tired and has gained weight during the past month. She says she has been struggling with depression for a long time, but it has gotten worse since she found out her husband was diagnosed with cancer. She also says that she previously had an episode of decreased need for sleep, irritability, increased goal-directed activity, but it never significantly impaired her work or home life. Her temperature is 37.1oC, pulse is 80 /min, respiratory rate is 12/min, and blood pressure is 110/75 mmHg. Physical examination is normal. She says that she has been compliant with her antidepressant therapy. Which of the following is the most likely diagnosis?

A Schizoaffective disorder
B Bipolar I
C Cyclothymic disorder
D Seasonal affective disorder
E Bipolar II
A

bipolar II - never significantly impaired her work or home life

E Bipolar II

Bipolar II disorder is characterized by one or more hypomanic episodes and one or more major depressive episode. Hypomanic episodes do not go to the full extremes of mania.

Bipolar II disorder is characterized by at least one or more hypomanic episodes and one or more major depressive episode. Hypomanic episodes do not go to the full extremes of mania (i.e., do not usually cause severe social or occupational impairment, and are without psychosis), and this can make bipolar II more difficult to diagnose, since the hypomanic episodes may simply appear as a period of successful high productivity and is reported less frequently than a distressing, crippling depression. The diagnosis of bipolar II disorder requires that the individual never experienced a full manic episode. The most common treatment of bipolar disorder requires the administration of mood stabilizers such as lithium.

396
Q

A 49-year-old woman comes to the emergency department because of a depressed mood, increased fatigue, poor appetite, and trouble concentrating at work for the past month. She says that she was previously treated for bipolar II disorder, but does not currently take medication for it. She has hypertension, diabetes mellitus type 2, and end-stage renal disease. She had a hysterectomy and a thyroidectomy three years ago. Her temperature is 37.1°C (98.8°F), pulse is 84 /min, respirations are 10/min, and blood pressure is 110/75 mm Hg. She moves sluggishly and has a slow speech. Which of the following is the most appropriate next step in management?

A Lamotrigine
B Lithium
C Olanzapine
D Valproic acid
E Sertraline
A

A - Lamotrigine

This patient is in the depressed stage of bipolar II disorder. The patient also has end-stage renal disease (ESRD) which is a strict contraindication for the use of lithium. Lithium can be used in both the acute manic and depressive stages of bipolar disorder but has a narrow therapeutic range and should not be used in patients with impaired renal function. Lamotrigine is a second-line agent for the treatment of depressive symptoms of bipolar II disorder and is the only agent listed aside from lithium which is used in the depressive stage. Also, Lamotrigine is now the drug of choice in all 3 trimesters of pregnant patients with bipolar disorder, although, one must take special care during the first trimester.

397
Q

A 20-year-old woman comes to the emergency department because she was having suicidal ideation. Her father says that last week, she was fully of energy, talking excessively, and staying out all night despite having school and work. When questioned, she says that she discontinued using her medication lithium because of the way it made her feel. Which of the following is a common adverse effect of lithium?

A Agranulocytosis     
B Fetal neural tube defect     
C Sexual dysfunction     
D Tremors     
E Weight loss
A

D Tremors

Agranulocytosis is a known serious side effect of clozapine, an atypical antipsychotic. Although atypicals can be used as mood stabilizers, clozapine would not be an ideal choice given its potential for agranulocytosis.

valproic acid is also used for mood stabilization and has been associated with this fetal anomaly - fetal neural tube defect

Sexual dysfunction is a common side effect for patients taking antidepressants, specifically those in the selective serotonin reuptake inhibitor (SSRI) class.

Although it can depend on the individual, one of the more common side effects of taking lithium is weight gain, not weight loss. This is because lithium can cause changes in metabolism and an increase in appetite.

398
Q

A 25-year-old woman comes to the psychiatric office because of her mother’s referral. She recently dropped out of college to pursue a home business. For the past 4 weeks, she has exhibited an extraordinary level of self-confidence, racing thoughts, and insomnia. She recently maxed out her credit cards to make travel plans to Costa Rica, spending more than $10,000 in a week. During the patient interview, her speech is notably pressured and she is quick to change topics. Which of the following is the most appropriate treatment?

A Amitriptyline
B Buproprion
C Olanzapine
D Propranolol
E Sertraline
A

C Olanzapine

Acute mania is a component of bipolar disorder. It is a medical emergency and warrants prompt treatment with an antipsychotic and mood stabilizer, such as lithium or valproate. Signs of mania include elevated mood, reckless behavior and pressured thoughts and speech.

First line treatment in a manic episode is lithium or valproate, a mood stabilizer, in combination with an antipsychotic

399
Q

A 17-year-old girl is brought to the emergency room by her parents after she was found trying to ingest bleach. She appears restless and agitated, and says that she is a terrible person who does not deserve to be alive. Her parents report that she had been more talkative and irritable than usual. However, since yesterday, she has been confused and withdrawn. She had not slept more than four hours a night for the last two weeks, and she was fired from her summer job due to making inappropriate sexual innuendos. She tells you that her bad behaviors have been broadcasted on the national news, and that the police are looking for her, however, neither of these things have happened. Which of the following is the most appropriate medication to be administered as a first-line therapy for her condition?

A Buspirone
B Haloperidol
C Lithium
D Mirtazapine
E Phenobarbital
A

bipolar I = restlessness / agitated / talkative / sleep deficit / psychosis

C Lithium

Bipolar 1 disorder is a mood disorder in which patients experience manic and major depressive episodes. Lithium is first-line treatment for bipolar disorder.

Manic symptoms consist of elevated or irritable mood lasting for at least a week. Grandiosity, decreased need for sleep, talkativeness, racing thoughts, hypersexuality, distractibility, and psychomotor agitation are other symptoms of mania.

400
Q

A 28 year-old woman with a 12-year history of bipolar disorder type II has been stable on valproate and bupropion for the past 3 years. She and her husband want to start a family, and she inquires if she should change her medications in order to have the best chances of having a healthy baby. She expresses concern about having to discontinue her medications as past attempts to do so have resulted in relapse. Given the circumstance, which of the following is the best recommendation for her treatment regimen?

A No change of medications as both drugs are considered safe in pregnancy
B Replacing both medications with lithium
C Replacing bupropion with fluoxetine
D Replacing valproate with carbamazepine
E Replacing valproate with lamotrigine

A

valproic acid –> can lead to neural tube defects
lamotrigine = drug of choice for bipolar pregnancy

E Replacing valproate with lamotrigine

The first-line treatment of bipolar disorder is a mood stabilizer, like valproate, with an antipsychotic. Valproate inhibits maternal folate absorption and cause neural tube defects. Thus, lamotrigine can be used in pregnant women instead.

Valproate is a category D medication for use in pregnancy because of its known teratogenic effects. In utero exposure can cause neural tube defects and decreased IQ score.

Not only is lithium rarely given alone for the treatment of bipolar disorder, lithium is a category D medication because of its known teratogenic effects. It is associated with Ebstein anamoly in newborns if lithium is taken while pregnant.

Although fluoxetine is not a known teratogen, it is not approved for treatment of bipolar disorder. Serotonin-specific reuptake inhibitors (SSRIs) should be avoided in patients with bipolar disorder because of the risk of precipitating a manic or hypomanic episode.

Carbamazepine is a category D medication. Known teratogenic effects include spina bifida, craniofacial defects, and cardiovascular malformations.

Although valproate is often considered in the first-line treatment of bipolar disorder, it poses a risk to pregnant women because it inhibits maternal folate absorption and thus can cause neural tube defects. Lamotrigine can be used as a substitute, and is not a known teratogen.

401
Q

which personality disorder is marked by unstable relationships, marked sensitivity to criticism and rejection, threats to hurt themselves?

a) histrionic
b) borderline
c) schizotypal
d) narcissistic
e) OCPD

A

BORDERLINE PERSONALITY DISORDER

402
Q

according to DSM-5, for ADHD diagnosis, must have inattentiveness and hyperactivity with what other factor?

A

must affect patient in 2+ settings

403
Q

pupillary dilation, tearing, piloerections, myalgias, chills, N/V, diarrhea are SE of withdrawal from

a) benzos
b) cocaine
c) opioids
d) alcohol
e) marijuana

A

c) OPIOIDS

404
Q

patient with PMDD - what is the 1st line treatment?

A

SSRIs

405
Q

patient has racing thoughts, increased risk taking, impulsivity, excessive speech, etc = most likely experiencing what?

A

BIPOLAR I DISORDER

406
Q

patient has symptoms of depression, then has 1 week of delusions / hallucinations

A

BRIEF PSYCHOTIC EPISODE

407
Q

which of the following can increase levels of lithium and cause lithium toxicity?

a) SSRIs
b) Hydrochlorothiazide
c) aspirin
d) benzos

A

HYDROCHLOROTHIAZIDE

loop diuretics (furosemide) and thiazides
NSAIDS (ibuprofen, indomethacin)
408
Q

patient has symptoms of depression (decreased activity, weight gain, hypersomnia, etc), which medication is 1st line

A

SSRIs - Zoloft (sertraline)

409
Q

treatment for bulimia

A

CBT, counseling and Prozac (fluoxetine)

410
Q

which of the following is a negative symptom of schizophrenia?

a) hallucinations
b) delusions
c) racing thoughts
d) speech poverty
e) disorganized behavior

A

d) speech poverty

POSITIVE SYMPTOMS OF SCHIZOPHRENIA (new / additional)
⦁ delusions
⦁ hallucinations
⦁ disorganized speech (word salad)
⦁ disorganized behavior (ex: wearing winter coats in summer)

NEGATIVE SYMPTOMS OF SCHIZOPHRENIA (removal or reduction of processes)
⦁ affective flattening - inappropriate response**
⦁ poverty of speech (alogia)
** = lack of content in their speech
⦁ blocking (losing train of thought)
⦁ poor grooming
⦁ lack of motivation (avolition**)
⦁ anhedonia (don’t find pleasure in anything anymore)
⦁ social withdrawal

411
Q

A 22-year-old woman is found to have a several month history of mood fluctuations that are predominately elevated, expansive and at times irritable and three additional symptoms of mania are noted. She denies a history of clinical depression. Which of the following is the most likely diagnosis?

a. Elevated mood disorder
b. Anxiety disorder
c. Schizotypal disorder
d. Bipolar disorder

A

d. Bipolar disorder

412
Q

Which of the following medications is the most appropriate treatment for acute bipolar disorder?

a. Chlorpromazine (Thorazine)
b. Diazepam (Valium)
c. Quetiapine (Seroquel)
d. Fluoxetine (Prozac)
e. Haloperidol (Haldol)

A

lithium
mood stabilizers = lithium, carbamazepine, valproic acid or atypical antipsychotics
not Prozac, not Valium

c. Quetiapine (Seroquel)

Mood stabilizers and atypical antipsychotics are the drugs of choice for the treatment of bipolar disorder***

First line treatment in a manic episode is lithium or valproate, a mood stabilizer, in combination with an antipsychotic

413
Q

The diagnosis of Bipolar I differs from Bipolar II in that Bipolar I is characterized by:

a. Mild depression
b. Hypomania
c. Major depression
d. Full manic episodes

A

d. Full manic episodes

414
Q

A previously healthy 25-year-old woman is brought in by her mother for being “out of control.” The mother states that over the past few weeks her daughter has not been sleeping, has been spending money recklessly and is unable to maintain employment as she has been fired from four jobs over the past 6 months. Mental status examination reveals pressured speech, lack of focus, and distractibility. Which of the following is the most likely diagnosis?

a. Schizophrenia
b. Avoidant disorder
c. Histrionic disorder
d. Generalized anxiety disorder
e. Bipolar disorder

A

e. Bipolar disorder

415
Q

Which of the following is the drug of choice for first-line treatment for acute depression in bipolar disorder?

a. Lamotrigine (Lamictal)
b. Venlafaxine (Effexor)
c. Paroxetine (Paxil)
d. Bupropion (Wellbutrin)

A

a. Lamotrigine (Lamictal)

lithium or lamotrigine

Lamotrigine = 2nd-line agent for the treatment of depressive symptoms of bipolar disorder and is the only agent listed aside from lithium which is used in the depressive stage.

Also, Lamotrigine is now the drug of choice in all 3 trimesters of pregnant patients with bipolar disorder, although, one must take special care during the first trimester.

416
Q

A 38-year-old woman comes in with symptoms of abdominal pain. History taking is difficult as she is inconsistent and dramatic about her pain. Physical examination reveals multiple scars on her abdomen. When asked about her previous surgeries and the physicians who cared for her she is reluctant to provide any information. What is the most likely diagnosis?

a. Generalized anxiety disorder
b. Bipolar disorder
c. Factitious disorder
d. Borderline personality disorder

A

c. Factitious disorder (Munchausen’s)

417
Q

Signs and symptoms of autism spectrum disorder typically become apparent at what age?

a. Infancy to less than 3 years of age
b. Less then 1 month of age
c. School age children (6-12 years old)
d. Adolescence (> age 12 years old)

A

a. Infancy to less than 3 years of age

418
Q

At what age is screening for autism spectrum disorder with the M-CHAT first recommended?

a. 6 months
b. 12 months
c. 18 months
d. 36 months

A

c. 18 months

419
Q

Which disorder type does autism spectrum disorder belong in?

a. Conduct
b. Neurodevelopmental
c. Dissociative
d. Somatoform

A

b. Neurodevelopmental

420
Q

An 8-year-old boy is brought in to the clinic for a routine visit. His mother reports that he does not engage verbally with other people but, he is an intelligent boy who likes school. For example, when the other kids are playing kick ball, he is reading about the Civil War. He knows more about the Battle of Gettysburg than anyone else in Yellowstone County but cannot be bothered with third grade Social Studies. Adults enjoy him but he has few peer relationships and really does not want them. What is his most likely diagnosis?

a. Autism spectrum disorder
b. ADHD
c. OCD
d. Social phobia
e. Dysthymia

A

a. Autism spectrum disorder

421
Q

A 5-year-old boy is brought into the emergency department by his mother because of an uncontrollable, violent tantrum for more than an hour. The patient is shaking his head from left to right vigorously and is kicking and screaming. He is spitting and trying to bite anyone who gets near him. The mother states that for the last 2 years, she has noticed her son is very socially isolated. He does not make friends and does not get along with his older brother. She reports her son speaking in short sentences and sometimes not even saying anything, resorting to pointing, pulling, and shouting. He has an intense interest in doorknobs, always playing with them and sometimes locking himself inside rooms. The patient refused to look at you or anyone that tries to talk him into calming down. Which of the following are FDA approved to treat the patient’s acute presentation?

A Aripiprazole, risperidone
B Carbamazepine, gabapentin
C Chlorpromazine, lithium
D Fluoxetine, paroxetine
E Haloperidol, thioridazine
A

A Aripiprazole, risperidone

Risperidone (Risperdal) and Aripiprazole (Abilify) are the only medications FDA-approved for use on hyperactivity and irritability associated with autism.

The only FDA approved selective serotonin reuptake inhibitor (SSRIs) for mood disorders in children are fluoxetine (down to age 6) and escitalopram (down to age 12). Paroxetine is contraindicated in children.

Carbamazepine and gabapentin are both medications used to treat seizure disorders. Carbamazepine is used to treat young patients with epilepsy, while gabapentin is used for the treatment of partial seizures in pediatric patients three years of age and older.

422
Q

A 3-year-old boy is brought to the pediatrician so his parents can discuss some concerns they have with his development. Their son does not make eye contact when spoken to and has yet to meet any of his language milestones. The caregivers at his daycare have noticed that he does not like playing with other children and spends most days sitting quietly alone, playing with a set of blocks. He is at the 50th percentile for both height and weight. Physical exam is within normal limits with no significant findings. This child will most likely be diagnosed with which of the following?

A Autism spectrum disorder     
B Childhood disintegrative disorder     
C Down syndrome     
D Klinefelter syndrome
E Rett’s disorder
A

no eye contact
developmentally delayed
isolation

has yet to meet any milestones = not childhood disintegrative disorder

A Autism spectrum disorder

This child’s history of delayed language development and inability to make eye contact are common features. Autistic children often focus more on objects then people and have below normal intelligence. Autism is also more common in boys and apparent before the age of three.

423
Q

A 51-year-old woman is seeking a psychological evaluation for a learning disability. Her mother was 45 at the time of birth, and she was told as a child that she was deprived of oxygen during delivery which caused minor brain damage. She does not think this is true. She states that she has always felt awkward and incapable in social situations, never knowing the right thing to say. She has always been clumsy. Her mood will change rapidly, especially in public, and she will withdraw into herself and only feel capable of glaring and muttering under her breath. While she has few interests, they are topics about which she has an incredible knowledge. She follows the same exact schedule every day and becomes extremely flustered when something interferes with her plans. She tends to avoid eye contact or stare blankly. Which of the following is the most likely diagnosis?

A Autism spectrum disorder
B Dissociative identity disorder
C Generalized anxiety disorder
D Neonatal asphyxia
E Schizoid personality disorder
A

A Autism spectrum disorder

The patient in this question most likely has autism spectrum disorder which is the DSM 5 category that consolidates autistic disorder, Asperger’s disorder, and pervasive developmen­tal disorder.

Autism spectrum disorder is characterized by atypical social development, including a lack of social awareness or inability to socialize, coupled with extreme behavioral rigidity. Although the cause is largely unknown, there are multiple etiologies associated including birth complications and genetic factors. During preschool years children with autism spectrum disorder may be asymptomatic. Later children start to have delayed use of nonverbal components of communication, limited attention, and do not respond to social communication demands. They can have normal language development.

Behavioral disturbances are also present, and other features include clumsiness, average or above average school performance, and comorbidity with psychiatric conditions. Clinical assessments and a formal diagnostic interview along with clinical judgement are the standard for diagnosing autism spectrum disorder.

424
Q

A 2-year-old girl is brought to the family medicine office for a well-child check. She was born full-term with no complications. Her development was normal for the first year. Her mother notes that recently the child has been less responsive to people, and that she seems to fixate on one toy at a time. She also does not respond when someone calls her, and often avoids eye contact. An appropriate diagnosis is made, and the parents are educated on their daughter’s condition. Which of the following is the most likely comorbidity that would be seen in this child?

A Delirium
B Dementia
C Epilepsy
D Conduct disorder
E Attention deficit hyperactivity disorder
A

C Epilepsy

Children with autism spectrum disorder (ASD) are at an increased risk to develop epilepsy, with a comorbidity of about 20%. Current guidelines do not recommend screening for epilepsy in children with ASD, but it is important for providers to be aware of the connection between the two.

425
Q

A 3-year-old girl comes to the office because of difficulty interacting with others. She has always had difficulty interacting with her peers and seems disconnected from others, preferring to play alone. When she was a year old, she began speaking and has a vocabulary typical for her age. Which of the following is the most likely diagnosis?

A Attention deficit/hyperactivity disorder
B Autistic spectrum disorder
C Childhood disintegrative disorder
D Fetal alcohol spectrum disorder
E Rett disorder
A

B Autistic spectrum disorder

Autism spectrum disorder without intellectual or language impairments (formerly known as Asperger syndrome), is characterized by difficulty socializing, normal intelligence, and no language deficits. Patients often come to the office at an early age because they appear disinterested and detached from others.

426
Q

A 3-year-old girl is brought to the pediatrician by her parents who are concerned that she is not developing normally. She does not talk and avoids eye contact. She prefers to sit and play blocks by herself rather than engaging with other children. They note that she will occasionally have violent outbursts in inappropriate situations. She is otherwise healthy. In the office, the patient sits quietly in the corner of the room stacking and unstacking blocks. Examination of the patient shows a well developed female with no physical abnormalities. Based on the patient’s presentation, which of the following is the most likely diagnosis?

A Autism spectrum disorder
B Cri-du-chat syndrome
C Fragile X syndrome
D Oppositional defiant disorder
E Rett syndrome
A

A Autism spectrum disorder

Autism spectrum disorder is characterized by impaired social interaction with significant language and cognitive delays. Onset is typically in early childhood, and diagnostic criteria includes restricted repetitive range of interests, behaviors and activities.

427
Q

A 9-year-old girl comes to the office because of difficulty interacting with others. She has always felt awkward and incapable in social situations and is described by others as distant or “professional sounding.” Her mood changes rapidly, especially in public, and she withdraws into herself. She has few interests but detailed knowledge of them. She follows the same schedule every day and becomes flustered when something disrupts this. She has an appropriate vocabulary, attained language milestones at appropriate intervals, and has no difficulties in schoolwork. Which of the following is the most appropriate treatment for her condition?

A Behavior modification therapy
B Dialectical behavioral therapy
C Fluoxetine
D Methylphenidate
E Quetiapine
A

A Behavior modification therapy

Autism spectrum disorder without intellectual or language impairment (previously Asperger syndrome), is treated primarily with behavioral therapy. Pharmacology does not have a role in treating the core features of this disorder and only has a role if there are co-existing psychiatric disorders.

428
Q

An 18-year old woman comes to your clinic because of excessive mood lability that recurs every month, for the past 3 months. She describes these episodes as week-long bouts of anger, irritability, and sadness that interfere with her concentration. She also experiences breast tenderness and swelling, joint pain, and other muscle aches. Which of the following is the most appropriate initial step in management?

A Aripiprazole
B Buspirone
C Fluvoxamine
D Phenelzine
E Triamterene
A

PMS or PMDD

SSRIs = 1st line treatment

C Fluvoxamine

Premenstrual dysphoric disorder is typically treated with exercise followed by selective serotonin re-uptake inhibitors (SSRIs) such as fluvoxamine. Diuretics are sometimes used off-label (Spironolactone)

429
Q

A 24-year-old woman presents to the clinic at the urging of her friends. She denies feeling ill and states she has never felt better and is currently “at the top of her game.” One of her friends accompanying her to the visit reports that she has been acting strangely. Over the last 2 weeks, the patient has been staying up all hours of the night, has had several new sexual partners, and has been running up incredible amounts of credit card debt from wild shopping sprees. Her past medical history is remarkable for depression, for which she has been treated for the last 4 months with paroxetine (Paxil). Physical examination reveals a mildly agitated woman who cannot seem to sit still but otherwise appears normal. Neuoropsychiatric examination is remakable only for a virtually continuous flow of accelerated speech that is impossible to interrupt. Which of the following is the most appropriate management at this time?

a. Add gabapentin (Neurontin) to the patient’s medical regimen
b. Discontinue paroxetine (Paxil) and follow closely
c. Change paroxetine to amitriptyline (Elavil) or desipramine (Norpramin)
d. Discontinue paroxetine (Paxil) and start lithium (Lithobid) immediately
e. Increase the dose of paroxetine (Paxil) and follow closely

A

d. Discontinue paroxetine (Paxil) and start lithium (Lithobid) immediately

430
Q

Which of the following medications is periodic monitoring of liver function tests and platelet counts recommended?

a. Chlorpromazine (Thorazine)
b. Haloperidol (Haldol)
c. Lithium
d. Valproate (Depakote)

A

d. Valproate (Depakote)

431
Q

SE that can occur with taking Lithium during pregnancy?

a) virilization
b) ebstein’s anomaly
c) growth restriction
d) hypertension
e) accessory digits
f) fetal demise

A

b) ebstein’s anomaly

432
Q

most likely SE with administration with Lithium?

a) hypothyroidism
b) ulcers
c) hepatotoxicity
d) RVH
e) conductive hearing loss

A

a) hypothyroidism

433
Q

most likely SE with administration with Lithium?

a) hypernatremia
b) heart block
c) DM II
d) cushing’s syndrome
e) drug-induced lupus
f) toxic megacolon

A

b) heart block

434
Q

SE with Lithium?

a) ulcerative colitis
b) hemolytic anemia
c) aplastic anemia
d) pseudomembranous colitis
e) pernicious anemia
f) movement (tremor)

A

f) movement (tremor)

435
Q

SE with lithium?

a) hirsutism
b) atypical depression
c) nephrogenic diabetes insipidus
d) conductive hearing loss
e) malar rash
f) hepatotoxicity

A

c) nephrogenic diabetes insipidus

436
Q

medication class of lithium?

a) mood stabilizer
b) anticoagulant
c) antioxidant
d) antimetabolite
e) antiemetic

A

a) mood stabilizer

437
Q

A 3-year-old boy is brought to the office by his parents for a regular health supervision examination. His condition has not changed significantly since his last visit 6 months earlier. He still prefers to play by himself and rocks back and forth constantly. The mother has finally adjusted to the fact that her son shows no signs of emotional attachment to her. On physical examination the boy is noncooperative and utters unintelligible mumbles when the healthcare provider approaches him. The healthcare provider fails to establish any eye contact with him. Suddenly the boy starts screaming and runs over to the wall and repetitively bangs his forehead against it.
Which of the following is the most likely diagnosis?

 	Asperger disorder
 	Attention deficit hyperactivity disorder
 	Autism
 	Mental retardation
 	Rett syndrome
A

Autism

This is a typical clinical presentation of autism, a developmental disorder characterized by impaired social relatedness, deficits in verbal and nonverbal communication, and unusual responses to the environment. It develops before 30 months and is more common in boys (4:1). The cause is unknown. Clinical features include failure to attach as an infant, delayed or absent social smile, and failure to anticipate interaction with the caretaker. Patients demonstrate a delay in verbal and nonverbal communication skills. Behavioral characteristics can also include obsessional or perfectionistic behavior. Stereotypical movements and a need for sameness and routine are characteristic. Outbursts of anger are common, as well as self-injurious behavior. Autistic children are content to play alone. They may or may not be mentally retarded. Treatment is behavioral and educational, and is tailored to meet individual needs. Pharmacotherapy may be used to treat some target behaviors. Prognosis is poor and a very small number of these children will grow up to be marginally self-sufficient. The vast majority end up institutionalized. A better prognosis is associated with patients who demonstrate functional speech and higher intelligence.

438
Q

A 16-year-old girl is brought to the emergency department after a night of drinking. Her friend reports that she has consumed approximately 700 mL of vodka over the past three hours. After vomiting again, she is a little bit more responsive with short groans to verbal stimuli. A few hours later, physical examination shows slurred speech and cold and clammy skin. The patient reports seeing double. Which of the following most accurately describes the symptom she described?

a) binocular diplopia
b) temporary diplopia
c) monocular diplopia
d) permanent diplopia
e) voluntary diplopia

A

b) temporary diplopia

Temporary diplopia is a common symptom occurring in the context of acute intoxication (with either alcohol or drugs) as well as head trauma (especially concussions).

The under-aged teenager girl is experiencing symptoms of acute alcohol intoxication. Alcohol poisoning occurs when someone has drank too much alcohol in a short period of time. The signs and symptoms of alcohol poisoning include vomiting, slow respirations, cold and clammy skin, and loss of consciousness. Fluids must be replenished to avoid serious complications. Diplopia is the subjective complaint of seeing two images instead of one. Temporary diplopia can also be caused by alcohol intoxication or head injuries (particularly concussion). It can also be a side effect of anti-epileptic drugs (phenytoin and zonisamide) and dissociative drugs (ketamine and dextromethorphan).

439
Q

A 65-year-old man is brought to the emergency department by ambulance because of chest pain and difficulty breathing. He was hit by a car after stumbling into the street at a busy intersection and sustained trauma to his chest. Medical history is significant for multiple hospitalizations for alcohol-related injuries and seizures. The patient is intubated in the emergency department for a flail chest segment and spends one week sedated in the intensive care unit. After a successful intubation, he is transferred to the floor where it is noted that he is confused, unsteady, and easily agitated. Physical examination shows asymmetric eye movements, most notably a lateral rectus palsy. The patient is oriented only to himself and is unable to walk without assistance. Which of the following is the most likely diagnosis?

A Delirium    
B Delirium tremens    
C Korsakoff psychosis    
D Traumatic brain injury    
E Wernicke encephalopathy
A

ataxia
ocular disturbance
confusion
= wernecke’s encephalopathy

Korsakoff = amnesia

Wernicke encephalopathy presents most commonly as ataxia, encephalopathy, and ophthalmoplegia (lateral rectus or conjugate gaze palsy), as this patient is showing signs of. If not treated, it can progress to anterograde/retrograde amnesia, nystagmus, and confabulations known as Korsakoff psychosis.

440
Q

A 42-year-old man comes to the emergency department for evaluation after a motor vehicle accident. Paramedics say he jumped in front of a car that was leaving a parking garage at low speed. Past medical history is significant for hepatitis C, cirrhosis, and alcoholism. Physical examination shows an intoxicated male with slurred speech and lethargy, unable to fully cooperate with the exam. Cervical spine is non-tender, and there are several superficial abrasions on his limbs. Cardiopulmonary exam shows no abnormalities. 8 hours after initial physical examination, the patient becomes febrile, diaphoretic, tachycardic, agitated, and tremulous while remaining fully alert and oriented. Which of the following is the most likely etiology of his current symptoms?

A Alcohol withdrawal    
B Akathisia    
C Delirium    
D Hepatic encephalopathy    
E Pulmonary embolism
A

A Alcohol withdrawal

Alcohol withdrawal presents soon after the cessation of alcohol ingestion in chronic alcoholics and begins first with symptoms like tremor, tachycardia, and hypertension.

Hepatic encephalopathy occurs in patients with liver dysfunction due to buildup of toxic metabolites in the brain. The classic exam finding is asterixis. This patient has cirrhosis, but not confusion or decreased level of arousal to suggest encephalopathy. Furthermore, hepatic encephalopathy does not usually cause autonomic hyperactivity (tachycardia, hypertension, diaphoresis).

441
Q

A 58-year-old homeless man is brought to the emergency department by the police. The police officers mention that they found the patient screaming near a shopping mall. When taken into custody, the officers mention that he was aggressive and kept claiming he was a diplomatic leader from Iceland. When asking the patient about his personal information, he looks apathetic, he has no recollection of who he is, where he is from, or what happened prior to being found by the police. Upon further interrogation, the patient seems confused, disoriented and has to be reminded several times that he is in the hospital. On physical examination, the patient looks malnourished he shows a strong alcoholic odor and facial hyperemia. His temperature is 36.6°C (97.8°F), pulse is 102/min, respirations are 20/min, blood pressure is 130/80 mmHg. The toxicology screen is positive for alcohol. Which of the following is the most likely diagnosis?

A Korsakoff's syndrome 
B Methanol poisoning
C Hepatic encephalopathy
D Alcohol use disorder 
E Wernicke encephalopathy
A

alcohol
no fever
tachycardia
hypertensive

he has no recollection of who he is, where he is from, or what happened prior to being found by the police = amnesia = Korsakoff’s syndrome

A Korsakoff’s syndrome

Korsakoff’s syndrome is often exacerbated by the neurotoxic effects of alcohol. Patients most often present with severe memory loss (anterograde/retrograde amnesia), confabulation (fabricated autobiographical memories), apathy, and lack of insight.

The mainstay treatment for patients with Korsakoff’s syndrome is high-dose parenteral thiamine followed by a maintenance treatment phase with vitamin and electrolyte supplementation. However, once there is neuronal damage in the thalamus, it can no longer be treatable with thiamine.

442
Q

A 56-year-old man with a long history of alcohol use disorder presents to the ER with multiple neurological problems and a history of multiple ER visits. The nurse sets him up with multiple fluids, including thiamine. If thiamine deficiency is presumed to be the reason for his presentation which of the following enzymes is most likely to be abnormally functioning?

A Dopamine hydroxylase
B Propionyl CoA carboxylase
C Pyruvate carboxylase
D Pyruvate dehydrogenase
E Thymidylate synthase
A

D Pyruvate dehydrogenase

Thiamine is involved with dehydrogenase reactions (PDH and alpha ketoglutarate dehydrogenase for TCA cycle); and transketolase for the HMP shunt.

443
Q

A 36-year-old man is reviewed on the surgical ward because of distress, fever, and diaphoresis for 2 hours. He was admitted for an urgent laparoscopic appendectomy yesterday, and said he felt well on the ward round this morning. Social history includes work as business man in a very high stress job. To cope with this stress, he drinks 4 glasses of wine every night. Temperature is 37.3°C (99.2°F), pulse is 125/min, respirations are 25/min, and blood pressure is 117/78 mm Hg. Examination shows a diaphoretic man who appears unwell. The surgical wounds are healing well and appear normal. ECG is obtained and shows an absence of P waves and irregular spacing between QRS complexes. Which of the following is the most likely diagnosis?

A Autonomic nervous system derangement
B Opioid withdrawal
C Pulmonary embolism
D Sepsis
E Systemic inflammatory response syndrome
A

A Autonomic nervous system derangement

Alcohol dependency changes autonomic regulation of baseline physiology and can lead to autonomic derangement when alcohol intake is suddenly reduced. Heavy alcohol consumption can temporarily suppress autonomic regulation of cardiac pacemakers.

444
Q

A 35-year-old man is brought to the emergency department by ambulance after he was found in an altered mental state outside of a liquor store. He is disoriented and cannot state the date or where he is. He is identified by his driver’s license (which is suspended) in his wallet and medical records show that he has a previous history of alcoholism. When asked what his name is, he is only able to mumble his first name. His temperature is 37.0°C (98.6°F), pulse is 95/min, respirations are 16/min, and blood pressure is 136/84 mm Hg. He walks with a wide-based gait and slow, short-spaced steps. Neurologic examination shows nystagmus evoked by horizontal gaze to both sides. Which of the following is the most likely diagnosis?

A Herpes simplex encephalitis 
B Korsakoff syndrome
C Marchiafava-Bignami syndrome
D Vitamin B12 deficiency
E Wernicke's encephalopathy
A

altered mental state
disoriented
alcoholism = thiamine deficiency
ataxia / nystagmus / ocular dysfunction

E Wernicke’s encephalopathy

Korsakoff syndrome is associated with isolated memory disturbance and confabulation. It is often seen in conjunction with Wernicke encephalopathy in patients who suffer from alcoholism.

doesn’t really have amnesia - just disoriented ; doesn’t know date or where he is, but doesn’t have loss of memory

445
Q

A 25-year-old Asian man comes to the emergency department because of flushing, diaphoresis, nausea, and vomiting for an hour. He states that the symptoms came on while he was at a friend’s birthday party. At the party, he drank 5 alcoholic beverages, but did not use any recreational drugs. He does not usually drink alcoholic beverages. Medical history is non-contributory. Vital signs are within the normal range. Appears very red/flushed, then back to normal skin tone next morning. Which of the following biochemical compounds is the most likely cause of this patients symptoms?

A Acetaldehyde
B Acetic Acid
C Dopamine
D Epinephrine 
E Serotonin
A

A Acetaldehyde

Alcohol flush reaction occurs commonly in Asian people who consume alcoholic beverages. The reaction is the result of an accumulation of acetaldehyde, a metabolic byproduct of the catabolic metabolism of alcohol, and is caused by an acetaldehyde dehydrogenase deficiency.

This syndrome has been associated with an increased risk of esophageal cancer in those who consume alcoholic beverages. It has also been associated with lower than average rates of alcoholism, possibly due to its association with adverse effects after drinking alcohol.

446
Q

A 59-year-old Caucasian man presents to the emergency room with ataxic gait and confusion. Physical examination shows he is mildly icteric. A positive fluid wave test is observed on abdominal examination, along with caput medusae. Lab results demonstrate a serum aspartate aminotransferase (AST) activity of 39 U/L and a serum alanine aminotransferase (ALT) activity of 54 U/L. Family history is positive for diabetes mellitus, Alzheimer’s disease, and ischemic stroke. You begin to administer glucose concurrently with a substance that you believe to be deficient in the patient. Which of the following cardiac abnormalities can be see in patients with the same deficiency?

A Aortic stenosis 
B Dilated cardiomyopathy 
C Hypertrophic cardiomyopathy 
D Mitral valve prolapse 
E Pericarditis
A

B Dilated cardiomyopathy

Vitamin B1 is integral to glucose metabolism, and Vitamin B1 deficiency can cause dilated cardiomyopathy is a characteristic feature of wet beriberi. Vitamin B1 deficiency is also often seen in patients with alcohol use disorder, and can lead to Wernicke’s encephalopathy and Korsakoff’s syndrome if untreated.

447
Q

A 61-year-old patient with a history of alcohol use disorder is also diagnosed with Wernicke-Korsakoff syndrome. This syndrome is a brain disorder due to thiamine (vitamin B1) deficiency composed of two different conditions. When symptoms of Wernicke encephalopathy are left untreated, the symptoms may progress to Korsakoff syndrome. Which of the following findings is most commonly associated with Korsakoff syndrome?

a) Alcohol withdrawal symptoms
b) Confusion and attention deficit
c) Demyelination causing gait ataxia
d) Hallucinations and inability to form new memories
e) Paresthesias in the lower extremities

A

d) Hallucinations and inability to form new memories

Korsakoff syndrome, which develops if Wernicke encephalopathy symptoms are left untreated, is the irreversible stage of Wernicke-Korsakoff syndrome. It is characterized by memory loss, personality changes, and confabulations.

Mnemonic for Wernicke-Korsakoff syndrome:
Wernicke wears a reversible COAT and Korsakoff covers with a black KAP.
Reversible COAT= reversible if treated, Confusion, Ophthalmoplegia, and ATaxia.
blacK KAP= “Konfabulation”, Amnesia, and Personality changes.

448
Q

A 34-year old man comes to the office because of confusion and disorientation for 12 hours. He is usually in good health, but has also had diarrhea for the past 2 days. Medical history is unremarkable. Social history includes incarceration at a medium security prison for 3 years and alcoholism. Temperature is 36.8°C (98°F), pulse is 87/min, respirations are 18/min, and blood pressure is 117/78 mm Hg. Physical examination shows glossitis and vesicle-like skin lesions distributed over his extremities. There is also desquamating, roughened skin on the hands as shown below. Which of the following is most likely the cause of this patients symptoms?

a) alcoholism
b) scabies
c) smoking
d) excessive hand washing
e) inadequate sunlight

A

a) alcoholism

Alcoholism is a possible cause of vitamin B3 (niacin) deficiency, also known as pellagra.

Pellagra commonly occurs in people from countries where maize is the staple diet. Pellagra has the “4Ds” symptoms of diarrhea, dermatitis, dementia, and death.

Glossitis is often caused by nutritional deficiencies
dermatitis = vesicle like skin lesions on extremities

449
Q

A 40-year-old woman is in the hospital recovering from a cholecystectomy performed 3 days ago that was complicated by cholangitis. She is being treated with IV piperacillin-tazobactam. She calls the nurse to her room because she says that her heart is racing. She also demands that someone come in to clean the pile of garbage off of the floor because it is attracting flies. The patient appears sweaty, distressed, and unable to remain still. She is oriented to person, but not place or time. Palpation of the abdomen shows no tenderness, rebound, or guarding. Her temperature is 38°C (100.4°F), pulse is 112/minute, respiration is 20/minute, and blood pressure is 150/90 mm Hg. Which of the following is the most likely diagnosis?

A Thyroid storm
B Hepatic encephalopathy
C Alcoholic hallucinosis
D Acute cholangitis 
E Delirium tremens
A

E Delirium tremens

Delirium tremens begins 2 to 4 days into alcohol withdrawal and is a life-threatening condition treated with IV benzodiazepines. Like alcoholic hallucinosis, DT is characterized by hallucinations, but they occur with disorientation and abnormal vital signs.

Alcoholic hallucinosis is not equivalent to delirium tremens. For one, alcoholic hallucinosis begins within 12 to 24 hours of abstinence, and resolves within 48 hours. Another difference is that the hallucinations are not associated with confusion; patients are aware that they are hallucinating. Their vital signs are usually normal.

DT typical begins 2 to 4 days after the last drink and lasts up to 5 days. Clinical manifestations include hyperventilation, hallucinations, tachycardia, hypertension, and hyperthermia. Unlike alcoholic hallucinosis, DT occurs later in the course of withdrawal and manifests with a clouded sensorium.
DT is a life-threatening condition requiring treatment, since fatal arrhythmias or complications such as pneumonia may occur.
Volume correction with isotonic IV fluid, thiamine, and glucose is standard. IV benzodiazepines are used to reduce psychomotor agitation and decrease neuronal activity

450
Q

A 36-year-old male with dementia of unknown etiology comes to the clinic to be evaluated for long term care. Physical examination shows he has aphasia, agnosia, and difficulty performing familiar tasks. His TSH, B12, folate, complete blood count, metabolic profile, VDRL, and rapid plasma reagin all show no abnormalities. A brain CT scan and MRI with contrast also show no abnormalities. Which of the following is the most likely diagnosis?

a) chronic alcohol use
b) NPH
c) pick’s disease
d) syphilis
e) uremia

A

a) chronic alcohol use

Alcohol use disorder can lead to dementia with common symptoms being memory impairment, aphasia, apraxia, agnosia, and disturbances in executive functioning. Exclusion of other causes of dementia is necessary to diagnose someone with alcohol-related dementia.

Pick’s disease also known as frontotemporal dementia is similar to Alzheimer’s disease but with frontal signs predominating (marked disinhibition and personality change). Brain imaging would be abnormal and show frontal/temporal atrophy and hypo-metabolism if this were the diagnosis.

451
Q

A 35-year-old man is brought to the emergency department because he was found obtunded and hypoactive on the side of the road 30 minutes ago. He says he is homeless, but can not remember when he last ate, or his medical history. Blood sugar level is normal. Examination shows an ulcer on the lateral aspect of the right heel (shown below). There is decreased sensation to light touch and pain around the site of the ulcer. Which of the following conditions is a risk factor for the development of this type of ulcer?

A Alcoholism
B Frostbite
C Insect bites
D Lymphoma
E Smoking
A

A Alcoholism

Alcoholic neuropathy is usually caused by chronic consumption of large amounts of alcohol. Over time, the nerves in the feet and hands can become damaged resulting in the same loss of sensation as that seen in diabetic neuropathy. The damage to these nerves is permanent.

452
Q

A 35-year-old man comes to the emergency department because of a seizure. The patient has been incarcerated for a day and hit his head on the floor during a single episode of seizing, lasting about 3 minutes, witnessed by prison staff. Past medical history is significant for gastroesophageal reflux disease. Physical examination shows a tachycardic, hypertensive, anxious, diaphoretic, tremulous man who is fully alert and oriented. There is evidence of urinary incontinence. Neurological exam shows tongue fasciculations and a resting tremor. Laboratory studies show:

Sodium: 136 mEq/L 
Blood urea nitrogen: 10 mg/dL 
Potassium: 4.5 mEq/L 
Creatinine: 0.7 mg/dL 
Chloride: 100 mEq/L 
Glucose: 75 mg/dL 
Bicarbonate: 15 mEq/L

Which of the following is the most likely etiology of this seizure?

a) alcohol withdrawal
B Hypoglycemia    
C Hyponatremia    
D Psychogenic    
E Traumatic subarachnoid hemorrhage
A

Alcohol withdrawal is initially characterized by agitation and tremors, but can progress to generalized seizures as the withdrawal worsens. This typically occurs 12-48 hours after the cessation of alcohol ingestion.

453
Q

A 68-year-old woman is brought to the emergency department by the police. The police officers mention that they found the patient “fortune-telling” and scamming people in the street, in addition to, having a half empty bottle of vodka in her handbag. She offensively kept claiming that she is a certified fortune teller. When asking the patient about her name, she looks apathetic and has no recollection of who she is or what happened prior to being found by the police. The patient seems confused and has to be reminded that she is in the hospital. Her medical records show a history of long-standing alcohol use. On physical examination, the patient looks disheveled with a strong alcoholic odor, facial hyperemia, and symmetrical lower extremities paresthesias. Her temperature is 36.6°C (97.8°F), pulse is 102/min, respirations are 20/min, blood pressure is 120/90 mmHg.

Which of the following best describes this patient’s findings?

A Confabulation 
B Delirium 
C Dissociative amnesia 
D Delusional disorder 
E Malingering
A

Wernicke-Korsakoff syndrome
Korsakoff - amnesia
Wernicke’s encephalopathy = ataxia / confusion / ocular dysfunction

A) CONFABULATION

Confabulation is characterized by a memory disturbance, defined as the production of fabricated, distorted or misinterpreted memories about oneself or the world, without the conscious intention to deceive.

Delirium is characterized by the acute onset of fluctuating decline of cognitive function, attentional deficits, and disorganized behavior. It typically involves perceptual deficits, altered sleep-wake cycle, and psychotic features (i.e., hallucinations, delusions). Although this patient seems confused and disoriented on interrogation, there are no other accompanying signs of delirium.

454
Q

A 33-year-old homeless woman comes to the emergency department because of hallucinations. Security guards found her unconscious at the parking lot of a mall. After paramedics arrived and stabilized her, she became very aggressive and started screaming at them to help her “get rid of the ants crawling all over [her].” Which of the following conditions is most likely to cause this type of behavior?

A Alcohol withdrawal
B Brain tumor
C Epilepsy
D Hypoacusis
E Schizophrenia
A

A Alcohol withdrawal

Tactile hallucinations, specifically of small insects crawling on the surface of the skin, are classically seen in cases of alcohol withdrawal.

Many conditions can cause multiple types of hallucinations. For example, alcohol withdrawal syndrome can cause tactile, visual, or auditory hallucinations; however with this condition, visual and tactile are more common than auditory hallucinations.

Patients with schizophrenia can have auditory, gustatory, olfactory, visual, and/or tactile hallucinations. However, auditory hallucinations, specifically hearing voices, are the most common. The most common types of tactile hallucinations in patients with schizophrenia are of the skin stretching, intimate touch, and small animals.

455
Q

A 50-year-old male, with a 25 year history of alcohol use disorder, comes to the emergency department complaining of a loss of appetite, abdominal pain and fever for the past 24 hours. He states that he had consumed twelve beers and a bottle of vodka two days ago. Laboratory studies are obtained, and results show the following:

Sodium: 135 mEq/L
Potassium: 3.5 mEg/L
Alanine aminotransferase: 250 U/L
Aspartate aminotransferase: 450 U/L

Six-months later, the same patient presents with diplopia, nystagmus, confusion, and loss of balance when walking. Which of the following is the most likely associated finding in this patient?

A Folic acid deficiency      
B Hyperammonemia     
C Memory loss      
D Vitamin B1 deficiency      
E Vitamin B3 deficiency
A

Wernicke-Korsakoff syndrome

D Vitamin B1 deficiency

456
Q

A 30-year-old Caucasian man is brought to the emergency department by the police because of acute cocaine intoxication. He has a history of asthma. His temperature is 38°C (100.4°F), pulse is 110/min, respirations are 16/min, and blood pressure is155/80 mm Hg. Physical examination shows a euphoric and diaphoretic male with dilated pupils. An electrocardiogram is obtained and shows tachycardia with normal sinus rhythm. The patient is admitted to the hospital for cocaine intoxication. Which of the following symptoms can the patient expect to experience as they begin to withdraw from the cocaine?

A Increased appetite
B Increased sympathetic stimulation
C Lacrimation
D Rhinorrhea
E Seizures
A

A Increased appetite

usage = decreased appetite
withdrawal = increased appetite

sympathetic stimulation = use, not withdrawal

lacrimation / rhinorrhea = opioid withdrawal
As patients withdraw from an opioid intoxication they experience dysphoria, lacrimation, rhinorrhea, weakness, diaphoresis, pilo-erection, myalgias, nausea, and vomiting. Note that opioid withdrawal symptoms are not life threatening.

seizures = alcohol withdrawal
Patients who are chronic alcoholics or benzodiazepine abusers can experience seizures as they withdraw. This can lead to a life threatening condition known as delirium tremens. Delirium tremens is characterized by altered mental status and seizures. Treatment is with benzodiazepines.

457
Q

A 23-year-old man comes to the emergency department because of complaints that insects are crawling on him and attempting to eat his skin. His temperature is 37.8°C (100.1°F), pulse is 150/min, respirations are 22/min, and blood pressure is 180/110 mm Hg. His pupils are dilated, and he is otherwise unresponsive to questioning. Physical examination of the skin shows no abnormalities. Which of the following is the most likely cause of his presentation?

A Central nervous system malignancy 
B Delusional disorder
C Drug Overdose
D Schizophrenia
E Temporal lobe encephalitis
A

dilated pupils = stimulants
tactile hallucinations
= cocaine

C Drug Overdose

458
Q

A 51-year-old man with a 20-year history of alcoholism comes to the emergency department because of abdominal pain, fever, and loss of appetite for the past 24 hours. He also complains of anxiety and insomnia. Additionally, he had a seizure 10 minutes prior to arrival. He says he has decided to stop drinking; he last drank 2 days ago, consuming 12 beers and a bottle of vodka. Laboratory studies show:

Sodium: 137 mEq/L
Potassium: 3.4 mEg/L
Alanine aminotransferase: 230 U/L
Aspartate aminotransferase: 450 U/L

Which of the following is the most appropriate treatment?

a) methadone
b) disulfiram
c) buspirone
d) chlordiazepoxide
e) carbamazepine

A

d) chlordiazepoxide

Benzodiazepines are the drug of choice for alcohol withdrawal syndrome. Chlordiazepoxide is a common choice, as well as diazepam and lorazepam.

459
Q

A 36-year-old man comes to the emergency department because of progressively worsening abdominal pain for the past 12 hours. The pain began in the epigastric region but now encompasses his entire abdomen. He admits he is a heavy drinker and went on a binge-drinking episode with his friends yesterday. Abdominal examination shows diffuse rebound tenderness and a negative Murphy sign. Serum lipase is 674 U/L. He is given appropriate initial treatment and is transferred to the inpatient medicine service. Ten hours later, the patient becomes increasingly agitated, develops with worsening tachycardia and hypertension, and experiences new-onset visual hallucinations. Which of the following is the most appropriate next step in management?

a) fomepizole
b) haloperidol
c) lorazepam
d) metoprolol

A

c) lorazepam

Alcohol withdrawal is characterized by increased patient agitation, autonomic derangement, deficits of orientation, and visual hallucinations. Treatment is with benzodiazepines titrated to the patient’s level of overall agitation.

460
Q

A 40-year-old woman presents to the hospital for an elective cholecystectomy. She has a 20-year history of alcoholism and drinks 4 L of cask wine every two days. She admits to continuing this pattern of drinking for the past week and only managing to abstain yesterday. Which of the following pharmacological agents will most likely play a key role in her management?

A Chlorpromazine and folic acid
B Diazepam and thiamine
C Disulfiram and thiamine
D Esomeprazole and ranitidine
E Haloperidol and multivitamin
A

B Diazepam and thiamine

Alcohol withdrawal syndrome is a potential medical emergency for patients who abruptly cease after an extended history of alcohol consumption. Benzodiazepines are the drug of choice to treat and prevent withdrawal symptoms; vitamins are often co-administered to treat nutrient deficiency and prevent additional complications; thiamine prevents encephalopathy.

461
Q

A 65-year-old male comes to the emergency department because of chest pain and difficulty breathing. History reveals the patient was hit by a car and sustained a diffuse trauma to his chest. Past medical history reveals multiple hospitalizations for alcohol related injuries and seizures. The patient is intubated in the emergency department for a flail chest segment and spends one week sedated in the intensive care unit. After a successful intubation, he is transferred to the floor where it is noted that he appears to be confused, has unsteady gait, and is easily agitated. Physical examination reveals asymmetric eye movements, most notably a lateral rectus palsy. The patient is oriented to himself and is unable to walk without assistance. Which of the following is the most appropriate treatment in this patient?

A An immediate dose of haloperidol injection
B Give him a high-dose cobalamin immediately
C Give this patient folate
D IV thiamine
E STAT lorazepam

A

D IV thiamine

Patients with alcohol use disorder would benefit from repletion of all of the vitamins and nutrients listed as answer choices. However, to treat Wernicke encephalopathy, intravenous thiamine is indicated. Wernicke encephalopathy is classically characterized as encephalopathy with confusion, gait ataxia, and ophthalmoplegia. If not treated with thiamine, it can progress to anterograde/retrograde amnesia, nystagmus, and confabulations known as Korsakoff psychosis. Korsakoff psychosis is irreversible.

Patients with alcohol use disorder are at risk of developing Wernicke-Korsakoff syndrome. Treat this syndrome by administering intravenous thiamine to all patients at risk or showing signs of this syndrome.

462
Q

A 34-year-old male is admitted to the emergency department after being brought to the hospital by his friends. The patient had been drinking alcohol “excessively” at a party and was found unresponsive on a couch. His pulse 150/min, blood glucose concentration is 60 mg/dL, and serum anion gap is normal. Laboratory studies show:

Serum potassium: 3.4 mEq/L
Serum magnesium: 1.4 mEq/L
Serum calcium: 7 mg/dL
Serum phosphate: 2.1 mg/dL

Based on the patient’s history and test results, workup would most likely reveal which of the following?
Elimination tool

A Crystalluria 
B Elevated isopropanol levels
C Kidney failure
D Photophobia
E Positive serum ketone test
A

E Positive serum ketone test

This patient is suffering from alcohol intoxication induced by the ingestion of ethanol (alcohol). In the human body, ethanol is first metabolized to acetylaldehyde by alcohol dehydrogenase, which is found in many tissues in the body. Subsequently, acetylaldehyde is metabolized to acetate by acetylaldehyde dehydrogenase, which is found predominantly in the liver.

Since the symptoms of ethanol intoxication are similar to other forms of alcohol intoxication, definitive diagnosis typically relies on a history of ethanol consumption and an ethanol blood test. Due to the increase in acetate, a serum ketone test (or nitroprusside test) will also normally come back positive.

463
Q

A 50-year-old man with altered mental status is brought to the emergency department by his estranged wife after being found stumbling and mumbling incoherently in his home. He lives alone and has not been seen by acquaintances for some time. His speech is unintelligible, and he is unable to walk steadily. He is lucid for a few moments but cannot remember who he is, or where he lives. He has a long history of alcoholism but urine toxicology is negative for alcohol ingestion, benzodiazepines, barbituates, opiates, or other substances. His symptoms are most likely to be resolved with the administration of what substance?

A Folate
B Glucose
C Iron
D Thiamine
E Vitamin B6
A

D Thiamine

464
Q

A 28-year-old Caucasian man comes to the emergency department because of watery eyes and rhinorrhea. He is also complaining of excess weakness and abdominal pains. He said he has vomited two hours ago and has diffuse myalgias. He has run out of medication for his back pain, and he needs to refill his prescription as soon as possible. He is anxious and afraid his symptoms are getting worse, and without intervention, he may suffer permanent harm. He has a history of similar admissions. Which of the following signs and symptoms would help to confirm that the patient is not fabricating the experience of having opioid withdrawal symptoms?

A Lacrimation and rhinorrhea
B Nausea and vomiting
C Piloerection and pupillary dilation
D Tachycardia and hypertension
 E Yawning and weakness
A

watery eyes (lacrimation)
rhinorrhea
abdominal pain / myalgias
back pain - pain meds

C Piloerection and pupillary dilation

A mnemonic to remember the symptoms of opioid withdrawal is ARMY FINDS:
Aches (muscular)
Rhinorrhea (or lacrimation)
Mood-dysphoric (depression)
Yawning
Fever
Insomnia
Nausea/vomiting
Diarrhea
Sweating

Also, be sure to remember pupillary dilation and piloerection.

465
Q

A 72-year-old man comes to the clinic for a checkup after recently falling asleep at a red light while driving. He has had fatigue for the past year which forces him to take multiple unplanned naps throughout the day. At night, he has difficulty falling asleep. However, once he does, he is able to stay asleep until his alarm rings in the morning. When napping, he does not notice any hallucinations or dreaming. He also denies a loss of muscle tone when laughing. His wife says that when he does sleep, he does not snore and seems to breathe normally. He walks his dog for 30 minutes each afternoon, maintains a dark bedroom at a comfortable temperature, and practices breathing relaxation techniques prior to bedtime. He takes no medications and denies alcohol, nicotine, or caffeine consumption. Which of the following medications is the most appropriate on an as-needed basis?

A Amitriptyline
B Estazolam
C Flurazepam
D Trazodone
E Zolpidem
A

insomnia
good sleep hygiene
initial insomnia

Sleep-onset insomnia, or difficulty falling asleep, is often treated with short-acting sedative-hypnotics, such as zolpidem, on an as-needed basis. They are first-line agents used in conjunction with non-pharmacological options.

Sedative-hypnotics such as zolpidem are excellent first-line agents for the treatment of sleep-onset insomnia, in which patients have difficulty falling asleep, but no trouble remaining asleep thereafter.

466
Q

A 50-year-old man comes to the office because of insomnia for 8 months. The patient states that he has difficulty falling asleep almost every night but does not awaken after he is asleep. To compensate for his insomnia he now takes multiple naps throughout the day. Medical history is non-contributory. Examination shows a man who appears tired. His body mass index is within the normal range, and there are no obvious airway issues. Which of the following factors would be a strict contraindication for initiating medical therapy?

A Actively abusing alcohol
B Cardiac arrhythmias
C Narrow-angle glaucoma
D Priapism
E Untreated obstructive sleep apnea
A

A Actively abusing alcohol

increased toxicity and central nervous system depression.

Patients with prior cardiac arrhythmias should not be prescribed amitriptyline to avoid further cardiac arrhythmic potential

Benzodiazepines such as estazolam and flurazepam should be avoided in patients with a history of angle-closure glaucoma. These agents do not possess anticholinergic activity but have very rarely been associated with increased intraocular pressure.

Trazodone, a serotonin antagonist and reuptake inhibitor (SARI) has a strong association with priapism (persistent and painful erection of the penis) and should be avoided in all persons suffering from this diagnosis in both the present or past.

Benzodiazepines such as estazolam and flurazepam should be avoided in patients with a history of untreated obstructive sleep apnea. This is because of their muscle relaxant action, which may cause respiratory depression in susceptible individuals.

467
Q

A 25-year-old man comes to the clinic because of daytime sleepiness for the past 3 months. He works in a banking office and says he often dozes off at his desk. His coworkers have woken him up a few of times at work, and he says he often does not recall even falling asleep. He also says his daytime naps have been increasing in frequency. He describes these naps as a “sudden desire to fall asleep, sometimes involuntarily”. He says he sleeps an average of 6-8 hours a night. His medical record reveals no anemia. A sleep study 2 months ago shows no sleep apnea. Which treatment will most likely help this patient?

A Amitriptyline
B Atomoxetine
C Clonazepam
D Modafinil
E Repeat the sleep study, then prescribe CPAP if sleep apnea is found
A

Modafinil is a central nervous system stimulant. It is one of the first-line medications approved for the treatment of narcolepsy.

468
Q

A 12-year-old boy comes to the office because of excessive daytime sleepiness. His mother says his grades are suffering because he cannot stay awake during class. The patient says sometimes when he gets really nervous or excited about something, he feels as if he cannot move his legs and might even fall down. His past medical history is noncontributory. Physical examination shows no abnormalities. Which of the following is the most appropriate pharmacological therapy?

A Hydroxyzine
B Imipramine
C Modafinil
D Zolpidem

A

C modafinil

patient has narcolepsy
cataplexy = gets really nervous or excited about something, he feels as if he cannot move his legs and might even fall down.

469
Q

A 55-year-old woman comes to the clinic because of an inability to move while falling asleep. She says it feels as if she were tied down to her bed. The episodes are accompanied by the sound of voices and the sense that someone was lying next to her. She says she feels more sleepy lately and mentions that once she caught herself starting to drift into an oncoming lane of traffic while driving. Medical history is significant for insomnia, for which she recently began taking a new medication as part of a clinical trial. She is unable to recall the name of the drug. Which of the following molecules is most likely affected by the drug and likely causing this patient’s side effects?

a Enkephalin
b GABA
c Glutamate
d Orexin
e Histamine
A

d Orexin

Suvorexant (Belsomra) is a medication for the treatment of insomnia that has produced narcolepsy-like adverse effects in some people.

The patient is taking medication for insomnia but is experiencing daytime sleepiness in addition to hypnagogic hallucinations and paralysis. These side effects are characteristic symptoms of narcolepsy.

470
Q

A 35-year-old man comes to the office because of sleepiness. He says every afternoon between 3:00-6:00 PM, he gets very tired for no reason. He dozes off in meetings, while watching TV, and even while driving. “Gallons of iced tea” help him get through the day. He says, “sometimes I cannot move when I wake up” and “I have very vivid dreams right before I fall asleep.” His past medical history is noncontributory. Physical examination shows no abnormalities. Which of the following additional symptoms is this patient likely to report?

A Brief, sudden, generalized loss of muscle tone precipitated by strong emotion
B Cold intolerance, dry skin, weight gain, constipation
C Intense urge to move legs while trying to fall asleep
D Random episodic mood changes
E Snoring at night

A

A Brief, sudden, generalized loss of muscle tone precipitated by strong emotion = Cataplexy

patient has Narcolepsy

“sometimes I cannot move when I wake up” = sleep paralysis
daytime sleepiness
vivid dreams
= Narcolepsy